Histopathology Flashcards

(1289 cards)

1
Q

Blood supply of the liver

A

Dual blood supply: portal vein, hepatic artery

How well did you know this?
1
Not at all
2
3
4
5
Perfectly
2
Q

What are the cells of the liver?

A

Hepatocytes

Bile ducts

BVs

Endothelial cells

Kupffer cells

Stellate cells

How well did you know this?
1
Not at all
2
3
4
5
Perfectly
3
Q

What is the basic structure of the liver?

A

Hepatic lobule

At the centre are the terminal branches of the hepatic vein. The angles of the hexagon are formed by the portal tracts that contain 3 structures: BD, hepatic artery and portal vein

How well did you know this?
1
Not at all
2
3
4
5
Perfectly
4
Q

Where are centrilobular hepatocytes found?

Where are periportal hepatocytes found?

A

Located near the terminal hepatic vein i.e. zone 3: more metabolically active.

Located near the portal tract, receive blood rich in nutrients and O2.

How well did you know this?
1
Not at all
2
3
4
5
Perfectly
5
Q

What organ is this?

A

Liver

How well did you know this?
1
Not at all
2
3
4
5
Perfectly
6
Q

What are the functions of the liver?

A

Metabolic: involved in glycolysis, glycogen storage, glucose synthesis, amino acid synthesis, FA synthesis, lipoprotein metabolism. Drug metabolism.

Protein synthesis: make all circulating proteins except gamma globulins, including albumin, fibrinogen, and coag factors

Storage: glycogen, viamins A, D and B12 in large amounts. Small amounts of vitamin K, folate, Fe, Cu

Hormone metabolism: activates vit D. Conjugation and excretion of steroid hormones (oestrogens, GCs)< peptide hormone metabolism (insulin, GH< PTH)

Bile synthesis: 600-1000ml daily.

Immune function: antigens from gut reach liver via portal criculation and phagocytosed by kuppfer cells.

How well did you know this?
1
Not at all
2
3
4
5
Perfectly
7
Q

Which protein is not synthesised in the liver?

A

Gamma globulins

How well did you know this?
1
Not at all
2
3
4
5
Perfectly
8
Q

Which vitamins are stored in the liver?

A

A, D , B12 (large amount)

K (small amount)

How well did you know this?
1
Not at all
2
3
4
5
Perfectly
9
Q

What is this structure

A

Portal tract

How well did you know this?
1
Not at all
2
3
4
5
Perfectly
10
Q

What cellular changes occur following injury to the liver?

A

Loss of hepatocyte microvilli

Activation of stellate cells

Deposition of scar matrix

Loss of fenestrae

Kuppfer cell activation

How well did you know this?
1
Not at all
2
3
4
5
Perfectly
11
Q

Definition of cirrhosis

A

Involves whole liver

Fibrosis

Nodules of regenerating heaptocytes

Distortion of liver vascular architecture: intra and extra hepatic shunting of blood

How well did you know this?
1
Not at all
2
3
4
5
Perfectly
12
Q

How can cirrhosis be classified?

A

According to nodule size: micro or macronodular

According to aetiology: ETOH/insulin resistance, viral heaptitis etc

How well did you know this?
1
Not at all
2
3
4
5
Perfectly
13
Q

What are the complications of cirrhosis?

A

Portal HTN

Hepatic encephalopathy

HCC

How well did you know this?
1
Not at all
2
3
4
5
Perfectly
14
Q

What are the causes of acute hepatitis

A

Viruses

Drugs

How well did you know this?
1
Not at all
2
3
4
5
Perfectly
15
Q

What is the process shown here?

What is this known as?*

A

Acute hepatitis

“Spotty necrosis”

How well did you know this?
1
Not at all
2
3
4
5
Perfectly
16
Q

What are the causes of chronic hepatitis?

What does the grade of chronic hepatitis refer to?

Stage?

A

Viral

Drugs

Autoimmune

Grade= severity of inflammation

Stage= severity of fibrosis

How well did you know this?
1
Not at all
2
3
4
5
Perfectly
17
Q

What does this show?

A

Portal hepatitis

How well did you know this?
1
Not at all
2
3
4
5
Perfectly
18
Q

What does this show?

Also known as?

A

Interface hepatitis

“piecemeal necrosis”

How well did you know this?
1
Not at all
2
3
4
5
Perfectly
19
Q

What does this show?

A

Liver fibrosis

How well did you know this?
1
Not at all
2
3
4
5
Perfectly
20
Q

What is the pathological course of hep C infection

A

Acute: asymptomatic, 15-30% clear the infection

Fibrosis: (F0-F3), virus unlikely to clear without treatment, people may still be asymptomatic. Fatigue, URQ discomfort, transient appetite loss. End stage symptoms: itching, depression, impaired memory.

Scarring can be mild to severe. Extrahepatic Cxs: cryoglobulinaemia, glomerulonephritis, kerratoconjunctivitis sicca.

Cirrhosis (F4): Symptoms due to hepatic insufficiency and portal HTN: ascites, oesophageal varices, hepatic encephalopathy. Consider OLT

HCC: early stage: OLT, Sx, percutaenous ablation. Intermediate: TACE, Terminal

How well did you know this?
1
Not at all
2
3
4
5
Perfectly
21
Q

Spectrum of ETOHic liver disease?

A

Fatty liver

Alcoholic hepatitis

Cirrhosis

How well did you know this?
1
Not at all
2
3
4
5
Perfectly
22
Q

What does this show?

A

Fatty liver disease

How well did you know this?
1
Not at all
2
3
4
5
Perfectly
23
Q

What does this show?

What are the arrows pointing to?

A

Alcoholic hepatitis

Mallory bodies

How well did you know this?
1
Not at all
2
3
4
5
Perfectly
24
Q

What does this show?

What are the features?

A

Alcoholic liver cirrhosis

Micronodular

How well did you know this?
1
Not at all
2
3
4
5
Perfectly
25
What does NAFLD look like? What causes it?
Histologically looks like alcoholic liver disease Due to insulin resistance associated with raised BMI and DM
26
What is PBC? Features Abs?
Primary biliary cirrhosis Bile duct loss associated with chronic inflammation: granulomas AMAs (anti-mitochondrial)
27
What does this show?
Primary biliary cirrhosis Bile duct loss with granulomas
28
What is PSC? What are the features? With what is it associated?
Primary sclerosing cholangitis Periductal bile duct fibrosis leading to loss Associated with UC Increased risk of cholangiocarcinoma ERCP Dxic
29
What does this show?
Primary sclerosing cholangitis
30
What is haemochromatosis? Cause? Cxs?
Genetically determined increased gut iron absorption Gene on chromosome 6 Parenchymal damage to organs secondary to Fe deposition-\> "bronzed diabetes"
31
What does this show?
Haemochromatosis
32
What is haemosiderosis?
Accumulation of Fe in macrophages Seen following blood transfusion
33
What is Wilson's By what is it caused Cxs?
Accumlation of Cu due to feailure of excretion by hepatocytes Genes on chromosome 13 Accumulates in the liver and CNS-\> hepatolenticular degenration
34
What is a histopathological stain for Wilson's?
Rhodanine
35
What disease is this?
Wilson's wth a rhodanine stain.
36
What is a clinical sign in Wilson's?
Keyser-Fleischer rings
37
What are the features of autoimmune hepatitis?
Interface hepatitis with plasma cells. Anti-SMA Abs Responds to steroids.
38
What is Alpha-one antitrypsin deficiency?
Failure to secrete alpha-one antitrypsin Leads to intra-cytoplasmic inclusions, hepatitis and cirrhosis
39
Alpha-1 antitrypsin
40
What are the causes of hepatic granulomas?
Specific: PBC, drugs General: TB, sarcoid
41
What are the benign liver tumours?
Liver cell adenoma bile duct adenoma haemangioma
42
What is the most common type of liver tumour?
Secondary \> primary
43
What are the primary liver malignancies?
HCC Hepatoblastoma Cholangiocarcinoma Haemangiosarcoma
44
What does this show?
HCC
45
With what is cholangiocarcinoma associated? Whence can it arise? How is the prognosis?
PSC, helminth infection, cirrhosis, congenital liver abnormalities, Lynch syndrome Type II Intrahepatic ducts, extrahepatic ducts (including GB) Poor
46
What does this show?
Cholangiocarcinoma
47
What are the clinical features of hepatic adenoma?
Associated with OCP Presents with abdo pain/ intraperitoneal bleeding Resection if symptomatic, \>5cm or if no shirnkage when stopping OCP
48
What are the features of haemangioma:
Most common benign lesion No Rx
49
What are causes of HCC? Ix?
Hepattis B + C, ETOHic cirrhosis. Haemocrhomatosis, NAFLD, aflatoxin, androgenic steroids. Ix: alpha fetoprotein, USS
50
What is Lynch Syndrome caused by? What are the classifications?
MMR defect Type 1: HNPCC Type 2: Extracolonic
51
What are the common sites to metastasise to the liver?
GI, breast or bronchus
52
What are the major causes of cirrhosis
ALD NAFLD Chronic viral hepatitis (B+/-D, C) Autoimmune Biliary causes:PBC and PSC Genetic: haemochromatosis (HFE gene), Wilsons (ATP7B gene), A1AT, galactosaemia, glycogen storage disease Drugs e.g. methotrexate
53
What are the causes of micronodular cirrhosis?
Uniform liver involvement (\<3mm) Alcoholic hepatitis, biliary tract disease
54
What are the causes of macronodular cirrhosis?
Variable nodule size (\>3mm) Viral hepatitis, Wilson's, A1AT
55
What is the pathological process in cirrhosis?
Chronic inflammation causes stellate cell activation in the space of Disse They become myofibroblasts that initiate fibrosis by deposition of collagen in space of Disse Myofibroblasts contract, constricting sinusoids and increasing vascular resistance. Undamaged hepatocytes regenerate in nodules between fibrous septa
56
What is the name of the score used to indicate Px in liver cirrhosis? On what is it based?
Modified Child's Pugh Ascites Encephalopathy Bilirubin Albumin Prothrombin time
57
What are the thresholds for the Child pugh score?
\<7: 45% 5 year survival 7-9: 20% \>10: \<20%
58
59
What is portal HTN secondary to? What happens?
Increased vascular resistance in liver Hyperdynamic circulation Sodium retention and plasma volume expansion When portal pressure \>10-12 mmHG, venous system dilates and collateral vessels form
60
Where are the collateral vessels found in portal HTN? GORLDRA
GO junction Rectum L renal vein Diaphragm Retroperitoneum Anterior abdominal:umbilical vein
61
What are the causes of portal HTN?
Pre-hepatic: Portal vein thrombosis: Factor V leiden _Hepatic_ Pre-sinusoidal: Schistosomiasis, PBC, sarcoid Sinusoidal: cirrhosis Post-sinusoidal: veno-occlusive disease Post-hepatic: Budd-Chiari syndrome
62
Causes of Budd-Chiari Syndrome Mx of Budd-Chiari
30% idiopathic Thrombophilia OCP Leukaemias Compression by renal tumours, HCC RTx Thrombolyse, treat underlying cause. TIPS
63
What is a TIPS?
Transjugular Intrahepatic portosystemic shunt
64
What are the macroscopic and microscopic features of Hepatic steatosis
Large, pale, yellow, greasy Accumulation of fat droplets in hepatocytes Fully reversible if ETOH avoided
65
What are the macroscopic and microscopic features of Alcoholic hepstitis
Large, fibrotic liver Hepatocyte ballooning and necrosis due to accumulation of fat, water and proteins Mallory bodies Fibrosis Seen acutely after heavy night of drinking. Can range from asymptomatic to fulminant liver failure
66
What are the macroscopic and microscopic features of Alcoholic cirrhosis
Yellow-tan, fatty, enlarged, transforms into shrunken, non-fatty brown organ Micronodular cirrhosis (\<3mm)
67
What are the features of NAFLD?
Hepatic steatosis in non-ETOHs Most common cause of chronic liver disease in West NAFLD: simple steatosis NASH: steatosis and hepatitis, can progresss to cirrhosis
68
What are the features of autoimmune hepatitis?
Common with other autoimmune disease e.g. coeliac, SLE\< RA, thyroiditis, Sjogrens, UC 78% female, young and postmenopausal HLA-DR3
69
What is Type 1 autoimmune hepatitis?
ANA, anti-SMA\< anti-actin, anti-soluble liver Ag
70
What is Type 2 autoimmune hepatitis?
Anti-LKM (liver, kidney, microsomal)
71
Mx of autoimmune hepatitis
Immunosuppression until transplant. Disease recurs in 40%
72
Features of PBC Epidemiology LFTs Abs Hx Mx
Auotimmune inflammatory destruction of medium sized intra-hepatic bile ducts-\> cholesstasis-\>slow development of crrhosis F\>M 10:1 Peak incidence 40-50y/o raised ALP, cholesterol, IgM, hyperbilirubinaemia (late) AMA in \>90% US scan shows no bile duct dilatation Histology: bile duct loss with granulomas Fatigue, pruritus and abdo discomfort. Skin pigementation, anthelasma, steatorrhoea, Vit D malabsorption, inflammatory arthropathy Ursodeoxycholic acid in early phase
73
Features of PSC Epidemiology Bloods Dx Cx
Inflammation and obliterative fibrosis of extrahepatic and intrahepatic bile ducts-\> multi-focal stricture formation with dilation of preserved semgnets M\>F Peak incidence at 40-50y/o IBD associated (UC) Raised ALP\< several associated auto-Ig (p-ANCA) US: bile duct dilatation ERCP: shows beding of the bile ducts due to multifocal strictures Increased incidence of cholangiocarcinoma
74
Haemochromatosis Epidemiology Pathophysiology Histology Signs/Symptoms Ix Treatment
Genetic 40-50y/o **Autosomal recessive:** mutated HFE gene 6p21.3-\> fe absorption which deposits in liver, heart, pancreas, adrenals, pituitarry, joints, skin-\> fibrosis Fe deposits in liver stains with Prussian blue Skin bronzing (melanin deposition), DM, hepatomegaly, cardiomyopathy, hypogonadism, pseudogout Ix: raised Fe, ferritin, transferrin sa \>45%, decreased TIBC Venesection Desferrioxamine 30% with cirrhosis -\> HCC
75
Wilson's disease Epidemiology Pathophysiology Histology Signs/Symptoms Ix Treatment
v. rare 11-14y **Autosomal recessive**: mutated ATP7B (Chr13) encodes Cu transporting ATPase-\> decreased biliary Cu excretion and deposition in liver, CNS, iris. Cu stains with Rhodanine stain, Mallory bodies and fibrosis on microscopy. Liver disease: acute hepatitis, fulminant liver failure or cirrhosis. Neuro disease: parkinsonism, psychosis, dementia Decreased serum caeruloplasmin, decreased serum Cu, increased urinary Cu Lifelong penicillinamine
76
A1AT Epidemiology Pathophysiology Histology Signs/Symptoms Ix Treatment
**Autosomal dominant:** A1AT accumulates in hepatocytes -\> intracytoplasmic inclusions -\> hepatitis. lack of A1AT in lungs -\> emphysema Intracytoplasmic inclusions of A1AT which stain with periodic acid Schiff Kids: neonatal jaundice Adults: emphysema and liver disease Ix: redcued A1AT absent alpha globulin band on electrophoresis.
77
What are the functions of bone?
Mechanical: support and site of muscle attachment Protective: vital organs and bone marrow Metabolic: Ca reserve
78
What is the composition of bone? Inorganic Organic
Inorganic: (65%) Ca hydroxyapatite (10Ca 6PO4 OH2) body store for 99% of body Ca 85% of P, 65% Na and Mg Organic: bone cells and protein matrix
79
What is the structure of bone medial to lateral?
Medulla, Cortex, Periosteum
80
What is the structure of bone proximal to distal?
Diaphysis Metaphysis Epihpyseal line Epiphysis Subchondral bone Articular cartilage
81
What are the features of cortical bones?
Long bones 80% of bony skeleton Appendicular 80-90% calcified Mainly mechanical and protective
82
What are the features of cancellous bone?
Vertebrae and pelvis 20% of skelton Axial 15-25% calcified Mainly metabolic Large surface
83
What type of bone is this?
Cortical
84
What type of bone is this?
Cancellous
85
What are the types/classifications of bone?
Woven/lamellour Anatomically: flat/long bones - intramembranous and anedochondral ossification Trabecular (cancellous)/compact (cortical)
86
What are the arrows pointing at?
87
What is the function of osteoBlasts?
Build bone by laying down osteoid
88
What is the function of osteoclasts?
Multinucleate cells of macrophage family Resorb bone
89
What are osteocytes?
Osteoblast like cells which sit in lacunae in bone
90
What is a lacuna?
In histology, a lacuna is a small space containing an osteocyte in bone or chondrocyte in cartilage. The Lacunae are situated between the lamellae, and consist of a number of oblong spaces.
91
92
Draw a diagram showing the modulation of osteoclastogenesis
RANK is expressed on the surface of osteoclast lineage cells RANKL expressed on MSCs of ostebolast lineage and on B and T Ls When RANKL binds to RANK this causes osteoclast precursor cell to differentiate, increasing bone resorption OPG competed with RANK for RANKL
93
How do tumour cells mediate local growth of tumour in bone?
Oncogene products produced by tumour cells metastasising to bone influence the bone cells to resorb bone and promote local growth of the tumour. This is mediated by the RANK /OPG signalling pathway.
94
What type of malignancy often causes more bone growth than destruction?
Prostate carcinoma
95
What is metabolic bone disease?
Disordered bone turnover due to imblanace of chemicals in body: vitamins, hormones, minerals Net effect: reduced bone mass-\> pathological #
96
What are the 3 main categroies of metabolic bone disease?
Non-endocrine e.g. age-related Endocrine: e.g. Vit D, PTH Disuse osteopenia
97
What are the histological characteristics in metabolic bone disease Where must the biopsy be taken?
Static parameters: cortical thickness and porosity, trabecular bone volume, thickness, number & separation of tranbeculae Biopsy from iliac crest
98
What is this showing? What is the pink?
Trabecular (cancellous) bone The trabecular bone, the grey is the marrow
99
What can be used to study the hisodynamic parameters of bone?
Fluorescent tetracycline labelling
100
What is the aetiology of osteoporosis?
1o: age, post-menopause 2o: drugs, systemic disease
101
What is the pathogenesis of osteoporosis? High turnover? Low turnover?
Pathogenesis: low intiial bone mass or accelerated bone loss can reduce bone mass below # threshold 90% of cases due to insufficient Ca intake and post-menopausal oestrogen deficiecny High turnover: increased bone resorption Low turnover: decreased bone formation
102
What are the influencing factors for osteoporosis?
Nutrition and social practices: ETOH, smoking, malabsorption, Vit C & DD Endocrine Immobilisation Iatrogenic: corticosteroids, long term heparin or phenyotin therapy, casstration, XS thyroid therapy
103
What are the risk factors for osteoporosis?
Advanced age Female Smoking ETOH Early menopause LT immobility Low BMI Poor Diet/malabsorption Thyroid disease Low testosterone Chronic renal disease Steroids
104
What is the UK societal impact of osteoporotic #s?
50% of patients cannot live independently post # 20% die
105
What is the presentation of osteoporosis?
Back pain and # Classic #s: wrist (Colle's) hip: NOF and intertrochanteric pelvis \>60% vertebral #s are asymptomatic with compresion # usually in T11-L2
106
Ix in osteoporosis
Lab: Serum Ca, P and ALP (usually N) Urinary Ca Collagen breakdown products Imaging Bone densitometry
107
What are the cut offs for bone densitometry?
T score: 1-2.5SD below normal peak bone mass= osteopenia \>2.5SD= osteoporosis
108
What is osteomalacia?
Defective bone mineralisation: either Vit DD or PO4 deficiency
109
What are the sequelae of osteomalacia?
Bone pain/tenderness # Proximal weakness Deformity
110
What does this show?
Osteomalacia (Rickets) ## Footnote There is a widening of the growth plate which is also irregular and the femur and tibia become bowed as the child starts to walk and the legs have to weight bear.
111
What is the abnormality What disease?
Horizontal # in Looser's zone Osteomalacia
112
Features of hyperPTHism
* Excess PTH * increased Ca + PO4 excretion in urine * hypercalcaemia * hypophosphataemia * skeletal changes of osteitis fibrosa cystica
113
What is the differnec between 1o and 2o hyperparathyroidism?
1o: parathyroid adenoma (85-90%) or chief cell hyperplasia 2o: chronic renal deficiency, Vit DD, malabsorption
114
Hypercalcaemic mnemonic
* Symptoms Mnemonic * Stones (Ca oxalate renal stones) * Bones (osteitis fibrosa cystica, bone resorption) * Abdominal groans (acute pancreatitis) * Psychic moans (psychosis & depression)
115
What is renal osteodystrophy?
Comprises all the skeletal changes of chronic renal disease Increased bone resorption: osteitis fibrosa cystica Osteomalacia Osteosclerosis Growth retardation Osteoporosis
116
What does this XR show?
Osteitis fibrosa cystica: increase bone resorption
117
Lab features of renal osteodystrophy
* PO4 retention – hyperphosphataemia * Hypocalcaemia as a result of decreased vit D * 2o hyperparathyroidism * Metabolic acidosis * Aluminium deposition
118
What is Paget's? What are the 3 phases?
Disorder of bone turnover Osteolytic Osteolytic-osteosclerotic Quiescent osteosclerotic
119
What is this?
Paget's disease of bone
120
Features of Paget's?
Onset \>40y M=F Rare in asians/africans Mono-ostotic in 15%, remained polyostotic Aetiology is unknown Familial pattern shows autosomal pattern with incomplete penetrance Parvomyxovirus type particles have been seen in Pagetic bone
121
What are the 5 most commonly affected sites in Paget's?
Vertebrae Skull Pelvis Femur Tibia
122
Clinical features of Paget's?
Pain micro# Nerve compression Skull changes may put medulla at risk +/- haemodynamic chanes, HF Development of sarcoma in area of involvement in 1%
123
What does this show?
Paget's disease affecting tibia
124
What are the indications for bone biopsy?
Suspected osteomalacia Diagnostic classification of renal osteodystrophy Osteopaenia: in young patients (\<50) or associated with abnormal Ca metabolism Classification of hereditary childhood bone disease
125
XR. histological and biochemical findings for Osteoporosis
No XR Loss of cancellous bone N Ca, N PO4, N ALP
126
XR and histological findings for Osteomalacia
Looser's zones: pseudo#s, splaying of metaphysis Excess of unmineralised bone: osteoid N/L Ca, L PO4, H ALP
127
XR and histological findings for 1o hyperparathyroidism
Brown's tumours, Salt and pepper skull, Subperiosteoal bone resorption in phalanges Osteitis fibrosa cystica: marrow fibrosis and cysts aka Brown tumour Raised Ca,, Low/N PO4, Raised/N ALP
128
What are Brown's tumours?
The brown tumor is a bone lesion that arises in settings of excess osteoclast activity, such as hyperparathyroidism. It is not a true neoplasm, as the term "tumor" suggests; however, it may mimic a true neoplasm.[1]Brown tumours are radiolucenton x-ray.
129
Hyperparathyroidism: Brown's tuour of the hands
130
Salt and pepper sign of the calvaria refers to multiple tiny hyperlucent areas in the skull vault caused by resorption of trabecular bone in hyperparathyroidism. There is loss of definition between the inner and outer tables of the skull and a ground-glass appearance as well as spotty deossification.
131
XR and histological findings for 1o Paget's?
Mixed lytic and sclertoic. Skull: osteoprorosis circumscripta, cotton wool. Vertebrae: picture frame, ivory vertebrae Pelvis: sclerosis and lucency Huge osteclosts w \>100 nuclei, mosaic (like jigsaw) pattern of lamellar bone N Ca + PO4, +++ALP
132
Osteoporosis circumscripta cranii (also known as osteolysis circumscripta) refers to discrete radiolucent regions of the skull on plain radiographs. They are often seen in context of the lytic (incipient-active) phase of Paget's disease of the skull, but may be observed in other circumstances as well, e.g. hyperparathyroidism, leontiasis ossea 9.
133
Cotton wool skull- Pagets
134
Picture frame vertebra Paget's
135
Ivory vertebra Paget's
136
What are the features of astrocytes?
Most abundant Anchor neurones by numerous projections, regulate environment e.g. ions, neurotransmitters and form BBB
137
Features of oligodendrocytes?
Coat axons with their cell membranes forming myelin
138
Features of ependymal cells?
Line the cavities of the CNS, make up walls of the ventricles, create and secrete CSF and beat cilia to help move the CSF. Act as neuronal stem cells
139
Radial glia features
Arise from the neuroepithelial cells in embrogenes, act as the scaffold for new neurones
140
Features of Schwann cells?
Provide myelination to PNS
141
Features of satellite cells?
Small cells that surround the neurones in sensory sympathetic and parasympathetic ganglia, helping to regulate the environment
142
Features of enteric glial cells
Intrinsic ganglia of the GIT
143
Function of microglia
Act as macrophages
144
What are the glial cells?
Astrocytes Oligodendrocytes Microglia
145
Which cells interface with the CSF?
Ependyma Choroid plexus epithelium Meninges
146
Which CNS cells interface with blood?
Endothelium and pericytes
147
What is the most common CNS tumour?
Metastatic neoplasm
148
What are the principle malignancies causing neurometastases?
Leukaemias and lymphomas, more so in young. Lung, breast and malignant menaloma
149
What are the features of brain mest? Pathology? Symptoms?
May involve the meninges as well as the parenchyma Well demarcated solitary or multiple lesions with surrounding oedema Neurological efffects and raised ICP
150
What are the 5 most common malignancies leading to CNS tumours in adults? BLCPB
Breast Lung Large Bowel Prostate Bladder
151
What are the most common group of CNS primary tumours?
Astrocytomas
152
Astrocytomas Age of onset Pathology Grading Symptoms Px
Any age, elderly usually worse An infiltrative growth pattern in the cerebral hemispheres Grading is based on the degree of dy/dx with histological grade an important predictor of behaviour Raised ICP and focal neuro signs Depends on site, grade and age of the patient
153
What are the types of astrocytoma?
Pilocytic astrocytomas: more common in children but can occur any age. Anaplastic astrocytoma Glioblastoma multiforme: necrotic, poorly differentiated tumour
154
Features of oligodendroglioma Px
Most common in adulthood Usually in the cerebral hemisphere and are soft and gelatinous Better demarcated than infiltrating astroctomas Calcifcaition common Px less predictable, dependant on grade site, patient age, cytogenetics etc
155
Features of ependyomas? Symptoms? NB AA?
Occur at any age Most offen within the ventricular cavities or within the canal of the SC Usually well demarcated Symptoms depend on site: intracranial: hydrocephalus or raised ICP Anaplastic astrocytomas/variants often also disseminate through the sub arachnoid space
156
Where are ependyomas found in \<20y/o? In adults?
In te ventricular cavities In the SC
157
Features of primitive neuroepithelial neoplasms
Composed of embryonal primitive cells Most common in children Undifferentiated lesions Px: most survive 5y or more
158
Medulloblastoma features
Lesion of the cerebellum in the first 2 decades of life Leads to raised ICP and cerebellar signs
159
Features of primary CNS lymphoma
Increased since AIDs
160
Features of meningiomas
Derive from meningioethlial cells Most occur in the brain parenchyma but can occur in the cranial vault and cord. Usually adults Increased in NF2 Usually lobulated lesions, sharp interface between tumour and parenchyma Overlying skull may be thickened or invaded by the tumour Symptoms: raised ICP with focal neurological signs
161
NF2 in brain tumours?
Meningioma
162
Ventricular tumour/hydrocephlaus in brain tumours?
Ependyoma
163
Indolent/childhood in brain tumours?
Pilocytic astrocytoma
164
Soft gelatinous calcified brain tumour
Oligodendroma
165
What genetic conditions predispose to CNS malignancy?
VHL NF 1 + 2 Li-Fraumeni Gorlin Turcot syndrome
166
Sings of supratentorial tumours
Focal neurological deficit Seizure Altered mental status Headache
167
Signs of subtenotrial tumours
Cerebellar ataxia Long tract signs CN palsy
168
What are the neurosurgical approaches to a CNS malignancy
Stereotacic biopsy Open biopsy Craniotomy for debulking
169
CNS tumours: Grade 1 2 3 4
LT survival Causes death in \>5y Death in \<5y Deaith within 6m-1y
170
Features of infiltrative gliomas
Account for 80% of gliomas. Astrocytomas- Oligodendrogliomas and mixed
171
What is the most aggressive infiltrative glioma?
De novo glioblastoma
172
What mutation is responsible for 80% of diffuse astrocytomas?
IDH1
173
What is secondary glioblastoma?
From progression of a lower grade astrocytoma
174
What is the proportion of diffuse astrocytomas found in the crebellum?
10%
175
Which has the better Px, astrocytomas or oligodendroglioma? What is important?
Oligodendroglioma Resection
176
* Usually 1st and 2nd decade - 20% of CNS tumours below 14 years and 15% between 14-18 years * Often cerebellar, optic-hypothalamic, brain stem * Often cystic. Always contrast enhancement * They can disseminate in the subarachnoid space (es: follow nerve roots) * Compressive margins (never diffuse infiltration) * Variable histological features * Very often Rosenthal fibres and granular bodies * Hallmark: Piloid “hairy” cell
Pilocytic astrocytoma
177
* Rare (0.5 per 100,000 year, in children) * 75% arise in the vermis in children and hemispheric in adults * Present with cerebellar signs, cranial hypertension
Medulloblastoma
178
* 24-30% primary intracranial tumours * Incidental in up to 10% of post-mortem * Usually adults – rare in patients younger than 40 (more aggressive) * Focal symptoms (seizure, compression) * Any site of craniospinal axis
Meningioma
179
What is crucial in the assesmnet of grade of CNS primaries?
Mitotic activity
180
What is occuring in this image?
Pseudoinvasion along Virchow-Robin's space
181
What is the commonest cause of CNS disease? What is the most common cause of this?
Infarction Cerebral atherosclerosis
182
What proportion of strokes are infarctive?
70-80%
183
Where do thrombosis occur in the CNS?
Atherosclerosis affects the larger extracerebral vessels worse, often near the carotid bifurcation or in the basilar artery
184
Which part of the CNS do emboli affect?
Intracerebral atery, usually from the heart or atherosclerotic plaques Usually occurs in the MCA branches
185
What is a watershed infarction?
Occurs on border zone, hypoperfusion of the most distal edges of the blood supply. Not necessarily occlusive. ACA and MCA most at risk Site: distally is affected more than proximal Dependant on the anastamoses around the zone
186
Cortical border zone
Between ACA and MCA
187
Internal border zone?
Between LCA and MCA
188
Cortical border zone
Between MCA and PCA
189
Def: TIA
Under 25hrs, self-limiting vascular obstruction, emboli and or platelet-fibrin aggregates
190
Px of TIA
1/3rd will get a significant infarct within 5 years
191
Def: intraparenchymal haemorrhage Presentation Site
Haemorrhage into the brain substance, usually rupture of one of the small intraparenchymal vessels Raised ICP, rapid LOC Basal ganglia, abrupt onset
192
Cause of ICH
50% HTN Weakening of the walls, accelerated atheroscleroris in LV, hyaline atherosclerosis in smaller vesseles. Clotting disorders, neoplasms, amyloid, vasculitis and vascular malformations also contribute
193
SAH most common cause Most cmmon sites
Rupture of Berry aneurysm (present in 1% of the population) 80% ICA bifurcation, 20% vertebrobasilar circulation 30% of patients they are multiple
194
Berry aneurysm \>6-10mm \>25mm
Greatest risk of rupture Mass lesion
195
Presentation and Px of SAH
Sudden onset, severe headache, vomiting, LOC 50% die within a few days, worse Px if there were warning bleeds
196
Types of vascular malformations Significance
AVM Capillary telangectasias Venous Angiomas Cavernous angiomas Important cause of ICH and developmental abnormalities
197
EDH features
Meningeal artery rupture which lies between the dura mater. Associated with skull fractures
198
Presentation of EDH
Lucid interval followed by progressive LOC. Bleeding is arterial and there is subsequent mass effect
199
SDH features
Between the dura and the arachnoid mater. Disruption of the bridging veins. Associated with rapid change of head velocity which leads to tearing of the veins. Usually clear Hx of tumour caused by raised ICP. Venous bleeding therefore slower development than EDH.
200
Presentation of SDH
There is a less good history of trauma and it is associated with brain atrophy. Often causes vague alteration in mental state rather than classical features of raised ICP
201
Features of Parenchymal injury
Occurs due to sudden acceleration or deceleration with sufficient force to tear nerve cell process in the cerebral white matter
202
Presentation of parenchymal injury Sequelae?
Concussion, transient LOC and neurological deficit. Sometimes with seizrues with recovery over hours or days. Diffuse axonal injury: causes most post-traumatic dementia and with hypoxic ischaemic injury is the leading cause of persistent vegetative state
203
Coup vs contracoup
Contustion at site vs away from site of impact Need to know the MOI
204
What is Duret's Haemorrhage
Bleeding in the ventral and paramedian part of the upper brainstaim (pons and midbrain)
205
Features of cerebral oedema
Gyral flat and te sulci are obliterated
206
Causes of brain oedema
Anything that can damage the BBB Can be classified as Vasiogenic: where the integrity of the BBB is disrupted which is aggravated by the lack of lymphatic CNS drainage Cytotoxic: secondary to cellular injury e.g. due to general hypoxic ischaemic injury.
207
Features of herniation
Due to raised ICp, can force the brain against unyielding bony wall
208
Subfalcine herniation
Cingulated gyrus displaced under falx cerebri
209
Transtentorial hernia
Uncal gyral, medial temporal lobe compressed against the free margin of the tentorium cerebelli Results in compression of the PCA and oculomotor nerve
210
Tonsilar herniation
Cerebellar tonsils herniate through the foramen magnum causing brainstem compression
211
Stroke Symptoms Vascular territories Ix Mx
Sudden onset, FAST, numbness, loss of vision, dysphagia MCA most common CT/MRI (infarct vs haemorrhage) Ix for vascular risk: BP, FBC, ESR, U&E, glucose, lipids, CXR, ECG, carotid doppler Aspirin +/- dipyridamole Thrombolytics (\<3h) +/'- caroit dendarterectomy LT: treat HTN, reduce lipids, anticoagulate
212
TIA Symptoms Vascular territories Ix Mx
\<24hrs, amaurosis fugax, carotid bruit Any however characteristically embolic atherogenic debris from the carotid artery travels to the opthalmic branch of the internal caroitd Cartoid USS IX for vascular risk as in stroke As for stroke except no thrombolysis
213
What proportion of SAH are from ruptured Berry Aneurysm What conditions increase risk of Berry Aneurysm
85% PKD, Ehler's Danlos and patients with coarctation Also associated with AVMs, capillary telangectasias, venous and cavrnous angiomas
214
Haemorrhage brain injury classification
Non traumatic: ICH SAH Traumatic: EDH SDH
215
6 types of brain herniation
Uncal Central (transtentorial) Cingulate (subfalcine) Transcalvarial Upward Tonsilar
216
Cerebral oedema
217
What are the two types of hydrocephalus?
Communicating/non-obstructive Non-communicating/obstructive
218
Causes of non-obstructive hydrocephalus
Impaired reabsorption of CSF: Normal pressure hydrocephalus Hydrocephalus ex-vacuo
219
Features of normal pressure hydrocephalus
Associated with elevated CSF causing increased ICP and increased ventricular size Thought to be scondary to an increase in fluid levels. Infection, tumours, trauma and haemorrhage
220
Features of hydrocephalus ex-vacuo
Atrophy causing secondary enlargement of the ventricle and subarachnoid space Not the result of increased CSF production pressure but a compensatory enlargement of ventricle due to loss of brain parenchyma
221
Causes of obstructive hydrocephalus
Actual obstruction to CSF flow Adhesion, external compression, interventricular cysts and tumours
222
Def of stroke This excludes
A stroke is a clinical syndrome characterised by rapidly developing clinical symptoms and / or signs of focal, and at times global loss of cerebral function, with symptoms lasting more than 24 hours or leading to death, with no apparent cause other than that of vascular origin (Hatano, 1976). This definition includes stroke due to cerebral infarction, primary intracerebral haemorrhage, intraventricular haemorrhage, and most cases of subarachnoid haemorrhage It excludes subdural haemorrhage, epidural haemorrhage, or intracerebral haemorrhage (ICH) or infarction caused by infection or tumour.
223
Two types of cerebral ischaemia
Focal: defined bascular territory Global: failure of systemic circulation
224
NB re atypical brain infarct
Atypical refers to odd anatomy – not classical vascular territory Clinical features and neuroimaging can be misleading. “Atypical brain infarct may mimic tumors”
225
Mx of AVM
Surgery, embolisation, Radiosurgery
226
Features of AVM
* Occur anywhere in the CNS * Becomes symptomatic between 2nd and 5th decade (mean age 31.2 years) * Present with haemorrhage, seizures, headache, focal neurological deficits * High pressure – MASSIVE BLEEDING!!! * Seen at angiography * Risk of bleeding 1.3-3.9% yearly * Risk of re-bleeding 6.0-6.9% during the first year * Morbidity after rupture 53-81% - high in eloquent areas * Mortality 10-17.6%
227
Features of cavernous angioma
“Well-defined malformative lesion composed of closely packed vessels with no parenchyma interposed between vascular spaces” Anywhere in the CNS Usually symptomatic after age 50 Pathogenesis unknown Rarely multiple – familial (linkage chrom. 7p, 7q, 3q) Present with headache, seizures, focal deficits, haemorrhage A few feeding vessels Low pressure – RECURRENT BLEEDINGS Angiographically usually negative Therapy: surgery
228
Histopathological differentiation between infarct and haemoorhage
INFARCT 1. Tissue necrosis (stains) 2. Rarely haemorrhagic 3. Permanent damage in the affected area (except for penumbra) 4. No recovery HAEMORRHAGE 1. Bleeding 2. Dissection of parenchyma 3. Less macrophages 4. Limited tissue damage (periphery) 5. Partial recovery
229
Lesions in fatal non-missile head injury
Primary Skull fracture 75% Surface contusions 95% Diffuse axonal injury (DAI) 30% Intracranial haematoma 60% Secondary Brain swelling 53% Ischaemic brain damage 55% Infection 4% Due to raised ICP 75%
230
Intracranial haematoma
66% of fatal non-missile head injury 10% extradural 56% subdural, subarachnoid, intracerebral or burst lobe surgical evacuation
231
232
SDH
233
EDH
234
235
What are connectomes?
Functional conective areas in the brain
236
What are the central neurotransmitters
GABA: inhibitory Glutamate: excitatory Ach, dopamine, serotonin
237
What are microglial cells?
Resident macrophage population constituting 20% CNS cells. Have motile processes. Involved in immune surveillance and tissue reomdelling (synaptic stripping) Cytotoxic in neurodegeneration and neuroinflammation
238
Def: neurodegenerative disease
Progressive, irreversible condition leading to neuronal loss Often caused by intra or extracellular accumulation of a misfolded protein. Usually sporadic resulting in dementia. Common in the ageing population
239
Def: neuroinflammatory disease
Conditions characterised by an innate and/or adaptive inflammatory response. Can be reveersible, progressive irreversible
240
Def: dementia
A serious loss of global cognitive ability in a previously unimpaired person beyond what might be expected from normal ageing. Can be a static event e.g. from TBI or due to neurodegeneration.
241
Dx of dementia
Difficult to diagnose on symptoms alone, some have a distinct clinical phenotype which gives a high probability of a dx e.g. PD. Based on neuroimaging, brain biopsy or PM examniation
242
Exmaples of misfolded proteins implicated in dementias?
Tau, beta-amyloid, alpha-synuclein, Huntingtin, PrP, Fus
243
Features of AD
Dense deposits around neurones (neuritic plaques) Twisted bands of fibre (NFTs) Begins after 5th-6th decade, pathogenesis unclear.
244
Pathological findings in AD
Senile plaques: complex spherical structures involving the grey matter. Diffuse plaques are amorphous and seen in ageing brains. Neuritic plaques consist of clusters of radially orientated abnormal axons and dendrites= dystrophic dendrites
245
Features of NFTs
Ubiquitin abnormally accumulates inside cells, indicates a disease process. These protein accumulations= inclusion bodies NFTs are aggregates of hyperphosphorylated Tau proteins.Intracellular structures composed of two filaments wound in a double helix
246
What conditions features Tau?
AD, progressive SNP, some other uncommon conditions
247
Features of Tau
Normal component of the neuronal cytoskeleton, stabilises MTs. 6 isoforms. Function is regulated by kinases. Unphosphorylated or hyperphosphorylated Tau fails to bind to MTs, accumulating in NFTs and neuritic plaques
248
Senile plaque
249
NFT
250
Risk factors for AD
Age FHx T21 Head trauma PD and depression
251
Dx of AD
Clinical suspicion MRI and PET scans helpful Role of biopsy unclear Dx and PM is definitive
252
Features of Beta amyloid
Derived from APP which is a ubiquitously expressed membrane molecule. Cleaved by secretases which are regulated by Presenilin 1 Amyloid accumulation results from defective APP cleavage
253
Beta amyloid
254
What fragment of amyloid accumulates in AD?
Fragment 39-42
255
What is the role of ApoE in AD?
Produced by astrocytes and mediates phospholipid and cholesterol mobilisation ApoE4 has a high affinity to beta amyloid and less activity. ApoE4 is a risk factor for late onset sporadic AD
256
Rx in AD Classes, e.g.
Acetylcholinesterases: Tacrine (Cognex(), Donepezil, Rivastigmine. Produce mild benefits without influencing progression nAChRs: galantamine Glutamate antagonist: memantine (also used in vascular dementia)
257
DDx in Parkinson
Parkinsonian syndromes: Primary: PD Secondary Parkinson-plus syndrome Familial Neurodegenerative diseases
258
Primary parkinsonism=
PD (sporadic, familial)
259
Causes of secondary parkinsonism
Drug induced: dopamine antags and depletor Hemiatryophy-hemiparkinsonism Normal pressure hydrocephalus Hypoxia Infectious: postencephalitic Toxin: e.g. MPTP Trauma Tumour Vascular: multiinfarct state
260
Causes of parkinson plus syndromes
Cortical-basal ganglionic degeneration Dementia sydnromes: AD, LBD, frontotemporal dementia Lytico-Bodig (ALS) Multiple system atrophy syndromes: Shy Drager, OPCA, MND parkinsonism Progressive pallidal atrophy PSN Palsy
261
Familial Neurodegenerative diseases causing parkinsonism
Hallervoden-Spatz Huntingoton Lubag Mitochondrial cytopathies with Striatal Necrosis Neuroacanthocytosis. WIlsons
262
Features of PD
Usually sporadic, develops after 50y Pathogenesis unclear
263
Symptoms of PD
Attributed to cell death of the dopamine producing neurones in the SN Need to lose 80-85% of the dopaminergic neurones and deplete dopamine levels by 70% before they appear. Cell death in substantita nigra results in depigmentation Lewy Bodies found in cell bodies and Lewy neurites in neuronal projections Alpha-synuclein and ubiquitin positive
264
Clinical features of PD
Resting tremor, rigidity, bradykinesia, autonomic dysfunction, dysphagia Psychiatric: hallucinations, anxiety and dementia L-DOPA responsiveness
265
What is alpha-synuclein
Small protin associated with synaptic membrane, accumulates and has toxic effect. Mutations reported in instances of familial parkinson's
266
Lewy Bodies
267
Causes of demyelinating lesions
Viral infections: PML (progressive multifocal leukoencephalopathy caused by JC virus in immune deficiencies Genetic: leukodystrophies Autoimmune: MS, acute haemorrhagic encephalomyeltisi, acute disseminated encephalomyelitis Nutritional: central pontine myleniosis
268
Function of myelin
Produced by oligodendrocytes, fundamental for axon conduction, highly susceptible to damage. Lipid rich insulating membrane
269
Features of MS
Peak age: 20-40y Assocaited with some HLAs Usually presents with focal symptoms, optic neuritis and poor coordination
270
What are the types of MS
Relapsing remitting: recovery less severe, evolves into secondary progressive agter years Primary progressive (10%): no recovery after episodes of demyelination: severe Progressive relapsing
271
What is neuromyeltitis optica
Rare variant of MS Aka Devic disease Autoimmune disease against the SC and optic nerves with auqaporin 4 attacked
272
What is Balo disease?
Rare variant of MS Rapidly progressive with concentric scoliosis
273
Pathology of MS
Loss of myelin, lesions centred by veins with sharpy margins Axonal preservation in early plaques Active plaques- glial scar Remyleinating shadow plaques Infiltration of macrohpages. Cortical involvement is the leading cause of disability, CI leading to dementia, related to inflamm rather than demyelination
274
Patholocial protein in: AD
Tau, beta-amyloid
275
Patholocial protein in: LBD
Alpha synuclein, ubiquitin
276
Patholocial protein in: Corticobasal degeneration
Tau
277
Patholocial protein in: Frontotemporal dementia
Tau
278
Patholocial protein in: Pick's disease
Tau
279
Imaging findings in AD
Generalised atrophy of the brain Widened sulci Narrowed Gyri Enlarged ventricles (most marked in the temporal and frontal lobes with loss of cholinergic neurones)
280
Features of LBD
Psychological disturbances occur early, day to fluctuations in cogntiive performance Vsiual hallucinations Spontaneous motor signs of Parkinsonism Recurrent falls and syncope
281
Myelin basic protein and proteo-lipid protein=
MS
282
Features of multiple system atrophy Types
Degenerative neurological disorder characterised by features that can present in a similar manner to Parkinson's but show a poort response to parkinson's medication Shy Drager Stratonigral Olivopontocerebellar
283
Shy Drager
MSA: autonomic dysfunction
284
Striatonigral MSA
Difficulty with movement
285
Olivopontocerebellar MSA
Difficulty with balance and coordination
286
Features of vCJD
vSporadic neuropsychiatric disorder vPatients \<45 yrs old vCerebellar ataxia vDementia vLonger duration than CJD vLinked to BSE
287
Neuropathology of AD
* Extracellular plaques * Neurofibrillary tangles * Cerebral amyloid angiopathy (CAA) * Neuronal loss (cerebral atrophy)
288
Cortical atrophy- AD
289
Diagnotics gold standard in PD
alpha-synuclein immunostaining
290
Functions of the kidney
Excretion of metabolic waste products and foreing chemicals Regulation of fluid and electrolyte balance, acid base balance Hormone secretion: EPO, Renin and 1,25 cholecaliferol
291
What are the two types of glomerulaur disease
Failure to: Filter an adequate amount of blood resulting in a lack of waste product excretion Failure to maintain a barrier function leading to the loss of protein and/or blood cells in the urine
292
What must be distinguished in renal disease
Syndrome: e.g. ARF, nephrotic etc. Morphological changes: glomerulonephritis, thrombotic microangiopathy Aetiologies: SLE, amyloidosis, drugs and infetions
293
What are immune complexes in the context of renal disease? Antigens Rate Stie
Composed of a lattice work of Ab and Ag, may become deposited in te glomerulus and lead to an inflammatory response-\> complement activation and stimulation of inflammatory cells through Fc receptors May be endogenous e.g. SLE or exogenous e.g. derived from an infective organism Rate: occur at different rates: rapidly progressive glomerulonephritis vs slow onset Site may bary but glomerular injury is determined by the immune complex location
294
Congenital diseases of the kidney
Bilateral/unilateral agenesis Ectopic e.g. pelvic Horseshoe (usually fused at the lower pole)
295
Features of Adult PKD
Causes 10% of ESRF Cysts arise from all portions of the nephron Renal failure develops from 40-70y Genes: PKD1 and PKD2
296
Features of acquired cystic disease
Cysts develop in the kidneys of ESRF on dialysis Carcinoma can develop in 7% in 10y
297
What is acute renal failure? What does it result in? Biochemically?
Rapid loss in golmerular filtration and tubular function Results in an abnormal water and electrolyte balance Reduced GFR manifestated as rasied serum creatinine (Normal= \<1.3) and urea (10-20mg/dL) May r3esult in acidosis, hyperkalaemia and fluid overload
298
How can disease of the kidney be classified? e.g.?
According to the site of the nephron it affects 1. Glomerulus Nephrotic syndrome Nephritic syndrome 2. Tubules and interstitium ATN Tubulointerstitial nephritis: acute phyelonephritis, chronic pyelonephritis & refulx nephropathy, interstitial nephritis 3. BVs: Thombotic microangiopathies (HUS, TTP)
299
Features of nephrotic sydnrome
Proteinuria (\>3g/24h) Hypoalbuminaemia Oedema (+hyperlipidaemia) "*Swelling"* facial in children, peripheral in adults *"frothy urine"*
300
What are three primary causes of nephrotic syndrome
Minimal change disease Membranous glomerular disease Gocal segmental glomerluosclerosis
301
Features of minimal change disease Light microscopy Electron microscopy Immunofluorescnece Px Response to steroids
Primary cause of nephrotic syndrome Most common in children (75%) with second peak in elderly No changes Loss of podocyte foot processes No immune deposits \<5% ESRF 90% respond
302
Features of membranous glomerular disease Light microscopy Electron microscopy Immunofluorescnece Px Response to steroids
Primary cause of nephrotic syndrome Common in adults (30%) Diffuse glomerular BM thickening Loss of podocyte foot processes. Subepithelial deposits **spikey** Ig and complements in granular deposits along entire GBM 40% ESRF at 2-20y Can be 1o or 2o to SLE, infections, drugs and malignancy Poor response to steroids
303
Features of Focal Segmental Glomerulosclerosis Light microscopy Electron microscopy Immunofluorescnece Px Response to steroids
Common in adults (30%), Afrocarribean Focal and segmental glomerular consolidation and scarring. Hyalinosis Loss of podocyte foot processes Ig and complement in scarred areas 50% ESRF in 10y 1o but can 2o to obesity and HIV nephropathy 50% respond
304
What are 2 secondary causes of nephrotic syndrome
DM Amyloidosis
305
Histology of DM nephrotic syndrome Hints in question
Diffuse GBM thickening Messangial matric nodules aka **Kimmelstiel Wilson nodules** Asian
306
Histology of amyloidosis nephrotic syndreom Hints
**Apple green birefringence** with congo Red May have chronic inflammation: RA, chronic infection. Causes AA protien depositions May have Ig light chain deposition most commonly from MM Clinical clues of amyloidosis: Macroglossia, HF, hepatomegaly
307
Features of nephritic syndrome
Manifestation of glomerular inflammation i.e. glomerulonephritis Haematuria Dysmorphic RBCs and RBC casts in urine May also have: Oliguria Raised urea and creatinine HTN Proteinuria (not in nephrotic range)
308
Causes of nephritic syndrome
Acute postinfectious IgA Nephropathy (Berger Disease) Rapidly progressive (crescenteric) GN Hereditary nephritis (Alport's) Thin BM Disease (Bening familal haematuria)
309
What are the features of Acute Postinfectious GN
Occurs 1-3w post **streptococcal throat infection or impetigo** (usually Group A S= Strep pyogenes) Glomerular damage due to immune complex deposition Haematuria (red cell casts), proteinuria, oedema, HTN Bloods: raised ASOT titre, decreased C3
310
Light microscopy: Increased cellularity of glomeruli FM: granular deposits of Ig Ga and C3 in GMB EM: subendothelial humps
Findings in Acute Postinfectous GN
311
Features of Berger disease
IgA Nephropathy Most common GN worldwide IgA complex deposition in glomeruli Presents 1-2d after URTI with Frank haematuria Main symptoms are persistent or recurrent frank haematuria or asymptomatic macroscopic haematuria Can rapidly prgoress to ESRF Biopsy: granular deposition of IgA and complement in mesangium
312
GN 1-2d after URTI with haematuria
IgA Nephropathy (Berger)
313
Nephropathy 1-3w post streptotoccal infection
Acute Postinfectious GN
314
Feature of Rapidly progresswive (Crescenteric GN)
Most aggressive GN causing ESRF within weeks Presents as nephritic syndrome but oliguria and renal failure are more pronounced
315
How is cresenteric GN calssified?
Based on immunological findings: Type 1: Anti-GBM Ab Type 2: Immune complex Type-3: pauci-immune/ANCA associated Regardless of cause all are characterised by crescenets in glomeruli on light micrscopy
316
Features of T1 crescenteric GN Cause Light micrscopy Fluorescnece microscopy Additional organ involvement
Anti-GBM Ab Goodpasture's. HLA-DRB1 association Crescents Linear deposition of IgG in GBM Lungs: pulmonary haemorrhage
317
Features of T2 crescenteric GN Cause Light micrscopy Fluorescnece microscopy Additional organ involvement
Immune complex mediated SLE, IgA nephropathy, postinfectious GN Crescents Granular IgG immune complex deposition on GBM/mesangium Often limited, except in SLE
318
Features of T3 crescenteric GN Cause Light micrscopy Fluorescnece microscopy Additional organ involvement
Pauci-immune i.e. lacking anti-GBM or immune complex c-ANCA: Wenger's granulomatosis p-ANCA: microscopic polyangitis Cresecents Lack/scanty immune complex deposition Vasculitis: skin rashes or pulmonary haemorrhage
319
Features of Alport's
Hereditary Nephritis Caused by mutation in Type IV collagen alpha 5 chain X-linked Nephritic syndrome + sensorineural deafness and eye disorders Presents at 5-20y with nephritic syndrome progressing to ESRF
320
Features of Benign Familial Haematuria
Thin BM disease nephirits Rarely causes nephritic syndrome, normally exclusively asymptomatic haematuria Diffuse thinning of the GBM caused by autosomal dominant mutation in type 4 collagen alpha 4 chain. Usually asymptomatic and diagnosed incidentally Usually normal renal function
321
Differentials for asymptoamtic haematuria
Benign Familal Haematuria Berger Alport NB: IgA and Thin BM are more common causes of asymptomatic haematuria than of nephritic syndrome. IgA more likely to cause frank haematuria and change in renal funciton and slighlty more common in Asian pop
322
Features of ATN Histologically
Damage to tubular epithelial cells-\> blockage of tubules by casts-\> reduced flow and haemodynamic changes-\> acute renal failure Necrosis of short segments of tubules
323
What is the most common cause of acute renal failure?
ATN
324
What are the common causes of ATN?
Ischaemia: burns, septicaemia Nephrotoxins: drugs (gentamicin, NSAIDs, radiographic contrast agents, mygolobin, heavy metals)
325
What is tubulointerstitial nephritis? Causes?
A group of renal inflammatory disorders involving the tubules and interstitium Acute pyelonephritis Chronic pyelonephritis and reflux nephropathy Acute interstitial nephritis Chronic interstitial nephritis/analgesic nephropathy
326
What is acute pyelonephritis?
Bacterial infection of the kidney, usually as a result of ascending infection, most commonly by E. Coli Presents with fever, chills, sweats, flank pain, renal angle tenderness and leukocytosis +/- frequency, dysuria, haematuria Leukocytic casts seen in the urine
327
What is chronic pyelonephritis and reflux nephropathy?
Chronic inflammation and scarring of the renal parenchyma caused by recurrent and persistent bacterial infection Can be due to: Chronic obstruciton: posterior urethral valves, renal calculi Urine reflux
328
What is acute interstitital nephritis
A hypersensitivity reaction usually to a drug (NSAID, Abx, diuretics) Usually begins days after drug exposure Presents with fever, skin rash, haematuria, proteinuria, eosinophilia
329
What is chornic interstitial nephirits?
Seen in elderly with long term analgesic consumption e.g. NSAIDs, paracetamol Symptoms occur late in disease: HTN, anaemia, proteinuria and haematuria
330
What are the thrombotic microangiopathies?
HUS and TTP
331
What characterises the thrombotic microangiopathies?
Thrombosis generally renal in HUS and widespread in TTP Triad of: Microangiopathic haemolytic anaemia Thrombocytopenia Sometimes renal failure (HUS)
332
Pathogenesis of the thrombotic microangiopathies
Widespread fibrin deposition in vessels forming platelet fbirin thrombi, damages passing platelets and RBCs Leads to platetel and RBC destruction Resulting in thrombocytopenia and MAHA
333
HUS Epidemiology Pathophysiology Sign/symptoms Renal involvement Dx
Usually affects children Associated with dairrhoea caused by E. Coli 0157:H7 Can be non-diarrhoea associated due to abnromal protines in the complement pathway Thrombi confined to kidney Decreased platelet count-\> bleeding, haematemesis, melena MAHA-\> pallor and jaundice Usually involves renal failure Dx: Anaemia, thrombocytopaenia Signs of haemolyiss: raised bilirubin, LDH and reticulocytes Fragmented RBCs on blood smear Coomb's negative as not AIHA
334
TTP Epidemiology Pathophysiology Sign/symptoms Renal involvement Dx
Usually affects adults Thrombi occurs throughout circulation with CNS involvement particularlr Reduced platelets-\> bleeding MAHA-\> pallor and jaundice Usually no renal involvement, neuro symptoms predominate (headache, altered consciousness, seizures, coma) Dx: As for HUS
335
What is ARF?
A rapid loss of renal funciton manifesting as increased serum creatinine and urea Complications include metabolic acidosis hyperkalaemia, fluid overload, HTN, hypocalcaemia, uraemia
336
Causes of acute renal failure
Pre-renal (Most cmmon): renal hypoperfusion e.g. hypovolaemia, sepsis, burns, acute pancreatitis and RAS Renal: ATN commonest renal cause of ARF Acute GN Thrombotic microangiopathy Post-renal: obstruction to urine flow Stones, tumour, prostatic hypertrophy and retroperitoneal fibrosis
337
Def: Chronic renal failure
Progressive irreversible loss of renal function characterised by prolonged symptoms and signs of uraemia
338
What are the signs of uraemia?
Fatigue, itching, anorexia, eventually confusion
339
What are the most common causes of CRF in the UK?
DM (20%) GN (15%) HTN and vascular disease (15%) Reflux nephropathy (10%) PKD (10%)
340
With what is HUS associated
E Coli O157: H1
341
How is CRF classified?
5 stages based on GFR: Stage 1: kidney damage with normal renal function (proteinuria) \>90 Stage 2: mildly impaired 60-89 Stage 3: moderately impaired 30-59 Stage 4: severely impaired 15-29 Stage 5: renal failure (RRT required) \<15 or if being treated with RRT
342
Features of Adult PKD
Autosomal dominant inheritance, 85% due to mutations in PKD1 remained in PKD2, both encode polycystin Pathological features: large multicystic kidneys with destroyed renal parenchyma, liver cysts (PKD1) and Berry aneurysms. Clinical features: haematuria, flank pain, UTI. Clinical features are often due to cysts cxs e.g. rupture, infection, haemorrhage
343
Features of lupus nephritis
Depends on site and intensity of immune complex deposition Presentation may be with Acture renal failure Urinary abnromalities Nephrootic syndrome or progressive CRF
344
What is Class 1 Lupus Nephritis
Minimal mesangial lupus nephirits Immune complexes with no structural alteration
345
What is Class 2 LN
Mesangial proliferative LN: immune complexes and mild/moderate increase in mesangial matrix and cellularity
346
What is Class 3 LN?
Focal lupus nephritis: active swelling and proliferation in less than half the glomeruli
347
What is Class 4 LN?
Diffuse LN involving more than half of the glomeruli
348
What is Class V LN?
Membarnous LN with subiepithalil complex deposition?
349
What is Class VI LN?
Advanced sclerosing: complete sclerosis of \>90% of the glomeruli
350
How do NSAIDs predispose to acute tubular injury?
Through inhibition of prostaglandins
351
Struture of the normal oesophagus
Proximal: squamous epithelium Distal: columnar (about 1.5-2cm below the diaphragm) LOS: 2-cm segment proximal to the OGJ OGJ: point where tubular oesophagus joins saccular stomach Squamo-columnar junction/ Z line: irregular serrated margin usually at +/- 40cm from incisors but may lie anywhere within distal 2cm
352
Where is teh squamocolumnar junction?
+/- 40cm from the incisors in distal 2cm of oesoophagus
353
What is the most common cause of oesophagitis? What is this? What is the pathology?
GORD Reflux of acidic gastric contents into the oesophagus Ulceration, necrotic slough, inflammatory exudates, granulation tissue, fibrosis
354
What are the Cxs of GORD?
Haemorrhage, perforation, stricture, Barret's oesophagus
355
How is GORD classified? What does this consitute?
Los Angeles Classificaiton Grade A: mucosal breaks confined to the mucosal fold, each no longer than 5mm Grade B: at least one mucosal break longer than 5mm coninfed to the mucosal fold but not continuous between two folds Grade C: mucosal breaks that are continuous between the tops of the mucosal folds but not circumferential Grade D: extensive mucosal breaks engaging at least 75% of the oesophageal circumference
356
What is Barret's oesophagus?
Re-epithelialisation by metaplastic olumner epithelium with goblet cells Barret's: columnar lined oesophagus Goblet cell/intestinal metaplasia Precancerous form: metaplastic galndular epithelium-\> dysplasia-\> Adenocarcinoma Surveillance: repeat endoscopy and biopsy to detect early neoplastic changes
357
Adenocarcinoma of the oesophagus
30-40% of primary oesophageal carcinoma Associated with Barrett's Other Cas: squamous or combined form
358
Features of squamous cell carcinoma of the oesophagus
Dysphagia, anorexia, weight loss 90% in the mid and lower oesophagus Invasion into the muscularis propria 30-40% have invasion of the mediastinum, 50% have LN mets Cytology + biopsy Can spread directly, by LNs or liver mets
359
Features of oesophageal varices
Extremely dilated submucosal veins in the lower third othe oesophagus. A consequence of portal HTN commonly due to cirrhosis
360
Featuires of the normal stomach
Made of the cardia, body and antrum Lined by gastric mucosa: columnar epithelium (mucin secreting) and glands, then lamina propria and muscularis mucosa
361
Features of acute gastritis
Acute insult Neutrophil infiltration Caused by chemicals e.g. Aspirin/NSAIDs, ETOH or corrosives or infection (H. Pylori)
362
Features of chronic gastritis
Persistent insult H. pylori associated Lymphocytic infiltration May also by neutrophils and MALT induction. Also associated with chemical, autoimmune and others
363
Features of chemical gastritis
Reactive or reflux, caused by NSAIDs. Pattern: foveolar hyperplasia (mucus producing cells), smp and sparse chronic inflammation +/- neutrophils
364
What differentiates between actue/chronic gastritis
Acute: neutrophil infiltration Chronic: lymphocyte
365
Features of helicobacter associated gastritis
H. pylori or Heilmanni Chronic gastritis +/- activity, severity varias Outcome variable: Persistence Intestinal metaplasia Dysplasia Ca and lymphoma Eradication willl reduce the risk of Ca but will not eliminate cancer due to [redetermined pathways
366
Causes of pernicious anaemia
Parietal cell Abs (90%) IF Abs (60%) Chronic gastritis and body atrophy Outcome variable: Vit B12, atrophy and Ca
367
What are some other causes of gastritis
Infection: CMV, Herpes, Strongyloides (immunosuppresion) IBD: Crohn's
368
Why should we treat gastritis?
Chronic gastritis/ulcer Intestinal metaplasia Dysplasia Cancer
369
Why should alll ulcers be biopsied? Cxs?
To exclude malignancy Bleeding (anaemia), perforation, peritonitis
370
What is intestinal metaplasia a response to? What is a consideration?
IM in gastric mucosa is a response to LT damage e.g. H. Pylori and Bile There is a Ca risk
371
What is gastric dysplasia?
An abnormal pattern of growth in which some of the histological features of malignancy are present but a non or pre-invasive stage
372
Features of Gastric cancer
High incidence in Japan, Chile, Italy, CHina, Portual and Russia M\>F 90% are carcinomas Environmental factors: smoking, diet
373
Mx of duodenitis?
Do a gastric Bx to assess H. pylori status, the principal cause is acid in the presence of gastric metaplasia
374
Normal architecture of the duodenum?
Villus:crypt 2:1 No increase in lamina propria cellularity No evdience of epithelial damage, no neutrophils Brunner's glands in 1st part
375
Pathogens in the duodenum?
Immunosuppressed CMV: microsporidois, crytposporidiosis in immunosuppression Giarda Lambila Infection Whipples disease: Tropheryma Whippeli
376
Endoscopic findings for partial villous atrophy
Show scalloping with a smooth shiny mucosa (or normal) Bx: early changes are hard to see on biopsy. There is normal variation in villous height May be crypt hyperplasia or intraepithelial lymphocytes
377
Cause of gastric lymphoma Small intestine?
H. pylori Coeliacs: 10% will get primary lymphoma (less often carcinoma of the gut) if not adeuqately treated.
378
Mx of GORD
lifestyle changes (smoking, weight loss), PPI/H2R antag
379
Prevalence of Barrett's? Pathogenesis
Seen in 10% of those with symptomatic GORD Upwards migration of the SCJ
380
Where is oesophageaul adenocarcinoma seen?
Lower 1/3rd of oesophagus due to association with Barrett's
381
Risk factors for SCC oesophageal carcinoma?
ETOH, smoking Achalasia of cardia Plummer-Vinson Nutritional deficiencies Nitrosamines HPV
382
Presentation of SCC oesophagus
Progressive dysphagia, odynopgagia, anorexia, severe weight loss
383
Mx of varices
Emergency endoscopy-\> sclerotherapy/banding
384
Def: gastric ulcer
Breach through muscularis mucosa into submucosa Epigastric pain +/- weight loss
385
What differentiates between gastric ulcer vs dudodenal ulcer?
Gastric ulcer is WORSE with food Duodenal ulcer is RELIEVED by food
386
Ix of gastric ulcer
Biopsy for H. pylori status: punched outm lesion with rolled margins
387
Rx for H. pylori
Triple therapy PPI Clarithromycin Amoxicillin or metronidazole
388
Features of coeliac
T-cell mediated autoimmune disease: DQ2, DQ8 HLA status Villous atrophy and malabsorption Presents in young children and Irish women (EMQs)
389
Symptoms of coeliac
Malabsorption: steatorrhoea, abdo pain, N+V, weight loss, fatigue, IDA, failure to thrive, rash (dermatitis herpetiformis)
390
Serological tests for Coeliac
Anti-endomysial Ab (best sensitivity and specificity) Anti-TTG Anti-gliadin (poor marker of disease control)
391
Gold standard Ix in coeliac?
Upper GI endoscopy and duodenal biopsy (villous atrophy, crypt hyperplasia, lymphocyte infiltrate 10% progress to duodenal T-cell lymphoma if not adequately treated
392
Stomach (body) lined by gastric mucosa columnar epithelium (foveolar, mucin secreting) specialised glands in the lamina propria muscularis mucosa
393
Stomach (antrum) lined by gastric mucosa columnar epithelium (fovelolar, mucin secreting) Non-specialised glands in the lamina propria (gastric pits) mucularis mucosa
394
Duodenum Glandular epithelium with goblet cells (intestinal type epithelium) Villous architecture villous:crypt ratio of \>2:1
395
Differences between metaplasia, dysplasia, cancer
Metaplastic glandular epithelium (intestinal type) Dysplasia changes showing some of the cytological and histological features of malignancy but no invasion through the basement membrane Adenocarcinoma invasion through the basement membrane
396
What are the two morphological classifications of gastric cancer?
Intestinal: well differentaited Diffuse: poorly differentiated Linitis plastica, includes signet ring cell caricnoma
397
VIllous atrophy Coeliac disease
398
Duodenal MALToma
399
Layers of the skin
Epidermis: tratum corneum Stratum lucidum Stratum granulosum Stratum spinosum Stratum Basale DEF Dermis: Paipllary Dermis Reticular dermis Subutis
400
What are these structures?
401
What is the organisation of the epidermis
Corneum Lucidum Granulosum Basale DEJ
402
What are the layers of the skin from superficial to deep?
Horny layer Granular layer Squamous cell layer Basal layer
403
Epidermis cell activitiy from dep to superfifical
Basalae: mitosis, cells bound to BM by hemidesomosomes Spinosum (prickle layer): cells linked by desmosomes S granulosum: nuceli disintegrate Corneum: non-nucleated, contains keratin
404
405
What does hyperkeratosis mean?
Increase in S. corneum/ kertain
406
What is parakeratosis?
Nuclei in S corneum
407
What is acanthosis?
Increases in spinosum
408
What is acantholysis?
Decreased cohesions between keratinocytes
409
What is spongiosis?
Intercellular oedema
410
What is legtiginious epidermis?
Linear pattern of melanocyte proliferation within epidermal basa cell layer
411
Features of dermatitis
All have same histpoapthology Spongiosis of the epidermis, perivascular chronic inflammatory infiltrate in the dermis Dilated dermal capillaries Chronic: acanthosis, crusting/scaling
412
What are the different types of dermetitis
Atopic (eczma) Contact Seborrhoeic
413
Features of atopic dermatitis
Infants: face, scalp Older: flexural areas If chronic may lead to lichenification, persists into adulthood in those with FHx of atopy
414
Features of contact dermatitis
Type IV hypersensitivity e.g. to Nickel, rubber Erythema, swelling, pruritus Commonly affects ear lobes and neck (from jewellery), wrst (leather watch straps), feet (from shoes)
415
Features of seborrhoeic dermatitis
Inflammatory reaction to a yeast- Malassezia Infants: cradle cap Young adults: mild erythema, fine scaling, mildly pruritic affecting face, eyebrow, eyelid, anterior chest, external ear
416
417
Features of Lichen Planus
Lesions are pruritic, purple, polygonal, papules and plaques Mother of pearl sheen and fine network on surface called Wickam's striae Usually on inner surfaces of wrists, can also affect oral mucous membranes. Aetiology unknown Hyperkeratosis with saw toothing of rete ridges and basal cell degeneration with chornic inflammatory infiltrate
418
Hyperkeratosis with saw toothing of rete ridges and basal cell degeneration with chornic inflammatory infiltrate
Lichen planus
419
Wickam's striae seen in?
Lichen planus
420
Features of Psoriasis
2% Commonest form is plaque is chronic plaque psoriasis with salmon pink plaques and a silver scale Rubbing them causing pin-point bleeding Koebner phenomenon: lesions form at sites of trauma Cells have increased proliferation rate Parakeratosis, loss of granular layer, clubbing of rete ridges giving test tubes in a rack appearance. Munro's microabscesses
421
Parakeratosis, loss of granular layer, clubbing of rete ridges giving test tubes in a rack appearance. Munro's microabscesses
Plaque psoriasis
422
What is Auspitz's sign
Rubbing of psoriatic plaques causing pin-point bleeding
423
What is Koebner phenomenon
Psoriatic plaques forming at trauma sites
424
What are the types of psoriasis
Chronic plaque psoriasis Flexural psoriasis: seen in later lift, usually groin, natal cleft and sub mammary areas Guttate psoriasis: rain drop plaque distribution seen 2 week post Strep-throat Erthrodermic/pustular psoriasis: severe widespread disease often systemic symptoms
425
With what is psoriasis associated?
Pitting Onycholysis Subungual hyperkeratosis Arthritis
426
427
428
Features of pityrisasis Rosea
Salmon pink scaly eruption on the trunk extending outwards Herald patch May be assocaited with a virus Pathology: non specific dermatitis
429
Features of erythema multiforme
Causes annular target lesions on hands and feet. Pleomorphic lesions that can be a combination of macules, papules, urticarial weals, vesicles, bullae and petechiae Subepidermal bulllae on histology
430
Causes of erythema multiforme
Infections e.g. HsV, mycoplasma or drug reactions e.g. penicllin, salicylates, anti-malarials
431
Spectrum of erythema multiforme
SJS, TEN
432
Bullous disesease Def: Site
Vesicles \<0,5cm Bullae \>0.5cm Can be sub-epidermal, intra-epidermal and subcorneal
433
What are the three types of bullous disease
Dermatitis herpetiformis Pemphigoid Pemphigus
434
Features of dermatitis herpetiformis
Itchy vesicles on extensor surfaces of elbows, buttocks. Associated with coeliac IgA Abs bind to the BM leading to subepidermal bulla Microabscesses which coalesce to form subepidermal bullae. Neutrophil and IgA deposts at the tips of the dermal papillae
435
Microabscesses which coalesce to form subepidermal bullae. Neutrophil and IgA deposts at the tips of the dermal papillae
Dermatitis herpetiformis
436
Dermatitis herpetiformis
437
Features of pemphiogid
Large tense bullae on erythematous ase. Often on forearms, groin and axillae, seen in the elderly Bullae do not rupture as early as pemphigus IgG Abs bind to demidesomosomes of BM-\> subepidermal bullae PemphigoiD: Bullae are deep Subepidermal bullae with eosinophils. Linear deposition of IgG along BM
438
Subepidermal bullae with eosinophils. Linear deposition of IgG along BM
Pemphiogid Deep: subepidermal bulla
439
Features of Pemphigus
Bullae are easily rupturd and found on skin AND mucous membranes IgG Abs bind to desmosomal protines-\> intraepidermal bulla PemphiguS- superficial Intraepidermal bulla. Netlike pattern of IgG deposits Acantholysis
440
Intraepidermal bulla. Netlike pattern of IgG deposits Acantholysis
Pemphigus
441
442
Features of lupus rash
Chronic, discoid LE, buttterfly rash, alopecia
443
444
What are the types of cutaenous tumours
Benign: Seborrhoeic keratosis Premalignant: Actinic Keratosis Keratoachthoma Bowen's Disease Malignant: SCC BSS Melanocytic: Melanocytic naevia Lentigines MM
445
Features of seborrhoeic keratitis
Rough, waxy plaques, stuck on appear in middle age/elderly
446
Features of Actinic keratosis
(Solar keratosis) Rough, sandpiper like lesions on sun-exposed areas SPAIN: Solar elastosis Parakeratosis Atypia/dysplasia Inflammation Not full thickness
447
SPAIN: Solar elastosis Parakeratosis Atypia/dysplasia Inflammation Not full thickness
Actinic keratosis
448
Features of keratoacanthoma
Rpaidly growing dome shaped nodule which may develop a necrotic crusted centre. Grows over 2-3w and clears spontaenously May be difficult to differentiate from SCC hsitologically
449
Features of Bowen's disease
Intra-epidermal SCC in situ Flat, red, scaly patches on sun-exposed areas Full thickness atypia/dysplasia with BM intact
450
Full thickness atypia/dysplasia with BM intact
Bowen's disease
451
Features of SCC
When Bowen's has spread to involve dermis. Similar clinical features to Bowen's but may ulcerate Atypia/dysplasia throughout epdiermis, nuclear crowding and spreading through BM into dermis
452
Atypia/dysplasia throughout epdiermis, nuclear crowding and spreading through BM into dermis
SCC
453
Features of BCC
Aka: rodent ulcer Slowly growing tumour, rarley metastatic but locally destructive Pearly surfact often with telangectasia Mass of basal cells pushing down into dermis. Nuclei align in outermost layer (pallisading)
454
Mass of basal cells pushing down into dermis. Nuclei align in outermost layer (pallisading)
BCC
455
Features of melanocytic naevi
Proliferation in the basal layer, nesting into dermis
456
What are lentigines?
Melanocyte proliferation limited to the epidermis
457
Features of MM
Atypical melanocytes which intiially grow horizontally in the epidermis (radial growth phase), then grow vertically into dermis (vertical growth phase). Vertical growth produces Buckshot appearance (=pagetoid cells)
458
What is the most important prognostic factor for MM?
Breslow Thickness
459
ABCD of MM
Asymmetry Border irregularity Colour Diamete \>6mm
460
What are the different classifications of MM
Lentigo MM Superficial spreading MM Nodular MM Acral lentiginous MM
461
Features of lentigo MM
Sun exposed areas of elderly caucasians: flat, flowly growing black lesions. MM can develop in a lentigo MM
462
Features of superficial spreding MM
Irregular borders with variation in colour. Not associated with any special site
463
Features of nodular MM
Can occur at all sites and seen in younger age group
464
Features of acral lentiginous MM
Palms, soles and subungual areas.
465
Erythema multiforme
466
467
468
469
470
SJS/TEN features
Dermatological emergency: sheets of skin detachment Nikolsky sign positive: mucosal involvement prominent Commonly caused by drug e.g. sulfonamide antibiotics
471
SJS vs TEN
\<10% body surface vs \>30%
472
Features of skin appenage tumours
Hair follicles and seat glands can produce specialised tumours of skin
473
What are the different types of cutaneous vascular tumours
Capillary haemangioma- strawberry mark Flat haemangioma- port wine stain Kaposi's sarcoma
474
What is mycosis fungiodes?
Most common form of cutaenous T-cell lymphoma generally affects skin. Is a NHL Has a muschroom like appearance
475
Hisotlogical features of herpes infection of skin
Cluster of inflamed pustules and vesicles
476
477
Viral infection
478
479
Pemphigoid
480
Pemphigus
481
Function of cortical bone
Aka Compact bone Facilitaties mane bone functions: suppors, protection and elverage Forms cortex, dense hard bone to which muscles are attached
482
Function of trabecula bone
Also called cancellous bone Higher SA, less dense, softer, weaker and less stiff, High vascular and contains red bone marrow where haematopoesis. Vertebrae Has role in homeostasis of Ca
483
What are the types of non-neiplastic bone disease
Trauma Infeciton Degeneration Inflammation Metabolic
484
What are the classifications of fractures
Complete or incomplete Closed Comminuted Compound
485
486
What are the stages of # repair
Organisation of a haematoma at # site (pro-callus) Formation of fibrocartilaginous callus Mineralisation of fiberocartilaginous callus Remodelling of bone along weight bearing lines
487
What are the factors influencing #] repair
Type Presence of infection Pre-existing conditions e.g. neoplasm, metabolic, drugs (steroids), Vit deficiency
488
What are the features of osteomyelitis in adults
Sites: direct spread, secondary to diabetic skin ulcer, dental abscess etc. S. aureus (90%) E coli Kleb Salmonella (associated with SCD) Pseudomonas IVDU
489
What are the features of osteomyelitis in children
Haematgoenous spread to long bones, usually metaphytic Neonates: Haemophils influenza, Group B strep
490
Clinical features of osteomyelitis
Fever, malaise, chills, leucocytosis Local: pain, swelling and tenderness Blood test +ve in 60% XR: mixed picture eventually lytic, 10d post onset Periosteal reaction Mottled rarefaction (increased osseus vascularity and lifting up of the periosteum) Involcrum: after 1w irregular sub-periosteal bone formation Lytic destruction (10-14d) Seqeustra: after 3-6w, some areas of cortex may become detached
491
XR features of osteomyelitis
Periosteal reaction Involcrum (after 1w: sub-periosteal bonefrmation) Irregular lytic destruction Sequestra: 3-6w, necrotic areas of cortex may become detached.
492
What is the clinical staging of osteomyelitis
Cierny-Mader Staging Anatomical Type: 1- medullary, 2- superficial, 3- localised, 4- diffuse Physiologic: Host A-normal, B- local or systemic compromise, C- treatment worse than disease
493
Mx of osteomyelitis
Biopsy: culture 6w IV antibiotics
494
Features of TB osteomyelitis
Rare cause mainly affecting immunocompromised More destructive and resistant to contral Spinal disease may result in psoas abscess: inguinal lump and severe skeletal deformity: Pott's Systemic amyloidosis may result in protracted cases as an inflammatory response to chornic TB infection Marrow replaced by inflammatory response and a granuloma forms. There are Langerhan's type giant cells which have a peripheral rim of nuceli
495
Marrow replaced by inflammatory response and a granuloma forms. There are Langerhan's type giant cells which have a peripheral rim of nuceli
TB osteomyelitis
496
With what are Langerhan's type giant cells associated
TB
497
Features of Syphillitic OM
Caused by Traponema pallidum Can be congenital or acquired Congenital: skeletal lesions: osteochondritis, osteoperiostitis, diaphyseal ostomyelitis Acquired: sekeltal lesions are late onset, require lack of Rx: non gummatous periosteitis gummatous inflammation of bone and joints neuropathic joints (tabes dorsalis) neuropathic shaft fractures
498
non gummatous periosteitis gummatous inflammation of bone and joints neuropathic joints (tabes dorsalis) neuropathic shaft fractures
Syphillitic OM
499
Features of Lyme disease
Inflammatory arthropathy as part of a complex multisystem illness resulting from tick bite Caused by Borrelia burgdorferi: ixodes dammini (tick spp.) Can be mistaken for OM Associated with erythema chronicam migrans
500
What are the stages of Lyme disease?
1: early localisation, selling + rash (90%) usually within 7-10d and in groin, axilla, earlobe or on thigh Stage 2: early dissemination, affects multiple organ systems Stage 3: low grade, late and persistent, dominated by arthritis, mimics RA
501
Mx of Lyme disease
Prevention: long trousers, vaccines Antibiotics for proven disease Dx is purely clinical
502
OM
503
What is a Brodie abscess?
Subacute OM
504
Features of OA
Degenerative joint disease: Disease of cartilage, cartialge is lost and osteophytes formed, pain due to friction Eboination of underlying bone Avascular necrosis of the femoral head can occur Cartilage degernation Fissuring Abnormal matrix calcification Osteophytes
505
What is the difference between primary and secondary OM
Primary is age related Secondary is to a prveiously damaged joint or congenitally abnormal joint
506
XR changes in osteoarthritis
Loss of joint space Osteophytes Subchondral sclerosis Subchondral cysts
507
Clinical features of OA
Big weight bearing joints, vertebrae and kness PIPJ: Bouchards' DIPJ: Herbeden May also affect MCP Synovial disease: synovium Bx is non-specific with increased vessels and chronic inflammation
508
XR features of RA
Loss of joint space Erosions Soft Bones Soft tissue swelling
509
Features of RA
Disease of the synovium/severe chronic relapsing synovitis Slow and progressive. F\>M RF +ve in 80%, immunocomplexes are more associated with extra-articular disease
510
What factors are involved in genetic predisposition to RA
HLA-DR4 and DR1 TNF, STAT
511
Clinical features of RA
Hand: radial deviation of wrist and ulnar deviation of fingers Swan neck deformity: extension of POPJ and flexion of DIPJ Boutonniere: flexion PIPJ Z shaped thumb Symmetrical pain and swelling DIPJ swelling Small joints of hands and feet, wrists, elbows, ankles and knees Mild anaemia, raised ESR, RF +ve, rheumatoid nodules?
512
Swan neck deformity
Extension of PIPJ and flexion of DIPJ
513
Boutonniere deformity
Flexion PIPJ, extension of DIPJ
514
Histopathology of RA
Proliferative synovitis with thickening of the synovial membranes, hyperplasia of surface synoviocytes, intense inflammatory cell infiltrate, fibrin deposition on the joint and encrosis Pannus: exuberant inflamed synovium of the articular surface, hyperplastic synovium. Proliferative synovitis Grimley-Sokoloff cells
515
What are Grimley-Sokoloff cells
Hyperplastic synovial cells
516
What are the stages of RA
Unknown antigen reaches the synovial membrane T-cell proliferation associated with increased B cells and angiogenesis Chronic inflammation with inflammatory cytokines Pannus Cartilage and bone destruction
517
What differentiates between RA and OA in hands
RA spares DIPJ
518
RA
519
OA
520
Features of Gout
Male, middle aged Increased dietary purin intake, ETOH, diuretics, inherited metabolic abnormalities Great toe: MTP (podagara), lower extremities Urate crystals: needle shaped Negatively birefringent crystals
521
Clinical features of gout
Hot, swollen, red, painful joint. Tophus is the pathognomic lesion (on pinna and hands)
522
Negatively birefringent crystals=
Urate GOUT
523
Featurs of pseduogout
\>50y Idiopathic, hyperPTH, DM, hypothyroid, Wilsons Knee and shoulder Hot, swollen joint with effucion Calcium pyrophosphate crystals, rhomboid shaped Positively birefringent
524
Positively birefringent crystals=
Pseudogout
525
Mx gout
Acute attack: colchicine LT: allopurinol Conservative: ETOH and purine intake (sardines/liver) reduction
526
Mx pseudogout
NSAIDs or intra-articular steroids
527
Subtypes of pseudogout
Sporadic Metabolic: haemachromatosis, primary HPT, mypo Mg, low PO4 Hydroxylapatite crystal deposition Hereditary: AD, ANKH mutation, transmembrane protein Traumatic: e.g. bad OA of the knee
528
Crystals in pseudogout
Calcium pyrophosphate (mainly knees) or calcium hydroxyapatite (knees and shoulders) Crystals survive processing, whereas crystals of gout dissolve in aqueous solution and need to be fixed in alcohol or sent unfixed. Crystals are not needle shaped but found in polyhonal or rhomboid islands
529
Gout
530
TB OM
531
AVN of femoral head with characteristic wedge shaped infarct
532
Swan neck deformity
533
Boutonniere
534
RA vs OA synovium RA= proliferative
535
What is the most common site for bone malignancy?
Knee
536
What is the clinical presentation of bone tumours?
Pain, swelling, deformity and fracture
537
Diagnosis of bone tumours
XR: evaluate site, margin and size. Number of lesions and soft tisse extension. Any associated disease or fratcure Biopsy: performed by radiologist with a Jamshidi needle to produce a core lesion. Can be CT or USS duiged. Open Bx for sclerot or inaccessible lesions
538
What are the different types of malignant bone tumour?
Osteosarcoma Chondrosarcoma Eqing's sarcoma Giant cell (borderline malignancy)
539
What are the most common malignant bone tumours?
Secondary metastatic
540
What adult malignancies metastasise to bone?
Breast Prostate Lung Kidney Thyroid
541
What childhood malignanices commonly metastasise to bone?
Neuroblastoma, Wilm's, osteosarcoma, Ewings, rhabdomyosarcoma
542
Features of osteosarcoma
Adolescents. M\>F. 75% 10-30y. Site: knee\>tibia\>humerus\>skull\>femur Originates from cells of mesenchymal origin, osteoblastic differentiation differentiation and produce malignant osteoid Commonest primary bone tumour Malignant mesenchymal cells +/- bone and cartilage formation. H&E stain of osteosarcoma will show mixed malignant bone in a backgrown of less cellular chondroid tumour. ALP +VE
543
Malignant mesenchymal cells +/- bone and cartilage formation. H&E stain of osteosarcoma will show mixed malignant bone in a backgrown of less cellular chondroid tumour. ALP +VE`
Osteosarcoma
544
osteosarcoma
545
Osteosarcoma ALP +ve
546
XR appearance of osteosarcoma
Usually metaphyseal Lytic Permeative Elevated periosetum (Codman's triangle)
547
Classification of osteosarcoma
Site within bone: intramedullary, cortical or surface Degree of differentiation: grade Multicentricity: synchronous or metachronous Primary or secondary
548
Mx of osteosarcoma
CTx and limb salvage surgery
549
Features of chondrosarcoma
Malignant cartilage producing sarcoma \>40 M\>F Pelvis, axial skeleton, proximul femur, proximal tibia XR: ltyic lesions with fluffy calcification involving axial skelton Malignant chondrocytes +/- chondroid matrix may dedifferentiate to high grade sarcoma Px: 70% 5y but depends on grade and size
550
XR: ltyic lesions with fluffy calcification involving axial skelton
Chondrosarcoma
551
Malignant chondrocytes +/- chondroid matrix may dedifferentiate to high grade sarcoma
Chondrosarcoma
552
Classiicaiton of chondrosarcoma
Site: intramedullary or juxtacortical Histologically: conventional (myxoid or hyaline), clear cell, dedifferentiated or mesenchymal
553
Features of Ewing's Sarcoma
AKA PNET (primitive peripheral neuroectodermal tumour) Highly malignant small round cell tumour found in bone and soft tissue \<20y/0 Long bones and pelvis: diaphysis or metaphsis XR: Onion skinning of periosteum Sheets of small round cells. CD99+ve, T 11.22 translocation Px: 75% 5y
554
XR: Onion skinning of periosteum
Ewing's Sarcoma
555
Sheets of small round cells. CD99+ve, T 11.22 translocation
Ewing's Sarcoma
556
Ewing's Sarcoma Onion skinning of the periosteum
557
With what is chromosomal translocation of 11:22 associated?
Ewing Sarcoma
558
What are the different types of Round cell tumours?
Ewing Small cell osteosarcoma Rhabdomyosarcoma Neuroblastoma Lymphoma SCC
559
What are the features of Giant Cell Tumours
Borderline malignancy 20-40y F\>M Epiphysis with metaphyseal extension Knee epiphysis, either femur or tibia, wrist, pelvis, sarcum XR: lytic lesions right up to articular surface Osteoclast-type multinucleate giant cells on background of spindle/ovoid cells Usually benign with unpredicatble behaviour Locally aggreswsive, may recur and can metastasise
560
XR: lytic lesions right up to articular surface Osteoclast-type multinucleate giant cells on background of spindle/ovoid cells
Giant Cell Tumour
561
Giant cell tumour Lytic lesion right up to articular surface
562
Features of malignant bone tumours
60 times rarer than lung Ca Primary malignant bone tumours are most cmmon in children and young adults On H&E appear red and destroy local bone
563
What are the types of benign bone tumours
Osteid osteoma Osteoma Enchondroma Osteochondroma Fibrous dysplasia Simple bone cysts Osteoblastoma
564
What are the features of fibrous dysplasia
Normal bone replaced with fibrous bone tissue causing abnromal sweeling and growth of bone. F\>M, after 3rd decade Any bone: ribs and proximal femur commonest. Monosototic\>polyostotic XR: soap bubble osteolysis. Shepher'd crook deformity Irregular trabeculae of woven bone in bland spindle cell stroma. Bone trabeculae shapes= chinese letters
565
XR: soap bubble osteolysis. Shepher'd crook deformity Irregular trabeculae of woven bone in bland spindle cell stroma. Bone trabeculae shapes= chinese letters
Fibrous dysplasia
566
Shepherds Crook Fibrous dysplasia
567
Chinese letters Fibrous Dysplasia
568
Albright syndrome
Polyostotic fibrous dysplasia Cafe au lait spots Precocious puberty (endocrine problems)
569
What are the tumour like coniditions of bone?
Fibrous dysplasia Metaphyseal fibrous cortical defect/non-ossifying fibroma Reparative giant cell granuloma Ossifying fibrome Simple bone cyst
570
What are the features of simple bone cysts?
Found in humer or femur Fluid filled and unilocular, lystic well defined margins
571
How can the benign tumours of bone be classified?
Cartilaginous differentiation: Osteochondroma Endochrondroma Chondroblastoma Bone forming tumours: Osteoid osteoma Osteoblastoma Osteoma
572
What are the features of osteoid osteomas
Adolescent M\>F Arise from osteoblasts, tend to be \<1.5cm, most common in long bones (tibia and femur) Night pain relieved by aspirin Radiolucent nidus with sclerotic rim Normal bone
573
Radiolucent nidus with sclerotic rim Normal bone
Osteoid osteoma
574
What are the features of osteoma
Middle age A new piece of bone usually growing on another piece of bone, tpically inthe head and neck Normal bone Assocaited with Gardner syndrome
575
What is Gardner Syndrome?
GI polyps + multiple osteomas + epidermoid cysts
576
What are the features of osteoblastomas?
Similar to osteoid osteomas. Osteoblastoma is a giant osteoid osteoma XR: speckled mineralisation
577
XR: speckled mineralisation
Osteoblastoma
578
Osteoid osteoma Radiolucent nidus with sclerotic rim
579
Osteoblastoma Speckled mineralisation
580
What are the features of enchondromas?
Benign tumours of cartilage. 55% of pts 10-40y Hand\>humerus\>knee\>foot XR: lytic lesion. Cotton wool calcifcation. Expansile O-ring sign
581
XR: lytic lesion. Cotton wool calcifcation. Expansile O-ring sign
Enchondroma
582
Features of osteochondroma
50%pts 10-20y. M:F Knee, humerus, tibia and femur. Growth plates of long bones A benign cartilage capped bony outgrowth. XR: well defined bony protruberance from bone Cartilage capped Most common neoplsam of the skeletonl. If there is irritation it can be removed by surgery
583
XR: well defined bony protruberance from bone Cartilage capped
Osteochondroma
584
Features of chondroblastomas
Rare, originating from chondroblasts, most common epiphyseal tumour in children
585
Ollier's Syndrome
Multiple endochondromas
586
Maffuci's syndrome
Multiple endochondromas with haemangioma
587
Diaphyseal aclasis/ hereditary multiple exostoses
Multiple exostoses (osteochondromas) + short stature + bone deformity
588
What is the definition of soft tissue tumour
Mesenchymal proliferation which occur in the extraskeletal non-epithelial tissues of the body excluding the meninges and the lymphoreticular system Anywhere but the majority occur in the large muscles of extremities, chest wall and retroperitoneum. Mostly older. M\>F but varies by histological type
589
What are the types of soft tissue sarcoma?
Myxoid liposarcoma Spindle cell sarcoma Pleomorphic tumours Synovial sarcoma: biphasic patter with epithelial and spindle cell areas.
590
What are the congenital lower GI diseases?
Atresia and stenosis Duplication imperforate anus Hirschprung's
591
What are the features of Hirschprungs?
Absence of ganlgion cells in myenteric plexus Distal colon fails to dilate 80% male Constipation, abdominal distension, vomiting and overlow diarrhoea in young babies. Associated with DS Genetics: proto-oncogene Cr10+
592
Dx and Mx of Hirschprung's
Clinical impression, biopsy of affected segment: hypertrophied nerve fibres but no ganglia Mx: resection of affected: constricted segment
593
How can the acquired diseases of the lower GI be classified?
Mechanical Inflammatory Ischaemic Malignant
594
What are the causes of GI obstruction
Constipation Diverticular disease Adhesions Herniation External mass Volvulus Intussuception
595
What is volvulus?
Complete twisting of bowel loop at mesenteric base around a vascular pedicle. Intestinal obstruction +/- infarciton. Infants: commonly small bowel Elderly: sigmoid\>caecal
596
Features of diverticular disease
High incidence in West due to low fibre diet and high intraluminal pressure Herniation occurs at weak points in the wall of the bowel, 90% in left colon. Seen as outpouchings on endoscopy. Cxs: diverticulitis, gross perforation, fistula to bowel, bladder or vagina, or obstruction
597
What are the inflammatory causes of bowel disease?
Acute: Infection Toxins (abx) CTx/RTx Chronic: IBD TB
598
Causes of acute colitis
Infection, durgs and toxins, CTx
599
Features of infective colitis
Secretory diarrhoea (toxin) Exudative diarrhoea (invasion and muscoal damage) Severe damage and perforation Systemic illness
600
What is pseudomembranous colitis
Abx associated colitis. Acute colitis with pseudomembrane formation. Caused by protein exotoxins of C. diff Dx; histology, biopsy, C diff toxin or stool assay Rx: metronidazole or vancomycin
601
Features of iscahemic colitis/ infarction
Can be acute or chronic Most common vascular disorder of the GIT Usually occurs in "watershed" zones e.g. splenic flexure (SMA/IMA) and the rectosigmoid (IMA/IIA) Can cause damage in tissue layers leading to perfroation: muscoal, mural, transmural
602
Causes of iscahemic colitis
Arterial: atheroma, thrombosis and embolism Venous: thrombus, hypercoagulable states Small vessel disease: DM, cholesterol emboli, vasculitis Low flow state: CCF, haemorrhage, shock Obstruction: hernia, intussuception, volvuluis and adhesions
603
Clinical features of IBD
Diarrhoea +/- blood Fever Abdo pain Acute abdomen Weight loss Extra-intestinal manifestations
604
Crohn's epidemiology
Western populations F\>M White\>non-white Smoking aggravates
605
UC epidemiology
More common than Crohn's Whites\>non-whites 20-25y
606
Aetiology of IBD
Genetic predispotiion (familial aggregation, twin studies, HLA0 Infection (MTB, measles) Abnormal host immunoreactivity
607
Pathophysiology of Crohn's
Affects whole GIT Most common in terminal ileum and caecam Skip lesions: patchy distribution Cobblestone appearance: areas of health mucose lie above diseased mucosa Apthous ulcer. Rosethorn ulcers can join together to form serpentine granulomas Transmural inflammation Non-caseasting granulomas Fistula/fissure formation more common
608
Skip lesions
Crohn's
609
Apthous ulcer
First lesion in Crohn's
610
Non-caseating granulomas with transmural inflammation
Crohn's
611
Pathophysiology of UC
Extends proximally from rectum Continuous Small bowel not affected unless v. severe colitis casuses backwash ileitis Extensive superficial broad ulcers confined to the mucosa No granulomas/fissures/fistulae/strictures Islands of regenerating muscoa bulge into lumen to cause pseudopolyps
612
Mucosal continous ulceration with no granulomas
UC
613
Backwash ileitis
Severe pancolitic UC
614
Thick rubber hose like bowel wall with narrow lumen Fat wrapping
Crohn's
615
Bowel wall normal thickness in IBD
UC
616
IBD: Intermittent diarrhoea, pain and fever
Crohn's
617
IBD: Bloody diarrhoea, mucus, crampy abdominal pain relieved by defacation
UC
618
Extra GI manifestation of IBD
Malabsorption and Fe def: stomatitis Eyes: uveitis, conjuncitvitis Skin: erythena nodosum, pyoderma gangrenosum, erythema multiforme, digital clubbing Joints: migratory asymmetrical polyarthropathy of large joints, sacroiliitis, myositis, ankylosing spondylitis Pericholangitis, primary sclerosing cholangitis, steatosis
619
PSC
UC Pancolitis= increased
620
Extra-intestinal manifestation of IBD: Eyes
Uveitis, conjunctivitis
621
Extra-intestinal manifestation of IBD: Skin
Eythema nodosum Pyoderma gangrenosum Erythema multiforme Clubbing
622
Extra-intestinal manifestation of IBD: Joints
Migratory asymmetrical polyarthritis Sacroilitis Anky spond Myositis
623
Extra-intestinal manifestation of IBD: Liver
Pericholangitis PSC Steatosis
624
Cxs of Crohn's
Strictures (recurrent) Fistulae Abscess fromation Perforation
625
Cxs of UC
Severe haemorrhage Toxic megacolon 30% require colectomy within 3y for uncontrollable symptoms Adenocarcinoma (20-30x risk)
626
Ix in Crohn's
Markers of inflammation: ESR, CRP, Ba contrast, endoscopy
627
Ix in UC
Rectal biopsy Flexible sig/colonoscopy AXR Stool culture
628
Mx Crohn's
Mild: prednisolone Severe: IV hydrocortisone, metronidazole Additional therapies: Azathioprine, methotrexate, infliximab
629
Mx UC
Mild: prednisolone and 5 ASA (meslalzine) Moderate: prednisolone, 5ASA and steroid eneme BD Severe: admit, nbm IV fluids and IV hydrocortisone, rectal steroids For remission: 5ASA 1st line, azathioprine 2nd line
630
Features of carcinoid syndrome
Diverse group of tumours of enterochromaffin cell origin Produce 5-HT Commonly found in the bowel but also lung, ovaries, testes, usually slow growing
631
Features of carcinoid syndorme
Bronchoconstriction Flushing Diarrhoea
632
Features of carcinoid crisis
Life threatening vasodilation, hypotension, tachycardia. Bronchoconstriction Hyperglycaemia
633
Ix of carcinoid syndrome? Rx
24h urine 5-HIAA- main 5HT metabolite Octreotide: somatostatin analgoue
634
What are the non-neoplastic polyps of the large bowel?
Hyperplastic Inflammatory: pseudopolyps Hamartomotous (juvenile, Peutz Jeghers)
635
What are the features of adenoma
Benign dysplastic lesions that are the precursor to most adenocarcinomas Mostly asymptomatci so need surveillance if \>3.4cm Classified based on architecture as tubular, tubulovillous or villous adenoma Size is most important risk factor for malignancy, in addition to degree of dysplasia and increased villous component
636
What can villous adenoma lead to and why?
Hypoproteinaemic hypokalaemia because they leak large amounts of protein and K
637
What is the sequence of metaplasia in adenocarcinoma
Normal colon-\> at risk mutation after first hit mutation in APC At risk\_\> adenoma after second hit to APC Progression to APC follows activation of KRAS, LOF in p53
638
Features of hamartomatous polyps
Found sporadically in some genetically acquired syndromes
639
Features of Juvenile polyposis
Focal malformations of mucosa and lamina propria Generally \<5y/o. Mostly in rectum leading to bleeding Usually solitary but \>100= juvenile polyposis which may require colectomy to stop haemorrhage
640
Features of Peutz-Jeghers
Multiple polyps, mucocutaneous hyperpigmentation, freckles around mouth, palms and soles Increased risk of intussucception and malignancy. Require regular surveillance of GIT, pelvis and gonds
641
Features of hyperplastic polyps?
Seen at 50-60y and though to be caused by shedding of epithelium-\> cell build up
642
What are the mesenchymal lesions of the GIT?
Stromal lesions Lipoma Sarcoma
643
Villous adenoma
644
Adenoma
645
Hyperplastic polyps
646
Epidemiology of CRC
2nd commonest cause of cancer death in UK 60-79y/o \<50y= ?familial syndrome 98% are adenocarcinomas, 45% in rectum
647
Aetiology of CRC
Diet low in fibre high in fat Lack of exercise Obesity Familaly syndromes Chronic IBD NSAID protective (COX2 overexpressed in 90%)
648
Clinical features of right sided CRC
Fe def, anaemia. Weight loss
649
Clinical features of left sided CRC
Change in bowel habit, crampy LLQ pain
650
Ix in CRC
Sigmoidoscopy, solonoscopy, proctoscopy Barium enem Bloods: FBC CT/MRI CEA to moniotr disease
651
How is CRC graded and staged?
Grade: level of differentiation Staging: Duke's
652
Outline Duke's staging
A: confined to the wall of the bowel \>95% B: Through wall of bowel 1: extending into muscularis propria 67% 2: transmural invasion without LN involvement 54% C: LN metastases 1: Extending into muscularis propria with LN involvement 43% 2: Transmural with involved LNs 23% D: Distant mets \<10%
653
Mx of CRC
Sx: Rectal cancer/ low sigmoid cancer: \<1-2cm above anal sphincter (lower 1/3rd of rectum): abdomino-perineal resection \>1-2cm above anal sphincter: anterior resection Sigmoid cancer: sigmoid colectomy Descending colon and distal transverse: left hemicolectomy Caecum, ascending colon and proximal transverse: right hemicolectomy Transverse colon: extended right hemicolectomy RTx to reduce local recurrence post-Sx CTx in pallliation: 5 fluorouracil
654
Features of FAP
70% AD mutations in APC gene with 30% AR mutation in MMR genes Presents 10-15y with \>100 adenomatous polyps required for Dx All will progress to adenoarcinoma if left untreated thereefore most have prophylactic colectomy Increased risk of neoplasia elsewheree.g. ampulla of Vater and stomach At birth hypertrophy of retinal pigment epithelium
655
Gardner Syndrome
Same clinical, pathological and aetiological features as FAP Distinctive extra-intestinal manifestations: Multiple osteomas of skull and mandible Epidermoid cysts Desmoid tumours Dental caries Post-surgical mesenteric fibromatoses
656
Turcot's syndrome
Malignant tumours of the CNS associated with FAP
657
HNPCC
AD mutation in MMR gene 3-5% of colorectal cancer at early age of onset Cacrinomas usually in right colon, few polyps but fast preogression Also assocaited with endometrial, ovarian, small bowel, transitional cell and stomach carcinoma
658
Lynch syndrome
HNPCC
659
Lynch syndrome I
Familial colon cancer
660
Lynch syndrome 2
Associated with other cancers of the GIT or reproductive system
661
What are the Amsterdam criteria
Used to determine testing of individuals at risk of HNPCC Amsterdam Criteria: Three or more family members with a confirmed diagnosis of colorectal cancer, one of whom is a first degree (parent, child, sibling) relative of the other two Two successive affected generations One or more colon cancers diagnosed under age 50 years Familial adenomatous polyposis (FAP) has been excluded Amsterdam Criteria II: Three or more family members with HNPCC-related cancers, one of whom is a first degree relative of the other two Two successive affected generations One or more of the HNPCC-related cancers diagnosed under age 50 years Familial adenomatous polyposis (FAP) has been excluded
662
What are the samples in cytopathology?
Gynae: NHSCSP Non gynae: exfoliative samples
663
Features of the NHS CP
Women invited from 25-65y Samples taken are liquid based cytology Viewed microscopically Also test for HPV and STDs Reduces incidence of invasive squamous cell carcinoma
664
What are the high risk types of HPV What account for 70% of cervical cancers
16 and 18
665
What are the significant mutations in lung adenocarcinoma?
EGFR, ALK-1
666
What are the significant mutations in melanoma
BRAF
667
What are the significant mutations in breast Ca
BRAC1/2 CERB-B2
668
What are the significant mutations in CRC
APC KRAS
669
Def: DM
Metabolic disorder characterised by chronic hyperglycaemia due to lack of insulin.
670
Aetiology T1DM T2DM
Autoimmune destruction of insulin producing beta cells by CD4 and CD8 T lymphocytes Strongly linked to insulin resistance and obesity
671
Presentation of DM
Polyuria (osmotic diuresis) Polydipsia (raised plasma osmolality) Hyperglycaemia predisposes to recurrent UTI and skin infections T1 can present as DKA
672
Dx of DM
Fasting glucose \>7mmol/l Random \>11.1mmol/l
673
Cxs of DM
IHD CKD Blindness PVD Foot ulceration
674
Ectopic pancreas
Pancreatic tissue located in the duodenum , stomach or Meckel's 25-50% asymptomatic in adulthood
675
Pancreas divisum
Failure of fusion of the ventral and dorsal panceatic duct system Higher risk of pancreatitis
676
Annular pancreas
Partial or complete obstructionof the duodenum by the pancres. Duodenum surrounded by a ring of pancreatic tissue which is continuous with the head of the pancreas
677
Exocrine functions of the pancreas
Produces 2l/d of enzymic HCO3- rich fluid, stimulated by secretin and CCL
678
Secretin features
Produced by s-cells of the duodenum Controls gastric acid secretion and buffering with HCO3
679
CCK features
Responsible for stimulating digestion of fat and protein Made by I-cells in the duodenum Causes release of digestive enzymes
680
Alpha cells of pancreas
Glucagon-\> increases blood glucose
681
Delta cells of pancreas
Somatostatin to regulate alpha and beta cells
682
D1 in pancreas
Vasoactive peptide stimulating the secretion of H2O into the pancreatic system
683
PP in pancreas
Pancreatic polypeptide Self-regulates secretion activities
684
Macrovascular cxs of DM
Cardiac- MI Renal- GN, pyelonephritis Cerebral- CVA
685
Microvascular Cxs of DM
Diabetic retinopathy Claudidcation
686
Causes of acute pancreatitis I GET SMASHED
Idiopathic Gallstones Ethanol Trauma Steroids Mumps Autoimmune Scorpion Venom Hyperlipidaemia ERCP Drugs e.g. thiazides
687
Presentation of acute pancreatitis
Severe epigastric pain radiating to back, relived by sitting forward. Vomiting prominent Hypoetnsive shock in severe cases A signficantly raised plasma amylase in clinical setting is virutally diagnostic Can result in the formation of a pseudocyst, associated with alcoholic pancreatitis
688
What cells perform the exocrine function in the pancreas?
Acinar cells
689
Amylase vs lipase in pancreatitis
Amylase is only transiently raised Lipase is more sensitive
690
Cxs of acute pancreatitis
Severe cases may lead to shock and multiorgan failure Infection of necrosed pancreas worsens prognosis
691
Histology in acute pancreatitis
Coagulative necrosis
692
Causes of chronic pancreatitis
Alcoholism CF Hereditary Pancreatic duct obstruction e.g. stone/tumour Autoimmune IgG4 sclerosing
693
IgG4 sclerosing disease
Autoimmune chronic pancreatitis
694
NB re chronic pancreatitis
Can very closely mimic pancreatic cancer clinically, radiologically and pathologically
695
What is this tissue?
Pancreas
696
Presentation of chronic pancreatitis
Chornic upper abdominal pain Weight loss Steatorrhoea and diarrhoea occur when most of the gland has been destroyed.
697
Pathology of Chronic pancreatitis
Very similar to pancreatic carcinoma: Grossly replaced with fibrous tissue, loss of exocrine tissue, duct dilatation with thick secretions, calcification
698
What are the features of Acinar cell carcinoma
Rare (1%) malingnant epithelial neiplasm of the pancreas demonstrating evidence of enzyme production by the neoplastic cells Older adults.
699
Presentation of acinar cell carcinoma
Non specific symptoms: abdominal pain, weight loss, N+D 10% get multifocal fat necrosis and polyarthralgia due to lipase secretion
700
Histopathology of pancreatic acinar carcinoma
Neoplastic epithelial cells gowing in sheets Some have abundnant eosinophilic granular cytoplasm Positive immunoreactivity for lipase, trypsin, chymotrypsin.
701
Px of acinar cell carcinoma?
5YSR= \<10%
702
What type of pancreatic malignancy is this?
Acinar cell carcinoma neoplastic epithelial cells growing in sheets, trabeculae and acini, some have abundant eosinophilic granular cytoplasm, positive immunoreactivity for lipase, trypsin and chymotrypsin.
703
What is the most common pancreatic malignancy? Epidemiology?
Ductal cell adenocarcinoma of the pancreas M\>F \>60y/o
704
What are the risk factors for pancreatic carcinoma?
Smoking is the main Diet Genetic e.g. FAP, HNPCC
705
What are the clinical features of ductal cell carcinoma
Cachexia and anorexia Upper abdominal and back pain that is persistent and severe Painless jaundice, pruritis, steatorrhorea DM Trousseau's syndrome Ascites Abdominal mass Vrichow's node Corvosier's sign
706
Troussau's syndrome
The Trousseau sign of malignancy or Trousseau's Syndrome is a medical sign involving episodes of vessel inflammation due to blood clot (thrombophlebitis) which are recurrent or appearing in different locations over time (thrombophlebitis migrans or migratory thrombophlebitis). The location of the clot is tender and the clot can be felt as a nodule under the skin.[1] Trousseau's syndrome is a rare variant of venous thromboembolism (VTE) that is characterized by recurrent, migratory thrombosis in superficial veins and in uncommon sites, such as the chest wall and arms. This syndrome is particularly associated with pancreatic and lung cancer. [2] Trousseau's Syndrome can be an early sign of gastric or pancreatic cancer,[3] typically appearing months to years before the tumor would be otherwise detected.[4] Heparin therapy is recommended to prevent future clots.[5] The Trousseau sign of malignancy should not be confused with the Trousseau sign of latent tetany caused by hypocalcemia.
707
Ix in ?pancreatic ductal cell carcinoma?
Bloods: decreased Hb, raised bilirbuin, hypercalcaemia CT/MRI/ERCP CA19.9 \>70IU
708
Tumour marker in pancreatic acrinoma?
CA19.9
709
Mx of pancreatic ductal carcinom
CTx palliative (5FU) Whipple's procedure Px v poor \<5% 5YSR
710
What type of pancreatic malignancy is this?
Ductal cell carcinoma
711
What are the features of pancreatic endocrine tumours?
Group of epithelial tumours of the pancreas showing endocrine differentiation May be functioning or non-functioning 2% of all pancreatic tumours 30-60y/o Associated with MEN1
712
Symptoms of functional neuroendocrine tumours of the pancreas
Present with symptoms related to hormone excess e.g. Insulinoma: hypoglycaemic attacks Gastrinoma: Zollinger-Ellison syndrome: recurrent ulceration VIPoma: diarrhoea Glucagonoma: necrolytic migrating erythemia
713
Ix in neuroendocrine tumour of pancreas Mx
CT/MRI Sx
714
MEN1 PPP
Parathyroid hyperplasia/adenoma Pancreatic endocrine tumour Pituitary adenoma
715
MEN 2A
Parathyroi Thyroid Phaeo
716
MEN2B
Medullary thyroid Phaeo Neuroma Marfanoid phenotype
717
What are the different types of pancreatic cystic tumours?
Intraductal papillary mucinous neoplasm Mucinous cystic neoplasm Serous cystic neoplasms Solid pseudopapillary neoplasm
718
What type of pancreatic malignancy is this?
Pancreatic neuroendocrine tumour
719
What is atherosclerosis?
A stereotypic arterial response to injury caused by an excessive and harmful inflammatory and fibroproliferative response Chronic inflammation in the intima of large arteries
720
What are the sites of predilection for atherosclerosis?
Branch points of arteries, bifurcations and curvatures
721
What is shear stress?
Sideways force on the endothelium representing the tendency to be dragged along with blood flow. Turbulence is atherogenic
722
Pathophysiology of atherosclerosis?
Damage to smooth endothelium LDL enters hte endothelium, becomes trapped and undergoes oxidative modification Monocyte recruitment via chemokines (MCP-1) Adhere to endothelium via ICAM-1 and VCAM-1 Transendothelial migration Oxidised lipoproteins are taken up by monocytes, recognised by macrophage scavenger receptors and PRRs (can stain for macrophages with CD68) These form foam cells which excrete inflammatory mediators e.g. IL-1 Collagenolysis via MMP occurs Foam cells die by apoptosis releasing cholesterol into a lipid core. Fatty streaks are hte earliest lesions seen in atherosclerosis Platelets stick to the damaged tissue Endothelium proliferates: fibrous cap forms on top of the endothelium and more cholesterol is deposited in the core The plaque enlarges blocking arteries
723
What is the earliest lesion seen in atherosclerosis
Fatty streaks
724
What is the function of LDL
Cholesterol rich, carries cholesterol from liver to tissues Oxidised LDL causes inflammation, endothelial damage and VSMC death
725
What are the functions of VSMCs?
Two phenotypes: contractile and synthetic: Synthesise collagen and strengthen the fibrous cap
726
Function of endothelial cells
Biologically active lining of BVs Portal of LDL emigration Modualted by shear stress Express adhesion molecules
727
Features of familial hypercholesterolaemia?
AR Extreme cholesterol levels \>20mmol/L which is resistant to lipid lowering agents Xanthomas in skin and tendons Rapid atherosclerosis
728
Timescale for atherosclerosis
Intermediate lesions: 20y Advanced lesions: 40y Cx \>40y
729
What are the Cxs of atherosclerosis?
Stenosis: carotids Rupture: leading to sudden thrombosis and or emoblism Erosion: can lead to thrombosis Intraplaque haemorrhage Aneurysm
730
What are the structures of unstable plaques
Large lipid core, fibrous cap with a prominent inflammatory infiltrate Inflammation has a lytic effect and there are fewer synthetic VSMCs There is a structural breakdown of the superficial cap which tears causing intraplaque haemorrhage and superficial endothelial erosion
731
Symptoms of atherosclerosis
Insufficient blood supply leads to cellular dysfunction which is often experienced as pain on exertion: angina pectoris, intermitten claudication Critical iscahemia results in cell death: MI, cerebral infarction, gangrene Cell death occurs, then micrsscopic changes, then gross cahnges
732
What is usually the cause of death in MI?
VF
733
MI Histology: \<6h
Normal
734
MI Histology: 6-24h
Loss of nuclei, homogenous cytoplasm and necrotic cell death Neutrophil infiltration occurs first
735
MI Histology: 1-4d
Infiltration of polymorphs Then macrophages- clear debris
736
MI Histology: 5-10d
Removal of debris
737
MI Histology: 1-2w
Granulation tissue, new blood vessels, myofibroblasts and collagen synthesis Weeks-months-\> strengthening and decellularising scar
738
What are the complications of acute MI
Arrythmias Congestive heart failure Fibrionous pericarditis Cardiac rupture Papillary muscle dysfunction or rupture Aneurysm PE Dressler syndrome Cardiogenic schock M
739
What are the mechanical complications of MI?
Rupture of the LV, rupture of the interventricular septum and MR
740
What are the pericardial complications of MI
3 types: Early infarct associated pericarditis Pericardial effusion (+/- tamponade) Dressler's
741
Why does MI lead to increased risk of arrythmia?
The damaged myocardium acts as a substrate for re-entrant circuits due to changes in tissue refractoriness: Generalised autonomic dysfunction resulting in enhanced automaticity of the myocardium and conduction system There is an electrolyte imbalance: hypokalaemia and hypomagnesia Transmural infarction can interrupt afferent and efferent limbs of the SNS
742
How does the risk of VF change after MI
Greatest immediately after, declines afterwards
743
What proportion of patients with AMI are susceptible to arrythmia?
90%
744
Which type of MI has an increased risk of VF?
STEMI
745
What is the consequence of papillary muscle dysfunction or rupture
Can cause mitral valve incompetence Usually occurs in the first few days
746
When does thromboembolism follow MI?
\>1w due to mural thrombi overlying infarcted tissue
747
How do aneurysms arise in AMI
End-stage scars can form large transmural infarcts which result in large bulging aneurysms \>4w
748
What is the triad in Dresslers?
Triad of fever, pleuritic pain and pericardial effusion
749
What are the causes of HF?
IHD Valve disease Myocarditis HTN Cardiomyopathy
750
Pathology seen in HF?
Dilated heart Scarring and thinning of the walls Pleural effusion Pulmonary oedema Hepatomegaly Ascites Peripheral oedema
751
Why does ventricular remodelling occur?
Neurohumoral imbalance, increased cytokine expression, immune and inflammatory changes and altered fibrinolysis. Leads to multiple effects
752
Hypertrophic heart leads to?
Diastolic heart failure
753
Dilated heart leads to?
Systolic heart failure
754
Features of liver in HF
nutmeg liver Venous congestion and fatty changes Hepatic cirrhosis with firosis
755
Def: cardiomyopathy
Intrinsic disease of heart muscle
756
Def: dilated cardiomyopathy
Progressive loss of myocytes, characterised by a dilated heart, HF, arrythmias and sudden death
757
Pathology of dilated cardiomyopathy
Myocytes replaced with fine interstitial fibrosis
758
Causes of dilated cardiomyopathy
Idiopathic Infective Toxic: ETOH, CTx, cobalt Fe Hormonal: hyper, hypothyroid, DM, peri-partum Genetics: haemochromatosis, Fabry's, Mc ARdle's Immunological: myocarditis
759
Def: hypertrophic cardiomyopathy
LV hypertrophy, characteristic fibre dissaray Familial in 50% (AD with variable penetrance) Thickening of the septum narrows the LV outflow tract
760
Clinical findings in hypertrophic cardiomyopathy
Systolic murmur Arrhythmias Hx of sudden death
761
Cause of HOCM
Sarcomere mutations (myosin heavy chain)
762
Def: restrictive cardiomyopathy
Walls are rigid, heart is prevented from stretching and filling with blood
763
Causes of restrictive cardiomyopathy
Priamry: Loffler's endocarditis Endocardial fibroelastosis Secondary: Infiltrative: amyloidosis, sarcoidosis, haemochromatosis Interstitial: postradiation fibrosis
764
Loeffler endocarditis
Loeffler endocarditis is a form of restrictive cardiomyopathy which affects the endocardium and occurs with white blood cell proliferation, specifically of eosinophils.[1] Restrictive cardiomyopathy is defined as a disease of the heart muscle which results in impaired filling of the heart ventricles during diastole.[1][2]
765
Features of chronic rheumatic valve disease?
Sequaelae of earlier rheumatic fever, predominantly left sided Mitral\>aortic\>tricuspid\>pulmonic
766
Which valve is most commonly affected in rheumatic fever?
Mitral
767
What are the pathological chagnes occuring to the valve in chronic rheumatic valvular disease?
Leaflets thicken Fusion of the commissures Chordae tendinae also thicken, shorten and fuse
768
What are Aschoff bodies?
Nodules found in hearts of individuals with rheumatic fever which resulted from inflammation of the heart muscle
769
What are the causes of AR?
Rigid: rheumatic and degenerative Destructive: microbial endocarditis Collapse: Prolapse through a VSD or due to myxomatous degeneration Disease of the arotic valve ring Mecahnical: if the heart dilates the valve may become insufficient to cover the dilated area, can occur in Marfan's dissecting aneurysms, ank spond, syphillitic aortitis and cystic medial degeneration of the aortic media
770
Presentation of MR
Middle aged woman with SOC, chest pain, mid systolic click, late systolic murmur
771
Histology of MR
Increased glycosaminoglycans which are prominent on Alvian blue stain
772
Types of pericarditis?
Fibrinous: MI and uraemia Purulent: staphylococcus Granulomatous: TB Haemorrhagic: tumour, TB, uraemia Fibours aka constrictive
773
What is usually the cause of pericardial effusion?
Chronic heart failure
774
Features of LV failure
Damming of blood within pulmonary circulation\_\> dyspnoea, orthopnoea, PND, wheeze, fatigue Eventually leads to reduced peripheral BP and flow
775
Ix in HF
BNP, CXR, ECG, Echo
776
What is the most common mutation in HOCM?
Beta MHC
777
Features of acute rheumatic fever?
Heart: pancraditis Joints: arthritis and synovitis Skin: erythema marginatum, subcut nodules Neurological: Encephalopathy, Sydenham's chorea
778
Clinical presentaiton of rheumatic fever
Clinical features develop 2-4w post strep throat
779
What is the main group causing acute rheumatic fever?
Lancefield GAS
780
Anitschkov myocytes?
Regenerating myocytes seen in acute rheumatic fever
781
Verrucae Aschoff bodies Anitschkov myocytes
Rheumatic fever
782
Treatment of acute rheumatic fever
Benpen Erythromycin if penallergic
783
What are the causes of vegetative endocarditis?
Rheumatic heart disease Infective endocarditis Non-bacterial thrombotic endocarditis (marantic) Libman Sacks endocarditis
784
Large irregular masses on valve cusps extending into the chordea
Infective endocarditis
785
Small, bland vegetations attached to lines of closure, formed of thrombi
Marantic
786
\<2mm vegetations that are sterile and platelet rich
Libman=Sacks Endocarditis
787
With what is Libman Sacks endocarditis associated?
SLE and anti-phsopholipid
788
What are the predisposing factors leading to infective endocarditis?
Rheumatic heart disease Mitral valve prolapse Calcified valves Bicuspid aortic valve Prosthetic valve Congential defects
789
What is infective endocarditis usually secondary to?
Bacteraemia as a consequence of poor dentalo hygiene IVDU Soft tissue infection Dental treatments Cannulae Cardiac Sx/pacemeakers
790
What are the casuative organisms in acute infective endocarditis?
Staph aureus Strep pyogenes
791
Pathology of acute infective endocarditis
Larger and more localised vegetations spreading into the aorta
792
What are the causative organisms in subacute infective endocarditis?
Strep viridans Staph epidermis HACEK Coxiella Mycoplasma Candida
793
Pathology in subacute infective endocarditis?
Friable, soft thrombi, small Extending into chordae
794
Clinical features of infective endocarditis
Fever, malaise, anaemia, rigors, splenomegaly, new murmur Roth spots Splinter haemorrhages Janeway lesions Osler's nodes Usually mitral/aortic unless IVDU
795
Dx of infective endocarditis
Duke's criteria
796
Rx in infective endocarditis?
Benpen + gentamicin
797
Site, pathology, aetiology: Chronic bronchitis
Bronchus Dilatation nof the airways and excess mucus production Tobacco smoke, air pollution
798
Site, pathology, aetiology: Bronchiectasis
Bronchus Airway dilatation and scarring
799
Site, pathology, aetiology: Asthma
Bronchus SM cell hyperplasia, excess mucus, inflammation Immunologic
800
Site, pathology, aetiology: Emphysema
Acinus Airspace enlargement, wall destruction Tobacco smoke, alpha-1 anti-trypsin deficiency
801
Site, pathology, aetiology: Bronchiolitis
Bronchiole Inflammatory scarring, obliteration Tobacco smoke, air pollutants
802
Clinical features of chronic bronchitis
Cough and sputum on most days for 3m over 2 years
803
Dilatation of the airways Goblet cell hyperplasia Hypertrophy of mucuous glands
Chronic bonchitis
804
Cx of chronic bronchitis
Reuccrent infections Chronic hypoxia Pulmonary HTN
805
Permanent dilatation of the bronchi
Bronchiectasis
806
Curschmann spirals Charcot-Leyden crystals
Asthma
807
Whorls of shed epithelium
Curschmann spirals
808
Whorls of shed epithelium Eosinophils Charcot-Leyden crystals
Asthma
809
Loss of the alveolar parnehcyma distal to the terminal bronchiole
Emphysema
810
Cx of emphysema
Pneumothorax Respiratory failure Pulmonary HTN
811
Cx of bronchiectasis
Recurrent infections Haemoptysis Pulmonary HTN Amyloidosis
812
Symptoms of bronchiectasis
Cough Purulent sputum Fever
813
Causes of bronchiectasis
Inflammatory: Post-inefctious Abnromal host defence: hypogammaglobinaemia, CTx Ciliary dyskinesia: Kartageners and 2o Obstruction Post-inflammatory (aspiration) Secondary to bronchiolar disease and interstitial fibrosis Systemic disease Asthma Congenital: CF Primary ciliary dyskinesia Hypogammaglobulinaemia
814
Def: interstitial lung disease
Group of \>200 disease characterised by inflammation and fibrosis of the pulmonary connective tissue, particulary the most peripheral and delivate interstitium of the alveolar wall
815
What are the features of interstital lung disease
Restrictive features on spirometry: Decreased CO diffusion capacity Decreased lung volume Decreased compliance
816
Presentation of interstitial lung disease
SOB End-inspiratory crackles Cyanosis, pulmonary HTN and cor pulmonale All have honey-comb lung in end stage
817
Honey-comb lung
Interstitial lung disease
818
How can interstitial lung disease be characterised?
Fibrosing: Idiopathic pulmoanry fibrosis Pneumoconiosis Crpyotgenic organising pneumonia Associated with CTD Drug-induced Radiation pneumonitis Granulomatous: Sarcoid EAA Associated with vasculitides e.g. Wegener's, Churg-Staruss, microscopic polyangitis Eosinophilic Smoking related
819
Features of cryptogenic fibrosin alveolitis/idiopathic pulmonary fibrosis
M\>F Unknown causative agents Usual intersitital pneumonia required for Dx (also seen in CTD, asbestosis and EAA)
820
Progressive patchy, interstitial fibrosis with loss of normal lung architecture and honeycomb change, beginning at the periphery of the lobule, usualy sub-pleural Hyperplasia of type 2 pneumocytes causing cyst formation= honeycomb
Usual Interstitial fibrosis
821
Clinical presentation of Idiopathic interstitial fibrosis
Increasing exertional dyspnoea and non-productive cough 40-70y/o Hypoxaemia-\> cyanosis and pulmonary HTN +/- cor pulmonale and clubbing
822
Rx of interstitial fibrosis
Steroids Cyclophosphamide Azathioprine (Limited impact on survival)
823
Which CTDs can have prominent lung fibrosis?
RA, systemic sclerosis, SLE
824
Def: pneumoconiosis
Typically and occupational lung disease: non-neoplastic reaction to inhalation of mineral dusts or inorganic particles Majority affect the upper lobe e.g. coal workers penumoconiosis, silicosis, asbsestosis (tends to affect the lower lobe)
825
Granulomatous infections
TB, fungal (histoplasma, cryptococcus, coccidioides, aspergillus, pneumoystitsi) parasites
826
Non-infectious granulomatous disease
Sarcoid Foreign body Drugs or occupational lung disease
827
What are the group of immune-mediated lung disorders characterised by intense/prolonged exposure to inhaled organic antigens leading to widespread **alveolar** inflammation
EEA Hypersensitivity pneumonitiis Crypotgenic organisng pneumonia Bronchiolitis obliterans organising pneumonia
828
Polypoid plugs of loose connective tissue within alevoli/bronchioes?
Feature of granuloma formation and organisn pneumonia
829
Acute presentation of EEA etc
Inhalation of antigenic dust in sensitised individual leading to systemic symptoms (fever, chills, chest pain, SOB, cough) within hours of exposure Usually settles by following day Progresses to EEA
830
Progressive, persistent productive cough and SOB Finger clubbing and severe weight loss
EEA
831
Different types of EEA
Farmers lung: moudly hay - Saccharopolyspora rectivirgula Pigeon fanciers lung - Proteins in excreta/feathers Humidifiers lung - heated reservoirs Malt-workers lung: - germinating barley Cheese washers loung: - fungi
832
What are the different categhroies of penumonia?
Bronchopneumonia: patchy bronchial/peri-bronchial distribution. Low virulence organisms Lobar pneumonis: fibrinosuppurative consolidation Atypical: interstitial pneumonitiis without intra-alveolar inflammation
833
Stages of lobar pneumonia
Consolidation Red hepatisation (neutrophilia) Grey hepatisation (fibrosis) Resolution
834
What are the diseases oif the pulmonary vasculature
PE Pulmonary HTN
835
What are the tumours of the lung
Squamous CC Adenocarcinoma Small CC Large CC Mesothelioma
836
What is the most common tumour of the lung?
Squamous CC
837
Which lung cancers are more common in smokers?
Squamous cell Small cell
838
Features of squamous cell carcinoma
Closely correlated with smoking M\>F Usually proximal bronchi, local spread with late metastasis Less responsive to chemo Variety of subtypyes: papillary, basaloid, Associated with caviation and hypercalcaemia
839
Progression of squamous cell carcinoma
Epithelium Hyperplasia Squamous metaplasia Angiosquamous dysplasia Carcinoma in situ Invasive carcinoma
840
Keratinisation, intercellular prickles Sqamous cells on cytology
Squamous cell lung carcinoma
841
Features of lung adenocarcinoma
Most common in womena dn non smokers Malignant epithelial tumour with glandular differntiation or mucin production Occurs peripherally and metastasises early
842
Histo: glandular differentiation Cyto: cells containing mucin vaculose EGFR mutations
Adenocarcinoma
843
Progression of adenocarcinoma
Atytpical adenomatous hyperplasia-\> non mucinous BAC-\> mixed pattern adenocarcinoma
844
Features of small cell carcinoma
Usually occurs centrally in the proximal bronchi Arises from neuroendocrine cellls Associated with ectopic ACTH secretion, Lambert-Eaton, cerebellar degeneration Highly malignant with early metastasis usually by diagnosis Poor prognosis p53 and RB1 mutations common
845
Lambert-Eaton
Lambert-Eaton myasthenic syndrome (LEMS) is an autoimmune disease — a disease in which the immune system attacks the body's own tissues. The attack occurs at the connection between nerve and muscle (the neuromuscular junction) and interferes with the ability of nerve cells to send signals to muscle cells.
846
Paraneoplastic cerebellar degeneration syndrome
Paraneoplastic syndromes are a group of rare disorders that are triggered by an abnormal immune system response to an underlying (usually undetected) malignant tumor. Patients with paraneoplastic neurological syndrome (PNS) most often present with neurologic symptoms before an underlying tumor is detected. Paraneoplastic neurologic syndromes include many neurologic disorders including paraneoplastic cerebellar degeneration (PCD) caused by an immune-mediated mechanism other than a metastatic complication in patients with an underlying malignancy. Any part of the nervous system can be involved depending on the type of primary malignancy. These syndromes affect 1-3% of all cancer patients.[1] These syndromes are difficult to diagnose and respond poorly to treatment. However, the oncologic outcome of patients with antibody-associated paraneoplastic syndromes does not significantly differ from that of patients who do not have the antibodies or a paraneoplastic syndrome.
847
Anti-Yo
Paraneoplastic cerebellar degeneration
848
Features of large cell carcinoma
Poorly differentiated malignant epithelial tumour: large cells, large nuclei with prominent nuceloli No evidence of glandular or squamous differentiation Associated with paraneoplastic syndromes: ADH, ACTH, PTH, PTHrP, Calcitonin, serotonin, bradykinin
849
NSCLC: ERCC1
Poorer response to cisplatin
850
EGFR in adenocarcinoma
Target for anti-EGFR TKI therapy
851
Kras mut in adeno/squamous cell carcinoma
Poor prognosis- no response to TKI
852
EML4-ALK
Usualy adenocarcinoma= no benefit from TKI
853
Staging of lung malignancy
T1-4 based on size, invasion of pleura and pericardium LN metas: N0-2 (N0- no LNs, N1-N2 LNs involved, dependant on extent) M0-1: mets or na
854
Features of mesothelioma
Arises from either parietal or visceral pleura Spreads within pleural space Associated with pleural effusion, chest pain and dyspnoea Latent period of 25-45 following asbestos exposure
855
How do large PEs affect pulmonary vasculature
Impact in the main pulmonary arteries leading to acute cor pulmonale, cardiogenic shock, and death if \>60% of pulmonary bed occluded
856
Saddle embolus=
Occluding the pulmonary trunk
857
How do small emboli impact pulmonary vasculature
May be silent or cause peripheral wedge infarctions Repeated infarctions can lead to pulmonary HTN
858
What are non-thrombotic emboli
BM, amniotic fluid, tumour, air, foreign body
859
Def: pulmonary HTN
Mean pulmonary arterial pressure \>25mmHg at rest
860
Causes of pulmonary HTN
Can be primary More often secondary due to: Chronic lung disease Heart diseases Recurrent thrombo-emboli Autoimmune disorders
861
What are the group most commonly affected by primary pulmonary HTN?
Women aged 20-40
862
How can secondary causes of pulmonary HTN be classified?
Pre-capillary: Chronic hypoxia/embolus Capillary: Fibrosis Post-capillary: Left heart disease, veno-occlusive disease
863
Def: pulmonary oedema
Intra-alvolar fluid accumulation leading to poor gas exchange, main aetiology is left heart failure
864
Iron-laden macrophages=
Heart failure cells Seen in pulmonary oedema due to heart failure
865
Causes of diffuse alveolar damage
ARDS in adults Neonates: hyaline membrane disease of newborn, insufficient surfactant production
866
Common congenital disorders of the lung?
Lung agenesis or hypoplasia Tracheal and bronchial stenosis Congeital cysts
867
Causes of ARDS
Infection Aspiration Trauma Inhaled irritant gases Shock Blood transfusion DIC Drug overdose Pancreatitis Idiopathic
868
869
Macroscopic features of asthma?
Mucus plug Overinflated lungs
870
What are the two types of emphysema and their causes?
Paracinar: caused by anti-trypsin deficiency Centrilobular: caused by smoking, loss of the centre of the bronchiole
871
What are the organs affected by CF?
Lung GIT: meconium ileus and malabsroption Pancreas: pancreatitis and malabsorption Liver: cirrhosis Male reproductive system: infertility
872
How does chronic hypoxia cause pulmonary HTN
Normal response of lung is to reduce blood flow to hypoxic areas Results in chronic vasoconstriction of the pulmonary arterioles which leads to morphological changes e.g. eccentric intimal fibrosis, thickening of the muscle wall
873
What is pulmonary veno-occulsive disease
Fibrotic occlusion of the pulmonary veins can be caused by: Idiopathic Herbal teas and diet pills CTx RTx BM transplant Renal transplant HIV Systemic sclerosis
874
Dx of idiopathic pulmonary fibrosis
HRCT +- biopsy
875
What is an issue with bevacizumab in squamous cell carcinoma
Some patients develop fatal haemorrhages with some durgs as it is an angiogenesis inhibitor
876
Stain for adenocarcinoma
TTF-1
877
Stain for SqCC
CK5/6, P63, TTF-1
878
Cx of lung cancer
Bronchial obstruction Invasion of local structures Extension through pleura or pericardium Diffuse lymphatic spread within lungs Metabolic spread Paraneoplastic ssyndromes
879
What are the paranetoplastic syndromes seen in lung cancer and their consequences?
ADH: SIADH causing hyponatraemia ACTH: Cushings PTHrP: hypercalcaemia Calcitonin; hypocalcaemia Gonadotrophin: gynaecomastia Serotonin: carcinoid syndrome Nonednocrine: haematological/coagulation defects
880
Consequences of lung cancer invading local structures
SVCO Oesophagus: dysphagia Horner's syndrome
881
Pathology in chronic meningitis?
Leptomeninges and sometimes dura thickened Arachnoid adhesions may be present Lymphocyte, plasma cell and epitheloid macrophages seen in the infiltrate Caseous necrosis and granulomatous inflammation may be present in TB Adhesions can cause a non-communicating hydrocephalus
882
What are the leptomeninges?
Pia and arachnoid mater and the subarachnoid space Inflammation usually due to infection.
883
Features of cystitis
Can be acute or chronic 85% caused by gram negative bacilli that are normally resident in GIT e.g. E Coli etc
884
What drug is associated with cystitis?
Cyclophosphamide with haemorrhagic cystitis
885
What are the risk factors for cystitis
Female Calculi Urinary obstruction DM Sexually active Instrumentation (Cyclophosphamide)
886
What are the types of bladder tumours?
Transitional cell Squamous cell Adenocarcinoma
887
What is the most common bladder malignancy?
Transitional cell
888
Epidemiology of TCC
90% of all bladder tumours Male\>Female 50-80y/o
889
Risk factors for bladder TCC
Cigarette smoking Aromatic amines
890
Clinical features of TCC
Painless haematuria, freuqency, urgency, pyelonephritis or hydronephrosis if ureteral orifices involved
891
Dx of TCC
Cystoscopy + biopsy
892
With what is squamous CC of the bladder associated?
Schistosomiasis
893
What is the aetiology of bladder adenocarcinoma?
Rare, arising from extensive intestinal metaplasia or from urachal remnants
894
What is the urachus?
The urachus is a remnant of a channel between the bladder and the umbilicus (belly button) where urine initially drains in the fetus during the 1st trimester of pregnancy. The channel of the urachus usually seals off and obliterates around the 12th week of gestation and all that is left is a small fibrous cord between the bladder and umbilicus called the median umbilical ligament.
895
What is BPH
DHT-mediated hyperplasia of prostatic stromal and epithelial cells resulting in the formation of large nodules
896
Symptoms of BPH
Difficulty urinating Retention Frequency Nocturia Overflow dribbling
897
Features of chronic pyelonephritis and reflux nephropathy
Tubulo-interstitial inflammation causing scars, leads to blunting of the renal calcyes and deformation, Xanthogranulomatous PN can result from Gram -ve infections
898
Mx of BPH
TURP 5 alpha reductase inhibitors
899
What is the most common form of prostate Ca
Adenocarcinoma Commonly seen in older gentlemen
900
What is the scoring system for prostate carcinoma?
Gleason scoring: Pattern 1- well differentiated tumours, uniform round glands packed into circumscribed nodules Pattern 5: no glandular differentiation Final score= predominant pattern plus worst pattern
901
Gleason grading
Grade 1 \<6 Grade 2 6+7 Grade 3 \>8
902
Dx of prostate adenocarcinoma
Hx Ex PSA (.4ng/ml)
903
Mx of prostate adenocarcinoma
Stage T1/T2: RTx and Sx T3: Extraprostatic spread: external beam radiation
904
Causes of acute prostatitis?
E Coli and other gram -ve rods Fever, chills and dysuria
905
Features of chronic abacterial prostatitis
Manifestation similar to chronic bacterial but without hx of recurrent UTI Culture negative Dysuria or mild suprapubic discomfort
906
Chronic bacterial prostatitis?
Symptoms of recurrent UTI
907
What is the most common cause of granulomatous prostatitis?
Bladder BCG for treatment of carcinoma in situ Non-specific granulomatous prostatitis Symptoms of urinary incontinence and pelvic discomfort
908
What are the most common types of testicular tumours?
Germ cells
909
Draw the types of testicular tumours
910
What is the most common type of germinal tumour?
Seminoma
911
What is the impact of undescended testes on cancer risk?
10x
912
What is PIN
Prostatic intraepithelial neoplasia Common in young men 33% risk of carcinoma Doesn't cause an elevated PSA but examine for carcinoma
913
What are the markers for germ cell tumours?
AFp beta HCG LDH
914
Seminoma: age group
15-34y
915
Seminoma marker?
Beta HCG (AFP and ALP not raised)
916
What is the most common testicular carcinoma in \<3y/o?
Embryonal carcinoma and yolk sac tumour Aggressive and malignant AFP
917
Marker of endodermal sinus tumour?
AFP
918
Marker of choriocarcinoma?
HCG
919
What is a condyloma acuminatum?
Genital wart
920
What is the malignant penile lesion?
Squamous cell carcinoma HPV-related
921
Clinical features of renal cell carcinoma?
Costovertebral pain Palpable mass Haematuria Paraneoplastic syndromes: Polycythaemia, hypercalcaemia, HTN, Cushing's, amyloidosis
922
Risk factors for testicular tumours?
Cryptorchidism Testicular dysgenesis Klinfelters Testicular feminisation
923
What are the types of renal carcioma
Clear cell Papillary: commonest in dialysis-associated cystic disease Chromophobe renal carcinoma: pale eosinophilic cells
924
What are the most common organisms causing PID
Chlamydia Gonorrhoea
925
Symptoms of PID
Lower abdo pain Dyspareunia Vaginal bleeding/discharge Fever Adnexal tenderness Cervical excitation
926
Fitz-Hugh Curtis
RUQ from perihepatitis Violin string peri-hepatic adhesions
927
Cxs of PID
FHC syndrome Infertility Increased risk of ectopic Intestinal obstruciton-\> bacteraemia Tubo-ovarian abscess Peritonitis Plical fusion
928
Def: endometriosis
Presence of endometrial glands or stroma outside of uterus 10% of women
929
What are the 3 theories around the aetiology of endometriosis?
Metaplasia of pelvic peritnoum Implanatation of endometrium through retrograde menstruation Induction of metaplasia
930
Pelvic pain Dysmenorrhoea Deep dyspareunia Reduced fertility Nodules/tenderness in posterior fornix or uterus Immobile uterus which is anteverted in severe disease
Endometriosis
931
Red-blue to brown nodules "powder burns" Choclate cysts in ovaries
Endometriosis Endometriomas
932
Def: adenomysosis
Ectopic endometrial tissue deep within the myometrium causing dysmenorrhoea May cause menorrhagia and deep dyspareunia Globular uterus
933
What is the most common tumour of the female genital tract?
Leiomyoma
934
Draw the location of fibroids
935
What is important re fibroids
Oestrogen stimulation: Enlarge during pregnancy Regress post-menopausally
936
Macroscopic and microscopic features of leiomyomas?
Sharp, circumscribed, discrete, round, firm, grey-white tumours of variable size Bundles of smooth muscle cells
937
Clinical features of fibroids
Menorrhagia Dysmenorrhoea Pressure effects: urinary frequency, tenesmus Subfertility Red degeneration in pregnancy May rarely transform to leiomyosarcoma
938
Leiomyosarcoma aetiology
May be due to transformation of leiomyoma More common post-menopausally
939
How is endometrial carcinoma staged?
FIGO Stage 1: confined to uterus Stage 2: involves cervix Stage 3: spread to adenezae, serosa, positive peritoneal cytlogy, LNs (pelvic or para-aortic) Stage 4: pelvic organs and distal spread
940
Draw the division of endometrial cancer types
941
What are the features of endometroid carcinoma
Look similar to normal endometrial glands Associated with oestrogen excess, uusally in peri-menopausal women
942
Risk factors for endometroid endometrial carcinoma
E2 excess: OBESITY Anovulatory amenorrhoea (PCOS) Nulliparity Early menarche Late menopause Tamoxifen DM HTN
943
What is the more aggressive type of endometrial carcinoma?
Non-endometroid
944
Features of VIN
Dysplasia of epithelium associated with HPV I, II and III Progression to invasive disease is lower than for CIN
945
Significanece of Paget's disease of vulva?
Adenocarcinoma in situ, rarely associated with invasive adenocarcinoma
946
Tissue type in VIN
Generally squamous cell
947
Draw the organisaiton of ovarian tumours
948
Psammoma bodies, columnar epithelium=
Serous epithelial tumour
949
No psammoma bodies Associated with pseudomyxoma peritonei
Mucinous tumour of ovary
950
Clear cytoplasm with hobnail appearance
Clear cell epithelial tumour of ovary
951
What is the female counterpary of the seminoma
Dysgerminoma
952
What is a mature teratome?
Dermoid cyst Contains skin, hair, teeth, etc.
953
What are immature teratomas
Germ cell tumours of the ovary Malignant and usually solid, contain immature, embryonal tissues
954
Meig's syndrome associated with?
Fibroma of ovary
955
Which ovarian malignancy produces E2?
Granulosa-Theca cell tumour
956
Which ovarian malignancy produces androgens?
Sertoli-Leydig tumour
957
What are the secondary ovarian tumours?
Endometroid, serousus, clear cell carcinomas: may be uterine primary Krukenbergy tumours: metastatic mucin producing adenocarcinomas from stomach or breast
958
Eponymous name for metastatic mucin-producing adenocarcinoma from stomach/breast
Krukenberg tumour
959
What HPV strains associated with CIN
Commonly 16 and 18
960
What is the transitional zone
Where the squamous cell transforms into columnar epithelium
961
CIN I-III
I: dysplasia confined to lower 1/3 II: lower 2/3 III: full thickness but with bm intact
962
Px of CIN grades
I: reverts to normal 30% of 3 progresses to cervical cancer over 10 yeras
963
What is CGIN
Less common, treatment requires excision of entire endocervix
964
What is the most common form of cervical carcinoma
Squamous cell although 20% can be adenocarcinomas etc.
965
What marks the change from CIN to carcinoma
Invasion through basement membrane
966
Clinical features of cervical carcinoma
PCB IMB Post MB Pain
967
Features of SLE
Autoimmune multi-system disorder Type III hypersensitivity reaction Increased in classical complement deficiciencies. Can be drug-induced Higher prevalence in Afrocarribean and women
968
What HLA is associated with SLE
HLA DR3 (or 2)
969
Autoantibodies in Lupus
ANA (95%) Anti dsDNA Anti-Sm Drug-induced Anti-histone
970
LE bodies Small vessel angiopathy Onion skin lesions in the spleen Libman-Sack endocarditis
Lupus
971
Signs and symptoms of Lupus SOAP BRAIN MD
Serositis Oral ulcers Arthritis Photosensitivity Blood disorders (AIHA, ITP, leucopenia) Renal involvement ANA +ve Immune phenomena (dsDNA, anti-SM, antiphospholipid Ab) Neuro symptoms Malar rash Discoid rash
972
Lupus
973
What are the two types of scleroderma?
Limited cutaneous (CREST) Diffuse scleroderma
974
Features of scleroderma
Autoimmune multi-system disorders SKin fibrosis
975
HLA associations in scleroderma
HLA DR5 and DRw8
976
Autoantibody in CREST (limited cutaenous)
Anti-centromere
977
Anti-centromere
CREST (limited cutaenous)
978
Increased collagen in skin and organs Onion skin thickening of arterioles
CREST
979
Signs and symptoms of CREST
Skin changes on face and distal to elbows and knees Calcinosis Raynauds Esophageal dysmotility Sclerodactyly Telangiectasia Associated with pulmonary HTN
980
Sclerodactyl CREST
981
Lung disease in scleroderma
CREST= pulmonary HTN Diffuse= pulmoanry fibrosis
982
Autoantibodies in diffuse scleroderma
Anti Scl-70 Fibrillarin RNA pol I, II, III PM-SCl
983
Anti Scl-70 Fibrillarin RNA pol I, II, III PM-SCl
Diffuse scleroderma
984
Inflammation within or around muscle fibres
Diffuse scleroderma
985
Signs and symptoms of diffuse scleroderma
Skin changes can occur anywhere Widespread organ involvement Associated with pulmonary fibrosis
986
Features of polymyositis/dermatomyositis
Autoimmune inflammatory disorder of muscle +/- skin Associated with underlying malignancy
987
Autoantibody in dermatomyositis?
Anti-Jo1 Anti-Mi2
988
Anti-Jo1
Dermatomyositis
989
Autoantibody in polymyositis
Anti-signal recognition peptide Ab (anti-Jo1/ anti-Mi2 (D\>P))
990
Endomysial inflammatory infiltrate
Dermatomyositis
991
Drop out of acpillaires and myofibre damage
Dermatomyositis
992
Symtpoms and signs of polymyositis
Proximal limb, anterior neck weakness and oesophageal and respiratoy muscle involvement. Elevated skeletal muscle enzymes Abnormal EMG Positive muscle biopsy Associated with pulmonary fibrosis
993
Signs and symptoms of dermatomyositis
Proximal limb, anterior neck weakness and oesophageal and respiratoy muscle involvement. Elevated skeletal muscle enzymes Abnormal EMG Positive muscle biopsy + skin changes: Heliotrope rash Gottron Papules Assoc c. pulmonary fibrosis
994
Gottron papules Dermatomyositis
995
Heliotrope rash Dermatomyositis
996
What complement defects associated with lupus?
Defects in the classical pathway
997
What drugs can induce lupus?
Hydralazine Pocainamide
998
How is lupus diagnosed
\>4 of the ACR criteria
999
Ix used in Lupus
CRp ESR Test for ANA Complement
1000
Complement in active lupus
C3 (common): Normal C4 (classical): low i.e. undetectable
1001
Complement in severe active SLE
C3 (common): Low C4 (classical): low
1002
Complement in inactive SLE
C3: normal C4: normal
1003
CRP in Lupus
Generally normal
1004
ESR in lupus
High
1005
Anti-dsDNA in Lupus
High Can be used for disease monitoring
1006
Anti-cardiolipin Anti-Ro, La, Sm or RNP in SLE
Sometimes high
1007
Anti-topoisomerase=
Anti-SCl70
1008
What are the large vessel vasculitides?
Takayasu's Temporal arteritis
1009
What are the medium vessel vasculitides?
Polyarteritis Nodosa Kawasaki's Buerger's disease
1010
What are the small vessel vasculitides?
Wegener's Churg Strauss Microscopic polyangitis HSP
1011
Features of Takayasu's arteritis
Pulseless disease Increased in Japanese women Vascular symptoms: absent pulse, bruits, claudication Granulomatous vasculitis with massive intimal fibrosis
1012
Pulseless disease Increased in Japanese women Vascular symptoms: absent pulse, bruits, claudication Granulomatous vasculitis with massive intimal fibrosis
Takayasu's
1013
Features of temporal arteritis
Elderly Scalp tenderness, temporal headache Jaw claudication, blurred vision **ESR** raised Overlap with polymyalgia rheumatica
1014
Elderly Scalp tenderness, temporal headache Jaw claudication, blurred vision ESR raised Overlap with polymyalgia rheumatica
Temporal arteritis
1015
Granulomatous transmural inflammation and giant cells and skip lesions
Temporal arteritis
1016
What is the most common form of arteritis?
Temporal
1017
Features of PAN
Renal involvement is main feature Can involve other ograns Spares lungs 30% have underlying Hep B
1018
Renal involvement is main feature Can involve other ograns Spares lungs 30% have underlying Hep B
PAN
1019
Microaneurysms on angiography
PAN
1020
Necortising arteritis with inflammation, infiltration of polymorphs, lymphocytes and eosinophils Arteritis is focal and sharply demarcated Healed by fibrosis
PAN
1021
Features of Buerger's disease
Heavy smokers, usually men \<35 Inflammation of arteries of extremities: usually tibial and radial Pain, ulceration of toes, feet, fingers Corkscrew appearance on angiogram
1022
Heavy smokers, usually men \<35 Inflammation of arteries of extremities: usually tibial and radial Pain, ulceration of toes, feet, fingers Corkscrew appearance on angiogram
Buerger's disease
1023
Features of Wegener's granulomatosis
Upper resp tract: sinusitis, epistaxis, saddle nose Lower resp tract: cavitation, pulmonary haemorrhage Kidneys: cresecenteric glomerulonephritis cANCA +ve
1024
Triad in Wegeners
Upper RT Lower RT Kidneys
1025
Saddle nose Pulmonary haemorrhage Crescenteric GN
Wegener's
1026
cANCA +ve
Wegener's
1027
Churg Strauss
Asthma, allergic rhinitis, eosinophilia Later systemic involvement pANCA (anti-PR3) +ve
1028
Asthma, allergic rhinitis, eosinophilia Later systemic involvement
Churg-Strauss
1029
pANCA (anti-PR3) +ve
Churg Strauss Microscopic polyarteritis
1030
Granulomatosis with polyangitis
Wegener's
1031
Eosinophilic granulomatosis with polyangitis
Churg Strauss
1032
What are the 3 types of ANCA-associated vasculitis
Microscopic polyangitis Wegener's Churg Strauss
1033
Features of microscopic polyangitis
Pulmonary renal syndrome: Pulmonary haemorrhage Glomerulonephritis
1034
Features of HSP
IgA mediated vasculitis Affects children URTI precedes Palpable purpuric rash, abdo pain, GN, arthritis, Orchitis
1035
IgA mediated vasculitis Affects children URTI precedes Palpable purpuric rash, abdo pain, GN, arthritis, Orchitis
HSP
1036
Features of Cryoglobulinaemia
Presence of abnormal proteins in the blood which becomes thick or gel-like in cold temperatures Cryoglobulins are Abs Cause a range of Sx from skin rashes to kidney failure
1037
What is amyloidosis?
Multisystem disorder caused by abnormal protein folding that are subsequently deposited as amyloid fibrils in tissues disrupting their function
1038
How can amyloidosis be classified?
Primary and Secondary
1039
What protein involved in primary amyloidosis?
AL Derived from light chains therefore more common in patients with myeloma
1040
What protein involved in secondary amyloidosis?
AA Derived from serum AA which is an acute phase protein Therefore secondary to chronic infection/inflammation
1041
Associations of primary amyloidosis
Most common form Most associated with plasma cell dyscrasias Most have Bence Jones protein in urine and increased plasma cells
1042
Causes of secondary amyloidosis
Autoimmune disease: RA, Ank Spond, IBD Chronic infections: TB, osteomyelitis, IVDU Non-imune: RCC, HL
1043
What is seen in haemodialysis associated amyloidosis?
Deposition of beta2-microglobulin
1044
What is the most common form of familial amyloidosis?
Familial Mediterranean Fever
1045
Features of Familal Mediterranean Fever?
++++IL-1 production leading to attacks of fever and inflammation of serosal surfaces AA amyloid, with predominant renal deposition
1046
Clinical features of amyloidosis?
Caused by amyloid deposits in various organs: Kidney: nephrotic syndrome Heart: conduction defects, heart failure Liver/spleen: hepatosplenomegaly Tongue: macroglossia Neuropathies: carpal tunnel
1047
Apple green birefringence with Congo red stain under polarised light?
Amyloidosis If not polarised light appears pink/red
1048
Def: sarcoidosis
Multisystemic disease of unknown cause Commonly affects young adults Characterised by non-caseating granulomas in many tissues
1049
Non-caseating granulomas, Schaumann and asteroid bodies
Sarcoidosis
1050
Features of sarcoidosis
Most severe disease in Afro-Carribeans Lungs most commonly involved Often detected at routine CXR Most seek help with pulmonary symptoms
1051
Bilateral hilar lymphadenopathy May also see fine nodular shadowing in mid zones Sarcoidosis
1052
What are the extrapulmonary manifestations of sarcoidosis
Skin: erythema nodosum, lupus pernio, skin nodules LNs: lymphadenopathy, painless and rubbery Joints: arthritis, bone cysts Eyes: anterior uveitis-\> misting of vision and painful red eye Posterior uveitis-\> progressive visual loss Hepatosplenomegaly Leukopaenia/anaemia Hypercalcaemia/hpercalcuria Myocardial involvement CNS involvement Keratoconjuncitvits sicca and lacrimal gland enlargement
1053
Symptoms of anterior uveitis
Misting of vision and painful red eye
1054
Symptoms of posterior uveitits
Progressive visual loss
1055
Dx of sarcoidosis?
Dx of exclusion Raised Ca Raised ESR Raised ACE Transcbronchial biopsy
1056
Lupus pernio
1057
ACE in sarcoid
ngiotensin converting enzyme (ACE) participates in the renin cascade in response to hypovolemia. Its peptidase action on the decapeptide angiotensinogen I results in the hydrolysis of a terminal histidyl leucine dipeptide and the formation of the octapeptide angiotensin II, a potent vasoconstrictor that increases blood pressure. The primary source of ACE is the endothelium of the lung. ACE activity is increased in sarcoidosis, a systemic granulomatous disease that commonly affects the lungs. In sarcoidosis, ACE is thought to be produced by epithelioid cells and macrophages of the granuloma. Currently, it appears that ACE activity reflects the severity of sarcoidosis: 68% positivity in those with stage I sarcoidosis, 86% in stage II sarcoidosis, and 91% in stage III sarcoidosis. Serum ACE also appears to reflect the activity of the disease; there is a dramatic decrease in enzyme activity in some patients receiving prednisone. Other conditions such as Gaucher disease, leprosy, untreated hyperthyroidism, psoriasis, premature infants with respiratory distress syndrome, adults with amyloidosis, and histoplasmosis have been associated with increased levels of ACE.
1058
1059
http://www.lab.anhb.uwa.edu.au/mb140/addons/mcqquiz.htm
Try it
1060
What are the features of acute mastitis
Painful red breast, fever Almost always occur during lactation and are due to staphylococcal infection via cracks in the nipples Involved breast tissue is necrotic and infiltratred by neutrophils
1061
Mx of acute mastitis?
Continued expression of milk Abx +/- surgical drainage
1062
Features of periductal mastitis
Mostly in smokers and is not associated with lactation
1063
Keritanising squamous epithelium extending deep into nipple duct orificies
Periductal mastitis
1064
Features of duct ectasia
Inflammation and dilatation of the large breast ducts Poorly defined palpable periarolar mass with thick white nipple secretions Caused by granulomatous inflammation and dilation of large breast ducts Mimics mammographic appearance of cancer
1065
Features of fat necrosis
Inflammatory reaction to damaged adipose tissue Presents as painless breast mass/skin thickening/mammographic lesion Caused by trauma, surgery, radiotherapy
1066
What are the benign proliferative breast conditions
Fibrocystic disease/ fibroadenosis Gynaecomastia
1067
What are the changes in breast tissue seen in fibrocystic disease?
Cystic change: small cysts form by dilation of lobules, contain fluid and often calcified Fibrosis: inflammation and fibrosis secondary to cyst rupture Adenosis: increased number of acini seen per lobule Exagerrated response to hormonal influence
1068
Breast lumpiness
Fibrocystic disease
1069
Features of gynaecomastia
Unilateral or bilateral enlargement of the male breast Indicative of hyperestrinism: ETOH, age, liver cirrhosis, functioning testicular tumour Epithelial hyperplasia with finger-like projections into ducts
1070
What are the benign proliferative conditions of the breast
Fibroadenoma Duct papilloma Radial scar (Phyllodes tumour)
1071
Features of fibroadenoma
Most common benign tumour, from stroma, often multiple and bilateral Occuring at any age within the reporudcitve period Epithelial response to size therefore increase during pregnancy and calcify after menopause
1072
Mobile, well circumscribed, spherical, rubbery breast lump in young woman
Fibroadenoma
1073
Features of duct papilloma
Benign papillary tumour arising within the duct system of the breast Can be peripheral (within small terminal ductules) or central (larger lactiferous ducts) Causes bloody discharge, not seen on mammogram, Ix with galactogram
1074
Mx of intraductal papilloma
Excision of the affected duct is curative
1075
Features of Radial scar
Benign sclerosing lesion of the breast Central scarring surrounded by proliferating glandular tissue in stellate pattern Resembles cancer on mammogram
1076
Complex sclerosing lesion of the breast
Radiat scar
1077
What are the proliferative breast diseases?
A diverse group of intraductal proliferative lesions of the breast associated with an increased risk of the development of subsequent invasvie breast carcinoma Usually microscopic asymptomatic lesions Usual epithelail hyperplasia Fat epithelial atypia: low grade precursor to DCIS In situ lobular neoplasia
1078
What are the risk factors for breast carcinoma
Susceptiblity genes Hormone exposure Age FHx Race: caucasian\>Afrocarribean\>asian\>hispanic Obesity, tobacco, alcohol
1079
What are the susceptibility genes associated with breast cancer
BRCA1/BRCA2 Increased risk of ovarian, prostate and pancreatic malignancy BRCA mutations cause a lifetime risk of breast carcinoma of up to 85%
1080
How does hormone exposure impact on breast cancer
Early menarche Late menopause Late 1st live birth (pregnancy leads to terminal differentiation of molk-producing luminal cells removing these from the potential cancer pool) OCP/HRT
1081
Presentation of breast cancer
Hard fixed lump Paget's disease of breast Peau d'orange Nipple retraction
1082
Paget's disease of breast
1083
Breast cancer screening programme
47-73 y/o women invited every 3 years for mammography
1084
Features of DCIS
Neoplastic intraductal epithelial proliferation with an inherent but not inevitable risk of progression to breast cancer Limited to ducts/lobules by basement membranes Increased incidence since mammography Appear as areas of microcalficiation
1085
Features of LCIS
Always incidental finding on biopsy as no microcalcifications or stromal reactions 20-40% bilateral Cells lack adhesion protein
1086
Features of invasive breast carcinoma
Malignant epithelial tumours which infiltrate hte breast with capacity to metastasise
1087
Low grade BCA
Arises from low grade DICS or in situ lobular neoplasia and show 16q loss
1088
What are the different histological classifications of invasive breast cancer
Ductal Lobular Tubular Mucinous
1089
Features of invasive ductal carcinoma
Carcinoma that cannot be classified into another group, most common
1090
Features of invasive lobular carcinoma
Cells aligned in single file chains/strands
1091
Features of tubular carcinomas
Well-formed tubules with low grade nuclei, rarely palpable as \<1cm
1092
Features of mucinous breast carcinoma
Cells produce abundant quantities of extracellular mucin which dissects into surrounding stroma
1093
What are the components of the triple assessment?
Examination Radiological examination (mammography, FNA, USS) FNA and cytology
1094
What are the components examined in histological grading of breast cancer?
Nuclear pleomorphism, tubule formation, mitotic activity
1095
What happens with all breast neoplasms
Investigated for receptor status: ER/PR: good prognosis- tamoxifen Her-2: bad prognosis
1096
What is the single most important factor in px of breast cancer
Axillary LN status
1097
Tamoxifen=
Mixed agonist/antagonist of ER
1098
Trastuzumab
Herceptin Monoclonal Ab vs Her2
1099
What is an important consideration for Herceptin?
Has a direct toxic effect on myocardium Must monitor LVEF
1100
Features of basal-like carcinoma of breast
Sheets ot atypical cells with lymphocytic infilrtate Stains positive for CK5/6/14
1101
Features of Phllodes tumour
Arise from interlobular stroma with increased cellularity and mitosies \>50y with a palpable mass Low or high grade lesions. Most benign but can be aggressive Therefore excised with wide margins to limit local recurrence
1102
Anterior pituitary secretions
1103
What are the symptoms of pituitary disease?
•Hyperpituitarism –Excess secretion of trophic hormones –Usually due to functional adenoma •Hypopituitarism –Deficiency of trophic hormones •Local mass effects
1104
What is the most common type of pituitary adenoma?
Prolactinoma
1105
Pituitary microadenoma=
\<1cm
1106
What are the clinical effects of prolacitnomas
Amenorrhea, galactorrhoea, loss of libido, infertility
1107
Clinical effects of growth hormone adenomas
Prepubertal children- gigantism Adults- acromegaly DM, muscle weakness, HTN, congesitve cardiac failure
1108
Clinical effects of corticotroph cell adenomas
Cushins
1109
What are the significant causes of hypopituitarism
Nonsecretory pituitary adenomas Ischaemic necrosis: Sheehan's DIC, SCA, elevated ICP, Shock Pituitary ablation by sx or irradiation
1110
What are the clinical manifestations of anterior pituitary hypofunction
* Children - growth failure (pituitary dwarfism) * Gonadotrophin deficiency - amenorrhea and infertility in women. Decreased libido and impotence in men * TSH and ACTH deficiency - hypothyroidism and hypoadrenalism * Prolactin deficiency - failure of post-partum lactation
1111
What are the peptides released by the posterior pituitary
ADH Oxytocin
1112
What are the local mass effects of pituitary tumours
Compression of optic chiasm leading to bitemporal hemianopia Signs and symptoms of raised ICP Obstructive hydrocephalus
1113
Anterior pituitary
1114
thyroid
1115
Thyroid
1116
Physiology of the thyroid
Responds to TSH from ant pit Follicular cells convert thyroglobulin into T3 and T3 Effect is to increase BMR Also contain a population of C -cells (parafollicular) that promote Ca absorption by the skeletal system
1117
Features of non-toxic goitre
Enlargement of the thyroid Common if there is impaired synthesis of thyroid hormone, most often due to I deficiency Can be: Seen at puberty in females Due to ingestion of substances that interfere with thyroid hormone synthesis Due to hereditary enzyme defects
1118
Features of multinodular goitre
* With time simple thyroid enlargement may be transformed to a multinodular pattern * May reach massive size * May lead to mechanical effects including dysphagia and airways obstruction * A hyperfunctioning nodule may develop leading to hyperthyroidism
1119
Def: thyrotoxicosis
•Hypermetabolic state caused by elevated circulating levels of free T3 and T4
1120
How can thyrotoxicosis be classified?
Primary: Grave's Hyperfunctioning multinodular goitre Hyperfunctioning adenoma Thyroidits Secondary: TSH secreting pituitary adenoma (rare) Not associated with thyroid disease: Struma ovarii (ovarian teratoma with ectopic thyroid) Factitious thyrotoxicosis (exogenous thyroid intake)
1121
What is the most common cause of endogenous hyperthyroidism
Grave's
1122
Triad in Grave's?
Thyrotoxicosis Infiltrative opthalmoapthy with exopthalmos Infiltrative dermopathy (myxoedema) F\>M
1123
Exopthalmos
1124
Pretibial myxoedema
1125
Pathogenesis of Grave's disease
Autoimmune disorder caused by autoantibodies including those vs TSHR and thyroglobulin Abs vs TSHR most important-\> stimulate release of thyorid hormones Associated with other autoimmune diseases (hunt in packs)
1126
How can hypothyroidism be classified
Primary: Postablative Autoimmune- Hashimoto's Iodine deficiency Congenital biosynthetic defect Secondary: Pituitary or hypothalamic failure
1127
What is the most common cause of hypothyroidism
Hashimotos'
1128
Hashimoto's thyroiditis
1129
What is suggestive of thyroid maligiance in terms of the nature of thyroid nodules
Number- solitary Consistency- solid Age of patient- younger Sex- Male Iodine uptake- Cold
1130
Features of thyroid adenomas
* Usually solitary * Well circumscribed lesion that compresses the surrounding parenchyma * Well formed capsule * Small proportion cause thyrotoxicosis * Important to examine the capsule for invasion to exclude follicular carcinoma
1131
Features of papillary carcinoma
Nonfunctional Painless mass in neck Cervical mets 10y survival up to 90% * May occur at any age * May have papillary architecture * BUT diagnosis is based on nuclear features –Optically clear nuclei –Intranuclear inclusions •May be psammoma bodies
1132
Papillary carcinoma
1133
Features of follicular carcinoma
* Peak incidence in middle age * Follicular morphology * May be well demarcated with minimal invasion or clearly infiltrative * Usually metastasise via bloodstream to lungs bone and liver
1134
Features of medullary carcinoma
* Neuroendocrine neoplasm derived from parafollicular C cells * 80% sporadic - adults 5-6th decade * 20% familial - MEN - younger patients
1135
Features of anaplastic carcinoma
* Occur in elderly patients * Very aggressive * Metastases common * Most cases death within one year due to local invasion
1136
Functions of parathyroids
* Activity controlled by level of free calcium in blood * Decreased calcium stimulates release of PTH * PTH –Activates osteoclasts –Increases renal tubular reabsorption of calcium –Increases conversion of vitamin D to its active form –Increases urinary phosphate excretion –Increases intestinal calcium absorption
1137
Causes of hyperparathyroidism
* 80-90% - solitary adenoma * 10-20% hyperplasia of all 4 glands –Sporadic or component of MEN type 1 •\<1% carcinoma
1138
Causes of hypoparathyroidism
* Surgical ablation * Congenital absence * Autoimmune
1139
Clinical manifestations of hypoparathyroidism
–Neuromuscular irritability - tingling, muscle spasms, tetany –Cardiac arrhythmias –Fits –Cataracts
1140
Adrenal gland
1141
Zona glomerulosa secretes?
Aldosterone
1142
Zona fasciculata secretes?
GCs
1143
Zona reticularis secretes?
Androgens and GCs
1144
Adrenal medulla secretes
Nadr + Adr
1145
Manifestations of adrenocortical hyperfunction
* Cushing’s syndrome – excess glucocorticoids * Hyperaldosteronism * Virilising syndromes – excess androgens
1146
Clinical features of Cushing's syndrome
* Hypertension and weight gain * Truncal obesity * ‘moon’ facies * ‘buffalo hump’ * Cutaneous striae
1147
What is the most common cause of Cushing's
Administration of exogenous GCs Atrophic adrenal glands
1148
What are the endogenous causes of Cushing's syndrome
* \>50% due to primary hypothalamic-pituitary disease with increased ACTH – Cushing’s disease * Most associated with ACTH-producing adenoma in the pituitary * Some have hyperplasia of ACTH secreting cells in pituitary * Adrenal glands show nodular cortical hyperplasia * 30% of cases – primary adrenal * Most cases are due to a solitary neoplasm –Adenoma –Carcinoma •Less commonly due to bilateral hyperplasia
1149
What lung tumour associated with ectopic ACTH secretion
Small Cell Carcinoma Adrenals show bilateral hyperplasia
1150
1151
Features of hyperaldosteronism
•Primary –35 % aldosterone secreting adenoma – Conn’s syndrome –60 % bilateral adrenal hyperplasia –Clinical manifestations are hypertension and hypokalaemia –Accounts for \<1% of causes of hypertension but important to recognise as surgically correctable
1152
Conn's syndrome=
Primary hyperaldosteronism
1153
Adrenogenital syndromes and neoplasms?
More commonly associated with carcinoma than adenoma
1154
•Congenital adrenal hyperplasias
–Group of autosomal recessive disorders –Hereditary defects in enzymes involved in cortisol biosynthesis –Decreased cortisol results in increased ACTH, adrenal stimulation and increased androgen synthesis –May present in childhood or less commonly in adults
1155
Causes of adrenal insufficiency
Primary Secondary due to reduced ACTH
1156
Causes of acute primary adrenal insufficiency
–Sudden withdrawal of corticosteroid therapy –Haemorrhage (neonates) –Sepsis with DIC (Waterhouse-Friderichson syndrome)
1157
Waterhouse-Friderichson syndrome
Waterhouse–Friderichsen syndrome (WFS), hemorrhagic adrenalitis or fulminant meningococcemia is defined as adrenal gland failure due to bleeding into the adrenal glands, commonly caused by severe bacterial infection: Typically the pathogen is the meningococcus Neisseria meningitidis.
1158
Addison's disease
Chronic primary adrenal insufficiency
1159
Causes of chronic primary adrenal insufficiency
–Autoimmune (75-90%) –TB –HIV –Metastatic tumour (lung and breast particularly) –Rarely amyloid, fungal infections, haemochromatosis, sarcoidosis
1160
What are the adrenocortical neoplasms
•Adenomas –most non-functional –May be associated with Cushing’s syndrome or Conn’s syndrome •Carcinomas –Rare –Usually large –More commonly associated with virilizing syndrome than adenoma
1161
What are the neoplasms of the adrenal medulla
Phaeo Neuroblastoma
1162
Rule of 10s in phaeo?
–10% arise in association with a familial syndrome inc. MEN 2A and 2B, von Hippel-Lindau disease and Sturge-Weber syndrome –10% are bilateral –10% are malignant –In addition 10% of catecholamine-secreting tumours arise outside the adrenal (paragangliomas)
1163
A 65 year old man is in hospital after suffering an acute myocardial infarction. The house officer hears a pansystolic murmur on auscultation. ## Footnote A. Dilated cardiomyopathy B. Hypertrophic cardiomyopathy C. Myomalacia cordis D. Rheumatic fever E. Congenital septal defect F. Pericardial effusion G. Infective endocarditis H. Aortic stenosis I. Myxomatous mitral valve J. Pericarditis K. Mitral regurgitation L. Aortic regurgitation
Myomalacia cordis
1164
A 46 year old women presents to A&E out of breath and with severe chest pain. On examination a mid systolic click late systolic murmur is revealed. ## Footnote A. Dilated cardiomyopathy B. Hypertrophic cardiomyopathy C. Myomalacia cordis D. Rheumatic fever E. Congenital septal defect F. Pericardial effusion G. Infective endocarditis H. Aortic stenosis I. Myxomatous mitral valve J. Pericarditis K. Mitral regurgitation L. Aortic regurgitation
Myxomatous mitral valve
1165
Myomalacia cordis
Myomalacia cordis (weakness of cardiac muscle due to disintegration of dead myocytes): Rupture of weakened cardiac muscle may result in: · Mitral regurgitation: Apical thrill, systolic murmur and pulmonary oedema · Ventricular septal defect: Left sternal edge thrill, murmur, hypotension · External rupture: Cardiac tamponade
1166
A 68 year old smoker presents with jaundice and worsening abdominal and back pain. Scratch marks are seen on his arms and legs. He has lost 5kg in 2 months. Ultrasound shows dilated intrahepatic bile ducts. A. Carcinoma head of the pancreas B. Gallstones C. VIPoma (Werner Morrison syndrome) D. Haemochromatosis E. Vibrio cholerae infection F. Iatrogenic pancreatitis G. Insulinoma H. Renal tubular acidosis I. Gallstone pancreatitis J. Pseudocysts K. Hypercalcaemia L. Chronic alcoholic pancreatitis M. Cystic fibrosis N. Carcinoma tail of the pancreas
Carcinoma head of pancreas
1167
What differentiates between carcinoma of the head of pancreas and tail of pancreas?
Carcinoma of the head causes jaundice, tail does not
1168
A 59 year old widow complains of persistent back pain, loss of appetite and that she has dropped from dress size 18 to a size 14 in just 2 months. She was recently diagnosed with diabetes. A large central mass is palpable as well hepatosplenomegaly. A. Carcinoma head of the pancreas B. Gallstones C. VIPoma (Werner Morrison syndrome) D. Haemochromatosis E. Vibrio cholerae infection F. Iatrogenic pancreatitis G. Insulinoma H. Renal tubular acidosis I. Gallstone pancreatitis J. Pseudocysts K. Hypercalcaemia L. Chronic alcoholic pancreatitis M. Cystic fibrosis N. Carcinoma tail of the pancreas
Carcinoma tail of the pancreas
1169
A 39 year old Nepalese man presents with severe watery diarrhoea. He is found to have hypokalaemia and, surprisingly, a metabolic acidosis. A RUQ mass is detected by contrast-enhanced spiral CT scanning. Stool bicarb is high and urine anion gap is negative. A. Carcinoma head of the pancreas B. Gallstones C. VIPoma (Werner Morrison syndrome) D. Haemochromatosis E. Vibrio cholerae infection F. Iatrogenic pancreatitis G. Insulinoma H. Renal tubular acidosis I. Gallstone pancreatitis J. Pseudocysts K. Hypercalcaemia L. Chronic alcoholic pancreatitis M. Cystic fibrosis N. Carcinoma tail of the pancreas
VIPoma | (Werner Morrison syndrome)
1170
profound and chronic watery diarrhea and resultant dehydration, hypokalemia, achlorhydria, acidosis, vasodilation (flushing and hypotension), hypercalcemia and hyperglycemia.[3]
VIPoma Werner Morrison Syndrome
1171
Werner Morrison Syndrome
A VIPoma (also known as Verner–Morrison syndrome, after the physicians who first described it)[1] is a rare (1 per 10,000,000 per year) endocrine tumor,[2] usually (about 90%) originating from non-β islet cell of the pancreas, that produce vasoactive intestinal peptide (VIP). It may be associated with multiple endocrine neoplasia type 1. The massive amounts of VIP in turn cause profound and chronic watery diarrhea and resultant dehydration, hypokalemia, achlorhydria (hence WDHA-syndrome, or pancreatic cholera syndrome), acidosis, vasodilation (flushing and hypotension), hypercalcemia and hyperglycemia.[3]
1172
65 year old female with a large, cystic mass on tail of pancreas imaged using computed tomography. Further cytology reported the presence of epithelium A. Cystadenoma B. Carcinoma of the Pancreas C. Whipples' resection D. Cystic Fibrosis E. Gall Bladder F. Agenesis G. Alcoholism H. Jaundice I. Pancreatitis J. Thrombophlebitis K. Trousseau’s Syndrome L. Scorpion Sting M. Hyperlipidaemia N. Type 1 Diabetes O. Pancreas Divisum P. Pseudocyst
Cystadenoma
1173
55 year old, diabetic, afro-Caribbean male presents with weight loss, poor diet and a gnawing pain in his back, which is sometimes felt ‘under his chest’ A. Cystadenoma B. Carcinoma of the Pancreas C. Whipples' resection D. Cystic Fibrosis E. Gall Bladder F. Agenesis G. Alcoholism H. Jaundice I. Pancreatitis J. Thrombophlebitis K. Trousseau’s Syndrome L. Scorpion Sting M. Hyperlipidaemia N. Type 1 Diabetes O. Pancreas Divisum P. Pseudocyst
Carcinoma of the Pancreas
1174
What differentiates between a cystadenoma and a pancreatic pseduocyst?
Pancreatic pseudocyst is a fluid filled collection contained within a well-defined capsule of fibrous or granulation tissue or a combination of both. It does not possess an epithelial lining. A cystadenoma has an epithelial wall or capsule that contains a fluid collection.
1175
Caused by the action of acid and pepsin on the duodenal mucosa. Associated with increased output of stomach acid. Symptoms include pain in the upper abdomen, especially when the stomach is empty. A. Oesophageal varices B. Duodenal ulceration C. Pernicious anaemia D. Intestinal metaplasia E. Reflux oesophagitis F. Campylobacter jejuni G. Gastric cancer H. Helicobacter pylori I. Gastric ulcer J. Squamous carcinoma K. Barrett’s oesophagus L. Adenocarcinoma
Duodenal ulceration
1176
Around 10 % eventually get primary lymphoma (less often, carcinoma) of the gut if not properly treated. HLA B8 is linked with this A. Pernicious anaemia B. H. pylori infection C. Coeliac disease D. Intestinal metaplasia E. Normal oesophagus F. GORD G. Chronic gastritis H. Partial villus atrophy I. Normal stomach J. Peptic ulcer K. Barrett’s oesophagus
Coeliac
1177
A 30 year old female complaining of diarhorrea and weight loss. Biopsy of duodenum shows increased intraepithelial cytotoxic T cells. A. Acute Gastritis B. Pernicious Anaemia C. Barretts Oesophagus D. Oesophageal Adenocarcinoma E. Gastric Carcinoma F. GORD G. Coeliac Disease H. Oesophageal Varices I. Active Chronic Gastritis J. Duodenal Ulcer
Coeliac
1178
A 26 year old man presents with watery diarrhoea, abdominal cramps, nausea, vomiting and a low grade fever. It started 3 days after eating some undercooked meat at a barbecue. . Carcinoma of the oesophagus B. Whipple's disease C. Coeliac disease D. Cryptosporidiosis E. Partial villous atrophy F. Hiatus hernia G. Helicobacter pylori H. Gastric ulcer I. Gastro-oesophageal disease J. Barrett's oesophagus K. Microsporidiosis L. Mucosal associated lymphoid tumour M. Tropical sprue N. Pernicious anaemia O. Lymphoma P. Duodenal ulcer
Cryptosporidios
1179
A 58 year old female presents with malnutrition. She complains of abdominal pain, weight loss and arthritis. She has steatorrhoea. A jejunal biopsy showed periodic acid-Schiff (PAS)-positive macrophages A. Carcinoma of the oesophagus B. Whipple's disease C. Coeliac disease D. Cryptosporidiosis E. Partial villous atrophy F. Hiatus hernia G. Helicobacter pylori H. Gastric ulcer I. Gastro-oesophageal disease J. Barrett's oesophagus K. Microsporidiosis L. Mucosal associated lymphoid tumour M. Tropical sprue N. Pernicious anaemia O. Lymphoma P. Duodenal ulcer
Whipple's disease
1180
A 20-year-old student gives an 8 hour history of very frequent vomiting and epigastric cramping. O/E she is pale and shivering. Her serum WBC is normal. A. Gastroenteritis (Staphylococcus aureus) B. Gastric ulcer C. Mucosal-associated lymphoid tumour D. Gastro-oesophageal reflux disease E. Bulbar palsy F. Zollinger-Ellison syndrome G. Duodenal ulcer H. Adenocarcinoma I. Barrett's oesophagus J. Mallory-Weiss tear K. Gastroenteritis (Salmonella) L. Haemorrhagic gastritis M. Pyloric stenosis N. Achalasia
Gastroenteritis- staph
1181
How to classify causes of splenomegaly?
Size: Massive, moderate, mild
1182
Causes of massive splenomegaly
CML, myelofibrosis (UK) Malaria, Kala-Azar (worldwide)
1183
Causes of moderate splenomegaly
Portal HTN Lymphoma CLL Thalassaemia Metabolic diseases e.g. Gaucher's
1184
Causes of mild splenomegaly
Infection e.g. EBV, hepatitis, bacterial CTD e.g. RA, PAN, SLE, Felty's Infiltrative disorders: amyloidosis, sarcoidosis
1185
AA amyloidosis
Secondary to inflammatory conditions e.g. RA, Crohn's etc
1186
Pancreatic pseudocysts usually develop
Post surgery
1187
Intestinal metaplasia
Intestinal metaplasia describes the transformation of an epithelium to the columnar epithelium found in the intestines. The distinguishing feature of intestinal epithelium is the presence of goblet cells. (NB. the stomach epithelium is also columnar, but it doesn't contain the distinctive goblet cells. It contains foveolar cells which form glands and secrete mucin) Intestinal metaplasia is a response to chronic inflammation. It is commonly associated with chronic gastritis - H. pylori-induced or autoimmune. The presence of intestinal metaplasia is a strong risk-factor for the development of adenocarcinoma.
1188
Barret's oesophagus
Barrett's oesophagus is another example of intestinal metaplasia due to chronic inflammation. This time chronic GORD is the cause. Again, the intestinal metaplasia is strongly associated with the development of adenocarcinoma. GORD and Barrett's is the reason that oesophageal adenocarinoma tends to affect the distal third of the oesophagus while squamus cell oesophageal cancer affects the upper two thirds.
1189
Which HLA alleles are assocaited with Coeliac disease?
DQ2 most strongly DQ8
1190
What differentiates between acute and chronic gastritis
Chronic gastritis is usually caused by H. pylori which would result in an antral gastritis. This patient had erosions throughout the gastric mucosa. Chronic gastritis would be have lymphocytes and plasma cells on biopsy - this patient had a neutrophilic infiltrate which is typical of an acute gastritis. Focal, acute mucosal defects are a well-known complication of NSAIDs which this patient was probably taking.
1191
Oesophageal carcinoma tissue type lower
Adenocarcinoma
1192
Oesphageal carcinoma tissue type middle
Squamous cell carcinoma
1193
A 20-year-old student gives an 8 hour history of very frequent vomiting and epigastric cramping. O/E she is pale and shivering. Her serum WBC is normal. Why is the WBC normal?
Staphylococcus aureus is a leading cause of gastroenteritis resulting from the consumption of contaminated food. Staphylococcal food poisoning is due to the absorption of staphylococcal enterotoxins preformed in the food. The onset of symptoms is rapid (from 30 min to 8 h) and usually spontaneous remission is observed after 24 h. As the presentation is so acute, the WCC often have not had a chance to increase (lag-time). The symptoms of staphylococcal food poisoning are abdominal cramps, nausea, vomiting, sometimes followed by diarrhea (never diarrhea alone).
1194
A 26 year old gentleman presents to A and E with acute abdominal pain. He has lost 5 kg of weight in the last 2 weeks, and has been passing bloody diarrhoea with mucus. He is pyrexial and on examination is noted to have angular stomatitis. A. Diverticulitis B. Diverticular disease C. Angiodysplasia D. Crohn's disease E. Intestinal TB F. Ulcerative colitis G. Gastroenteritis H. Colorectal carcinoma I. Pseudomembranous colitis J. Tubular adenoma K. Ischaemic colitis L. Sigmoid volvulus
Crohn's disease
1195
A 62 year old housewife returns to your outpatient clinic following another incidence of the passing of blood. Previous sigmoidoscopy, DRE and barium enema’s have failed to identify any lesion and she denies weight loss and diarrhoea. However, blood tests show a microcytic anaemia. A. Appendicitis B. Anorectal abscess C. Angiodysplasia D. Anterior rectocoele E. Diverticular disease F. Adenocarcinoma Duke's stage B G. Diverticulitis H. Perianal haematoma I. Benign adenoma J. Haemorrhoids K. Villous adenoma L. Crohn's disease M. Ischaemic colitis N. Irritable bowel syndrome O. Adenocarcinoma Duke's stage C1 P. Ulcerative colitis
olonic angiodysplasia is a common cause of acute or chronic rectal bleeding and iron deficiency anaemia. Angiodysplasias are tiny - 1-5 mm in diameter - hamartomatous capillary lesions in the colonic wall which produce bleeding out of proportion to their size. They are believed to be acquired, possibly as a result of tension on the veins where they pass through the muscularis. Diagnosis: subtraction mesenteric arteriography may demonstrate bleeding if rapid colonscopy: may visualise lesion Treatment: electrical coagulation via the colonoscope resection of segment of colon if the above is unsuccessful
1196
A 73 year old woman attends complaining of recent onset of tiredness. She is pale and has hepatosplenomegaly and generalised lymphadenopathy in the neck, axillae and groins. A. Sarcoidosis B. Severe emphysema C. Chronic lymphocytic leukaemia D. Cirrhosis with hepatoma E. Hameochromatosis F. Liver abscess G. Gaucher's disease H. Portal vein thrombosis I. Congestive heart failure J. Polycythaemia rubra vera K. Chronic myeloid leukaemia
Chronic lymphocytic leukaemia
1197
A 45 year old male was admitted to hospital after having a fall. Investigations reveal a leucocytosis with elevated bilirubin and transferases. Levels were albumin, folate and vitamin B12 were low. Clotting studies showed a prolonged prothrombin time. . Hereditary haemochromatosis B. Alcoholic liver disease C. Hepatitis A D. Hepatitis B E. Hepatocellular carcinoma F. Wilson's disease G. Autoimmune hepatitis H. Primary biliary cirrhosis
Alcoholic liver disease
1198
A 35 year old patient with known Hepatitis B visits the doctor complaining of recent weight loss, loss of appetite, fevers and an ache in the right hypochondrium. On examination an enlarged, irregular, tender liver can be palpated. A. Primary sclerosing cholangitis B. Alpha1 antitrypsin deficiency C. Wilson's disease D. Hereditary haemochromatosis E. Hepatocellular carcinoma F. Hepatitis B G. Primary biliary cirrhosis H. Ascending cholangitis I. Budd-Chiari syndrome J. Hepatitis C K. Alcoholic hepatitis
Hepatocellular carcinoma
1199
A 65 year old retired factory worker presents with dysuria and frequency. He has also noticed that his urine is red. PSA is normal. A. Renal cell carcinoma B. Pyelonephritis C. Cystitis D. Urothelial carcinoma E. Adult PCKD F. Bladder cancer G. Prostatitis H. Prostate cancer
Bladder cancer
1200
54 year old man presents to A&E with fever and weakness. Blood tests show hypercalcaemia . On eaxamination a palpable mass is found in the right loin area A. Renal cell carcinoma B. Pyelonephritis C. Cystitis D. Urothelial carcinoma E. Adult PCKD F. Bladder cancer G. Prostatitis H. Prostate cancer
. Renal cell carcinoma
1201
A 30 year old woman presents with lower abdominal pain accompanied by fever. She has been using an intra uterine contraceptive device since her wedding three years ago. A. Herpes virus infection B. Endometriosis C. Fibroids D. Endometritis E. Ectopic pregnancy F. Cervical polyps G. Arias-Stella phenomenon H. Polycystic ovary syndrome I. Candidiasis J. Salpingitis
Endometritis
1202
A 30 year old woman attends a fertility clinic as her and her husband are unable to conceive. For the last few years her periods have been fairly heavy and prolonged. She has stopped wearing skirts because she thinks her ankles look fat. A. Herpes virus infection B. Endometriosis C. Fibroids D. Endometritis E. Ectopic pregnancy F. Cervical polyps G. Arias-Stella phenomenon H. Polycystic ovary syndrome I. Candidiasis J. Salpingitis
Fibroids
1203
What is Charcot's triad
Fever RUQ pain Jaundice with rigors etc Bacterial cholangitis
1204
Features of PSC
Primary sclerosing cholangitis is a rare disease of unknown aetiology characterised by chronic inflammation and fibrosis of the bile duct. - usually men - associated with IBD, particularly ulcerative colitis - may be complicated by strictures, cholangitis, cholangiocarcinoma - autoantibodies are less frequent than in autoimmune chronic active hepatitis and primary biliary cirrhosis: in an exam, ANCA positivity might be a clue as P-ANCA positive in more than 50% of cases. Antimitochondrial antibodies almost never occur. - Liver Bx: fibrous obliterating cholangitis loss of interlobular and adjacent septal bile ducts - ERCP shows multiple annular strictures, separated by round or slightly dilated duct segments and the intrahepatic and extrahepatic bile ducts have a beaded appearance
1205
fibrous obliterating cholangitis loss of interlobular and adjacent septal bile ducts - ERCP shows multiple annular strictures, separated by round or slightly dilated duct segments and the intrahepatic and extrahepatic bile ducts have a beaded appearance
PSC
1206
Features of PBC
Primary biliary cirrhosis is a chronic, progressive, cholestatic liver disease which classically affects middle aged women (rare under 30 yrs). 95% of patients have anti-mitochondrial autoantibodies, especially E2 component of the pyruvate dehydrogenase complex. Pruritus and fatigue common, but also remember xanthelasma, hyperlipidaemia, feats of chronic liver disease.
1207
A 40 year old woman has always known cramping pain associated with her periods – which have usually been heavy. Recently this pain has become constant throughout the month, and her periods have become more frequent. She claims never to have used oral contraception and has no children. She is abstaining from sexual intercourse as it is too painful.
Although you may classically think of the pain of endometriosis as cyclical - responding to the hormonal influence of menstruation, the history is very variable and may be progressively worse to even continuous pelvic pain. Constant pain is often due to adhesions (which this patient may have now developed). In the history - nulliparity, due to subfertility secondary to the endometriosis itself or abstinence due to deep dyspareunia is common. Rare, but possible...are symptoms such as cyclical haematuria or haemopytsis....(which is important to keep in your differential diagnosis, albeit at the bottom of your list...) or painful, cyclical, expanding masses in a pelvic scar.
1208
This type of bone is 80-90% calcified and its function is mainly mechanical and protective.
Cortical
1209
This type of bone is immature and usually pathological. A. Lamellar B. Woven C. Osteoblast D. Osteoclast E. Metaphysis F. Osteocyte G. Periosteum H. Diaphysis I. Cortical J. Endosteum K. Epiphysis L. Cancellous
Woven
1210
A 15-year-old male presents with a 2-month history of increasing pain in his right upper arm. Bone biopsy reveals sheets of cells with small, primitive nuclei and scanty cytoplasm. A positive immunoreactivity is seen with the MIC2 (CD99) antibody. A. Primary osteosarcoma B. Osteogenesis imperfecta C. Osteopetrosis D. Osteomalacia E. Osteomyelitis F. Osteoporosis G. Osteoarthritis H. Chondrosarcoma I. Ewing's sarcoma
Ewing's sarcoma
1211
A 20-year-old male complains of a progressively enlarging painful mass on his right upper arm. Radiology demonstrates a lytic lesion of the proximal humerus with an accompanying Codman triangle. Microscopically, pleomorphic mesenchymal cells producing dark-staining osteoid are seen. A. Primary osteosarcoma B. Osteogenesis imperfecta C. Osteopetrosis D. Osteomalacia E. Osteomyelitis F. Osteoporosis G. Osteoarthritis H. Chondrosarcoma I. Ewing's sarcoma
Primary osteosarcoma
1212
A 24 year old police woman attends the clinic as her GP suspects she may have a parathyroid tumour. She has raised PTH and serum calcium. After a 24hr urinary collection it is noted the patient has a low urine calcium output A. Osteitis fibrosa B. Secondary hyperparathyroidism with chronic renal osteodystrophy C. Paget's disease D. Cushing's syndrome E. Cushing's disease F. Familial hypocalcuric hypercalcaemia G. Primary hyperparathyroidism H. Tertiary hyperparathyroidism I. Osteomalacia J. Osteoporosis K. Primary hypthyroidism (myxoedema)
Familial hypocalcuric hypercalcaemia
1213
ou see a new patient for the first time at the surgery, a 14 year old boy, complaining of pain in his right leg for 9 months and contracture of the right knee which has developed over this period. The child looks stunted and you can see bowing of the lower extremities in ambulators. His notes have not arrived yet from his previous doctor but his mother tells you ‘he was born 3 weeks premature with two small kidneys and three failed transplants means he has dialysis four times a day’. His blood results later show a raised PTH and phostate, low calcium and 1,25(OH)vitamin D. He is also acidotic. A. Osteitis fibrosa B. Secondary hyperparathyroidism with chronic renal osteodystrophy C. Paget's disease D. Cushing's syndrome E. Cushing's disease F. Familial hypocalcuric hypercalcaemia G. Primary hyperparathyroidism H. Tertiary hyperparathyroidism I. Osteomalacia J. Osteoporosis K. Primary hypthyroidism (myxoedema)
Secondary hyperparathyroidism with chronic renal osteodystrophy
1214
An 18 year old student presents to his GP with focal pain in his left fore-arm which is tender to touch and worsens at night. The pain is relieved with aspirin. An X-ray shows a 1cm are of radio-lucency in the tibia surrounded by dense bone. A. Osteoclastoma B. Osteosarcoma C. Osteochondroma D. Chondrosarcoma E. Ewing's tumour F. Osteoid osteoma G. Osteoporosis H. Simple bone cyst I. Osteitis J. Trauma K. Rheumatoid arthritis L. Osteoarthritis M. Metastases N. Fibrous dysplasia O. Echondroma
Osteoid osteoma
1215
A 14 year old boy complains to you of a painless lump on his left thigh, just above the knee which is slowly growing. His past medical history reveals that he fractured his femur in the same location several years before. A. Osteoclastoma B. Osteosarcoma C. Osteochondroma D. Chondrosarcoma E. Ewing's tumour F. Osteoid osteoma G. Osteoporosis H. Simple bone cyst I. Osteitis J. Trauma K. Rheumatoid arthritis L. Osteoarthritis M. Metastases N. Fibrous dysplasia O. Echondroma
Osteochondroma
1216
A 15 year old girl shows you a small lump on her upper arm on routine examination. She says the lump has been present for a couple of years and has slowly moved down, away from her shoulder. A. Osteoclastoma B. Osteosarcoma C. Osteochondroma D. Chondrosarcoma E. Ewing's tumour F. Osteoid osteoma G. Osteoporosis H. Simple bone cyst I. Osteitis J. Trauma K. Rheumatoid arthritis L. Osteoarthritis M. Metastases N. Fibrous dysplasia O. Echondroma
Simple bone cyst
1217
A 50 year old lady presents with pain in her jaw. She suffers from Paget’s disease. A ‘sunburst’ appearance is seen on X-ray along with a lifted periosteum (Codman’s triangle). A. Osteoclastoma B. Osteosarcoma C. Osteochondroma D. Chondrosarcoma E. Ewing's tumour F. Osteoid osteoma G. Osteoporosis H. Simple bone cyst I. Osteitis J. Trauma K. Rheumatoid arthritis L. Osteoarthritis M. Metastases N. Fibrous dysplasia O. Echondroma
Osteosarcoma
1218
An 8 year old boy is brought to his GP by his parents with pain in his hips and a fever. Blood results demonstrate a raised ESR and biopsy histology shows droplets of glycogen in the cytoplasm of small round cells in the pelvic bones. A. Osteoclastoma B. Osteosarcoma C. Osteochondroma D. Chondrosarcoma E. Ewing's tumour F. Osteoid osteoma G. Osteoporosis H. Simple bone cyst I. Osteitis J. Trauma K. Rheumatoid arthritis L. Osteoarthritis M. Metastases N. Fibrous dysplasia O. Echondroma
Ewing's tumour
1219
70 year old man presents to his GP with a four day history of haemoptysis. He has noticed he has been loosing weight over the last 4 months and has felt tired and unwell. On examination he has bilateral ptosis and proximal weakness in the limbs which improves on repeated testing A. Cystic fibrosis B. Acute asthma C. Cryptogenic fibrosing alveolitis D. Tuberculosis E. Emphysema F. Pulmonary oedema G. Pulmonary embolism H. Squamous cell carcinoma I. Pneumococcal pneumonia J. Small cell carcinoma K. Thymoma L. Chronic bronchitis M. Mycoplasma pneumonia
Small cell carcinoma
1220
A 35 year old woman presents with weight loss and tiredness. Her GP examines her and finds that she also has a fine tremor and is sweaty. Investigations: TSH \<0.01, Free T4 36.0. There is low uptake on a technetium scan. The aetiology could be viral or autoimmune. A. Grave's disease B. De Quervain's C. Hashimoto's thyroiditis D. Euthyroid state E. Medullary carcinoma F. Thyroid storm G. Subacute thyroiditis H. Postpartum thyroiditis I. Papillary carcinoma
Subacute thyroiditis
1221
An 18 year old man notices a lump on his neck and goes to his GP. As well as the lump, the GP discovers cervical lymphadenopathy. There is no family history of any endocrine disorder, nor is he suffering from any other illness. Thyroglobulin is 140. A. Grave's disease B. De Quervain's C. Hashimoto's thyroiditis D. Euthyroid state E. Medullary carcinoma F. Thyroid storm G. Subacute thyroiditis H. Postpartum thyroiditis I. Papillary carcinoma
Papillary carcinoma
1222
eoplasms found in women aged between 30 and 40 as ovarian masses, usually unilateral. They are usually benign (90%) and are often the largest ovarian neoplasm. A. Cervical Squamous Cell Carcinoma B. Cervical Intraepithelial Neoplasia (CIN) C. Endometrial Adenocarcinoma D. Endometrial Polyps E. Vaginal Squamous Cell Carcinoma F. Mucinous Tumour G. Serous Tumour H. Teratoma (Mature) I. Endometrioid Tumour J. Teratoma (Immature)
Mucinous ovarian neoplasms account for 10-15% of all ovarian tumours and are rarely malignant (~10-15%). They are rarely bilateral, but can be very large. In contrast, whilst serous tumours are also usually benign (30% malignant) and common - accounting for 30% of all ovarian tumours - they have a strong tendency to be bilateral.
1223
Squamous epithelium mixed with intestinal epithelium A. Serous cystadenocarcinoma B. Mature cystic teratoma C. Thecoma D. Dysgerminoma E. Fibroma F. Krukenberg tumour G. Mucinous cystadenocarcinoma H. Clear cell tumour I. Sertoli-Leydig tumour J. Immature teratoma K. Granulosa cell tumours
Mature cystic teratoma
1224
Fibrous tissue containing spindle cells and lipid A. Serous cystadenocarcinoma B. Mature cystic teratoma C. Thecoma D. Dysgerminoma E. Fibroma F. Krukenberg tumour G. Mucinous cystadenocarcinoma H. Clear cell tumour I. Sertoli-Leydig tumour J. Immature teratoma K. Granulosa cell tumours
.Thecoma Feedback: In a fibroma, there are intersecting bundles of spindle cells. Thecomas in addition contain lipid.
1225
Malignant cells surrounded by serous fluid and Psammoma bodies A. Serous cystadenocarcinoma B. Mature cystic teratoma C. Thecoma D. Dysgerminoma E. Fibroma F. Krukenberg tumour G. Mucinous cystadenocarcinoma H. Clear cell tumour I. Sertoli-Leydig tumour J. Immature teratoma K. Granulosa cell tumours
Serous cystadenocarcinoma
1226
Germ cells mixed with lymphocytes A. Serous cystadenocarcinoma B. Mature cystic teratoma C. Thecoma D. Dysgerminoma E. Fibroma F. Krukenberg tumour G. Mucinous cystadenocarcinoma H. Clear cell tumour I. Sertoli-Leydig tumour J. Immature teratoma K. Granulosa cell tumours
Dysgerminoma
1227
Seminomatous germ cell tumours are
radioSensitive, present in the 30s ; 15% secrete HCG ; 0% secrete AFP.
1228
radioSensitive, present in the 30s ; 15% secrete HCG ; 0% secrete AFP.
Seminomatous germ cell tumours
1229
Nonseminatous GCTs are
present in the 20s, and may secrete AFP and/or HCG.
1230
Germ cell tumour originating in testis that is radiosensitive and classically presents in the 4th decade. A. Diffuse large B cell lymphoma (DLBCL) B. Seminoma C. Embryonal carcinoma D. Acute lymphoblastic leukaemia/lymphoma (ALL) E. Choriocarcinoma F. Teratoma G. Leydig cell tumour H. Acute myeloid leukaemia (AML) I. Sertoli cell tumour J. Yolk sac (endodermal sinus) tumour
Seminoma
1231
Very aggressive tumour producing HCG and AFP; neoplastic cells are anaplastic. A. Diffuse large B cell lymphoma (DLBCL) B. Seminoma C. Embryonal carcinoma D. Acute lymphoblastic leukaemia/lymphoma (ALL) E. Choriocarcinoma F. Teratoma G. Leydig cell tumour H. Acute myeloid leukaemia (AML) I. Sertoli cell tumour J. Yolk sac (endodermal sinus) tumour
Embryonal carcinoma
1232
Very aggressive HCG-producing tumour composed of cytotrophoblast and syncytiotrophoblast cells that metastasizes early. A. Diffuse large B cell lymphoma (DLBCL) B. Seminoma C. Embryonal carcinoma D. Acute lymphoblastic leukaemia/lymphoma (ALL) E. Choriocarcinoma F. Teratoma G. Leydig cell tumour H. Acute myeloid leukaemia (AML) I. Sertoli cell tumour J. Yolk sac (endodermal sinus) tumour
Choriocarcinoma
1233
Commonest malignant cause of testicular mass in those aged under 5.. A. Diffuse large B cell lymphoma (DLBCL) B. Seminoma C. Embryonal carcinoma D. Acute lymphoblastic leukaemia/lymphoma (ALL) E. Choriocarcinoma F. Teratoma G. Leydig cell tumour H. Acute myeloid leukaemia (AML) I. Sertoli cell tumour J. Yolk sac (endodermal sinus) tumour
Acute lymphoblastic leukaemia/lymphoma (ALL)
1234
Commonest malignant cause of testicular mass in those aged 60. A. Diffuse large B cell lymphoma (DLBCL) B. Seminoma C. Embryonal carcinoma D. Acute lymphoblastic leukaemia/lymphoma (ALL) E. Choriocarcinoma F. Teratoma G. Leydig cell tumour H. Acute myeloid leukaemia (AML) I. Sertoli cell tumour J. Yolk sac (endodermal sinus) tumour
Diffuse large B cell lymphoma (DLBCL)
1235
A 22-year-old lady presented with a vaginal discharge. Gram staining revealed “Clue cells” surrounded by rods, that were “Gram variable”. A. Neisseria gonorrheae B. Trichomonas vaginalis C. Herpes zoster D. Cervical Intraepithelial Neoplasia I E. Herpes simplex F. Cervical Intraepithelial Neoplasia III G. Cervical Intraepithelial Neoplasia II H. Granuloma Inguinale I. Candida albicans J. Group B Streptococcus K. Chlamydia L. Lymphogranuloma venereum M. Treponema pallidum (syphilis) N. Cervical polyps O. Gardnerella vaginalis P. Cervical Microglandular Hyperplasia
Gardnerella vaginalis
1236
A pap smear taken from a chronic granulomatous ulcer shows a necrotic centre, periarteritis and endarteritis obliterans and an intense peripheral cellular infiltrate consisting mainly of mononuclear cells and giant cells. A. Neisseria gonorrheae B. Trichomonas vaginalis C. Herpes zoster D. Cervical Intraepithelial Neoplasia I E. Herpes simplex F. Cervical Intraepithelial Neoplasia III G. Cervical Intraepithelial Neoplasia II H. Granuloma Inguinale I. Candida albicans J. Group B Streptococcus K. Chlamydia L. Lymphogranuloma venereum M. Treponema pallidum (syphilis) N. Cervical polyps O. Gardnerella vaginalis P. Cervical Microglandular Hyperplasia
Treponema pallidum (syphilis)
1237
A young lady is found to have chronic irritation and inflammation of the vulva. A pap smear and use of a silver stain reveals fungi within the keratin layer and superficial epithelium. A. Neisseria gonorrheae B. Trichomonas vaginalis C. Herpes zoster D. Cervical Intraepithelial Neoplasia I E. Herpes simplex F. Cervical Intraepithelial Neoplasia III G. Cervical Intraepithelial Neoplasia II H. Granuloma Inguinale I. Candida albicans J. Group B Streptococcus K. Chlamydia L. Lymphogranuloma venereum M. Treponema pallidum (syphilis) N. Cervical polyps O. Gardnerella vaginalis P. Cervical Microglandular Hyperplasia
Candida albicans
1238
The metaplasia that occurs in the transformation zone involves which cell-types?
Glandular to squamous epithelium
1239
A 27 year old woman undergoes a routine Pap test. Histology shows increased squamous epithelium with atypical cells showing koilocytosis (nuclear enlargement with perinuclear halo = clear area around nucleus). Follow-up colposcopy shows hyperchromatic nuclei present in the lower 1/3 of the epithelial layer from the basement membrane. A. Invasive cervical cancer B. CIN I C. Squamous to glandular epitheliem D. 50-60 years E. CIN III F. 45 years G. Glandular to squamous epithelium H. Ovarian cancer I. 65 years J. 30 - 40 years K. CIN II
CIN I
1240
22 year old lady presented for her routine smear test. The test showed a slight increase in DNA staining, and also a slightly larger variation in size of nuclei. Further questioning revealed she was a single mother living with her boyfriend, and had had multiple sexual partners since her first sexual contact at age 16. A. Group B Streptococcus B. Cervical Microglandular Hyperplasia C. Lymphogranuloma venereum D. Cervical Intraepithelial Neoplasia III E. Trichomonas vaginalis F. Neisseria gonorrheae G. Cervical Intraepithelial Neoplasia I H. Mild dyskaryosis I. Herpes simplex J. Granuloma Inguinale K. Gardnerella vaginalis L. Treponema pallidum (syphilis) M. Severe dyskaryosis N. Chlamydia O. Candida albicans
Mild dyskaryosis
1241
Single most useful first-line cytological investigation for the confirmation of the benign status of an ovarian cyst. A. Exfoliative (brush) cytology B. Cystoscopy C. Cone biopsy D. Endometrial tissue sampling E. Percutaneous FNA biopsy F. Stereotactic radiographic cytological sampling method G. Fine needle aspirate H. Core biopsy I. Fluid cytology of alveolar washings
Core biopsies give HISTOLOGY Fine needle aspirates give CYTOLOGY - cannot assess basement membrane penetration (ie invasion) FNA
1242
Has a 100% positive predictive value for malignant cytopathological diagnosis.
FNA
1243
What is the importance of measuriny urinary calcium in ?hyperPTHism
Familial hypocalciuric hypercalcaemia (FHH) is a dominantly inherited condition caused by a mutation of the calcium sensing receptor. The result is that the kidney reabsorbes calcium very avidly as a primary problem. This results in hypocalciuria and hypercalcaemia. Because the parathyroid gland calcium receptor also does not work, the parathyroids continue to secrete excess PTH despite the high calcium, so it looks just like primary hyperparathyroidism. However removing the parathyroids does not help the calcium, as the primary cause of trouble is in the kidney. In true primary hyperparathyroidism however, the calcium that is filtered hugely exceeds the tubular maximum for calcium, so there is no chance of reabsorption, so the urinary calcium is high, with resulting renal stones (which NEVER happens in FHH). There are many surgeons who have operated wrongly on FHH patients, removing the parathyroid only to find the calcium does not get better. As a consequence, we have written guidelines to surgeons, reminding them of the importance of measuring urine calcium in such a patient before removing the parathyroid gland.
1244
70 year old man presents to his GP with a four day history of haemoptysis. He has noticed he has been loosing weight over the last 4 months and has felt tired and unwell. On examination he has bilateral ptosis and proximal weakness in the limbs which improves on repeated testing — Correct answer: Small cell carcinoma why is this Small Cell Carcinoma?
This guy has Myasthenic Syndrome Eaton-Lambert syndrome which occurs in ~3% of people with small cell l ung cancer. In contrast with myasthenia gravis, the fatigue gets BETTER with repeated use of the muscle.
1245
What differentiates between autoimmune and De Quervain's thyroiditis
Autoimmune tend to be painless
1246
endarteritis obliterans in the context of STIs=
Treponema pallidum
1247
An 86 year old man is admitted to A&E having collapsed at his home. He is unconscious and a couple of days later he is still deeply unconscious. His pupils are pin-point and their reaction to light is difficult to see clearly A. Vascular dementia B. Cerebellar stroke C. Opiate overdose D. Transient Ischaemic attack E. Cerebral embolus F. Subarachnoid haemorrhage G. Panic attack H. Pontine haemorrhage I. Subdural haemorrhage J. Temporal arteritis K. Transient hypotension L. Brain stem infarction M. Extradural haemorrhage N. Raised intracranial pressure O. Alzheimer’s disease P. Meningitis
Brain stem infarction
1248
Occlusion of this cerebral vessel can cause weakness and numbness in the contralateral lower limb and similar but milder symptoms in the contralateral upper limb. A. Anterior cerebral artery B. Lacunar infarcts C. Middle cerebral artery D. Berry's aneurysm E. Vertebrobasilar circulation F. Extradural haemorrhage G. Bacterial meningitis H. Hydrocephalus I. Amyloidosis J. Viral meningitis K. Subarachnoid haemorrhage L. Posterior cerebral artery M. Stroke N. Transient ischaemic attack (TIA)
ACA
1249
Transtentorial herniations can potentially compromise the sufficiency this particular part of the cerebral circulation and cause occipital lobe infarction. A. Anterior cerebral artery B. Lacunar infarcts C. Middle cerebral artery D. Berry's aneurysm E. Vertebrobasilar circulation F. Extradural haemorrhage G. Bacterial meningitis H. Hydrocephalus I. Amyloidosis J. Viral meningitis K. Subarachnoid haemorrhage L. Posterior cerebral artery M. Stroke N. Transient ischaemic attack (TIA)
PCA
1250
5 year old obese woman goes to GP about worsening headaches. They are especially bad in the mornings and she also feels nauseous with some visual disturbances. A. Subdural haemorrhage B. Trigeminal neuralgia C. Glaucoma D. Migraine E. Berry aneurysm F. Subarachnoid haemorrhage G. Tension headache H. Sinusitis I. Benign intracranial hypertension
BIH
1251
An elderly diabetic lady with an acute headache and associated blurred vision and vomiting. Pain is localised to the upper anterior region of her head. A. Subdural haemorrhage B. Trigeminal neuralgia C. Glaucoma D. Migraine E. Berry aneurysm F. Subarachnoid haemorrhage G. Tension headache H. Sinusitis I. Benign intracranial hypertension
Glaucoma
1252
A 60 year old lady presents with a hard palpable mass in the right breast. The mammography shows that the lump is calcified. FNA results show that the cells in the lump have pleomorphic nuclei and that the lump has a necrotic centre A. Invasive Carcinoma B. Gyaecomastia C. Lobular Carcinoma in situ D. Pagets Disease of the nipple E. Duct Papilloma F. Nipple adenoma G. Lymphoma H. Fibroadenoma I. Ductal Carcinoma in situ
Ductal Carcinoma in situ
1253
A 23 year old lady comes to the breast clinic because she is worried of having breast cancer as her mother and grandmother both have had the disease. The mammogram finds no abnormality but you think you feel a number of multifocal lumps A. Invasive Carcinoma B. Gyaecomastia C. Lobular Carcinoma in situ D. Pagets Disease of the nipple E. Duct Papilloma F. Nipple adenoma G. Lymphoma H. Fibroadenoma I. Ductal Carcinoma in situ
Lobular Carcinoma in situ
1254
A 45 year old lady is referred to the breast clinic with nipple retraction A. Invasive Carcinoma B. Gyaecomastia C. Lobular Carcinoma in situ D. Pagets Disease of the nipple E. Duct Papilloma F. Nipple adenoma G. Lymphoma H. Fibroadenoma I. Ductal Carcinoma in situ
Invasive Carcinoma
1255
A 30 year old woman is referred to the breast clinic with a red, roughened and ulcerated nipple. A. Invasive Carcinoma B. Gyaecomastia C. Lobular Carcinoma in situ D. Pagets Disease of the nipple E. Duct Papilloma F. Nipple adenoma G. Lymphoma H. Fibroadenoma I. Ductal Carcinoma in situ
Pagets Disease of the nipple
1256
A 56-year old female presents with a palpable mass in her left breast. Radiology shows microcalcification. A. Fine needle aspirate B. BRCA1 gene C. Hormone replacement therapy D. Paget's disease of the nipple E. Carcinoma-in-situ (lobular) F. Nulliparity G. Axillary lymph node metastases H. Fibroadenoma I. Carcinoma-in-situ (ductal) J. Invasive breast carcinoma K. Oestrogen-receptor positivity
Fibroadenoma
1257
involves focal calcification of the media of small medium-sized arteries. It usually presents in patients over 50 years of age and unlike atherosclerosis, there is no associated inflammation in the pathogenesis.
Monckeberg arteriosclerosis
1258
What differentiates dermatomyositis from MG
Sparing of the ocular muscles
1259
45-year-old man is referred to the gastroenterology outpatient clinic due to severe epigastric pain and an episode of haematemesis. Further testing reveals he is Helicobacter pylori positive and has a 20 pack–year history of smoking.
Peptic ulcer disease (I) can either be duodenal or gastric. Ulcers differ from erosions as they extend to the submucosa (sometimes to the muscularis mucosa) and take weeks to heal, whereas erosions breach the mucosa only and take days to heal. The main causes of peptic ulcers are Helicobacter pylori, NSAIDs, Zollinger–Ellison syndrome and smoking. These factors disrupt the balance between protective (mucus layer and bicarbonate secretion) and damaging (acid and enzymes) elements leading to ulceration.
1260
A 56-year-old woman is investigated by the hepatology team for decompensated liver disease. A liver biopsy sample stains blue with Perl’s Prussian blue stain. A Cholangiocarcinoma B Cirrhosis C α1-Antitrypsin deficiency D Haemosiderosis E Primary biliary cirrhosis F Haemochromatosis G Hepatocellular carcinoma H Primary sclerosing cholangitis I Wilson’s disease
Haemochromatosis (F) is an autosomal recessive condition that is due to a mutation in the HFE gene. The HFE protein regulates iron absorption that is stored as haemosiderin. Histological features of haemochromatosis include a golden-brown haemosiderin deposition in the parenchyma of many organs. Haemosiderin eventually leads to inflammation and subsequent fibrosis. Histological samples of affected tissue will stain blue with Perl’s Prussian blue. Organs affected include the liver (cirrhosis), pancreas (diabetes), skin (bronzed pigmentation), heart (cardiomyopathy) and gonads (atrophy and impotence).
1261
With what is PBC strongly associated?
Sjogren's
1262
Histological features include periductal fibrosis that eventually invades the lumen causing concentric onion-ring fibrosis.
PSC
1263
A 54-year-old man is referred to the dermatologist with a brown warty lesion on his nose which has a rough consistency. Biopsy of the lesion reveals solar elastosis. A Pemphigoid B Bowen’s disease C Pityriasis rosea D Lichen planus E Actinic keratosis F Psoriasis G Basal cell carcinoma H Erythema multiforme I Malignant melanoma J Pemphigus
Actinic keratosis
1264
begins as a single scaly macule that is salmon-pink, \<5 cm and known as a ‘herald patch’. After several days multiple small pink rashes appear posteriorly creating a fir-tree pattern.
Pityriasis rosea
1265
involves the following: haematuria, red cell and white cell casts, dysmorphic red cells, oliguria and hypertension.
Nephritic syndrome
1266
form as a result of Tamm–Horsefall secretions in the distal collecting duct and collecting duct that ‘glue’ cells together,
Cellular casts
1267
it mimics breast cancer; the presentation may be nipple retraction due to fibrosis and bloody discharge secondary to rupture of the ducts.
Duct ectasia
1268
usually occurs mainly in pre-menopausal women and is bilateral and multifocal. No calcification occurs and hence there is no detection on mammogram. Histologically there is no necrosis and uniform nuclei are present.
Lobular carcinoma in situ
1269
occur in pre- or post-menopausal women. They are usually unilateral and unifocal. On mammogram, microcalcification may be visible secondary to central necrosis.
DCIS
1270
Histological investigation reveals an ‘artichoke-like’ appearance as the stroma pushes up on the epithelium to form clubs.
Phylloides tumour
1271
has a scirrhous look whereby the centre is very fibrous giving a dense white appearance. has the worst prognosis compared to all other invasive carcinomas (medullary, mucinous, tubular and papillary carcinoma).
Infiltrating ductal carcinoma
1272
A 35-year-old man with pain and difficulty bending his left knee. X-ray reveals many lytic lesions in the epiphysis of the patient’s left knee. A Osteoporosis B Fibrous dysplasia C Paget’s disease D Osteomalacia E Osteochondroma F Osteoid osteoma G Renal osteodytrophy H Enchondroma I Giant cell tumour
Giant cell tumour (GCT; I) is a borderline malignant tumour of giant osteoclast cells. The cells are similar to those found in Paget’s disease as they have multiple nuclei (\>20). The osteoclastic cells cause lytic lesions in the epiphyses (especially around the knee) that are visible on X-ray and may give a characteristic ‘soap bubble’ appearance. Histological features include multinucleated giant osteoclasts with surrounding ovoid and spindle cells.
1273
Histological features include multinucleated giant osteoclasts with surrounding ovoid and spindle cells. visible on X-ray and may give a characteristic ‘soap bubble’ appearance
Giant cell tumour
1274
An 8-year-old boy has been diagnosed with precocious puberty. A routine examination by the paediatrician reveals café-au-lait spots on the child’s back. The boy has had numerous fractures of his femur and tibia bilaterally after falls. A Osteoporosis B Fibrous dysplasia C Paget’s disease D Osteomalacia E Osteochondroma F Osteoid osteoma G Renal osteodytrophy H Enchondroma I Giant cell tumour
Fibrous dysplasia (B) occurs due to the developmental arrest of normal bone structures secondary to an osteoblast maturation defect. The most common sites affected are the proximal femur and ribs. On X-ray, fibrous dysplasia may cause a ground-glass or soap bubble appearance. Histological investigation reveals trabeculae that lack osteoblastic rimming. Two possible syndromes can arise: 1) Mono-ostotic (70 per cent) affecting femurs more than ribs occurring in patients under 30 years of age, and 2) McCune–Albright syndrome (30 per cent) that is poly-ostotic and causes café-au-lait spots and precocious puberty.
1275
A 57-year-old overweight patient suffers an acute myocardial infarction and subsequently dies. A post-morterm examination of the infarcted area shows extensive cell infiltration including polymorphs and macrophages. There is also extensive debris post necrosis and the cytoplasm is homogeneous making it difficult to see the outlines of the myocardial fibres. There is no evidence of collagenization or a scar. How long after the initial attack did the patient die? A At the time of the attack (0–6 hours) B Hours after the attack (6–24 hours) C Days after the attack (1–4 days) D Within the first 2 weeks of the attack (4–14 days) E Weeks and months after (14 days +)
C In the first 6 hours (A) in the evolution of an acute myocardial infarction (MI), the histology is normal. Necrotic cell death takes place between 6 and 24 hours (B). Pathologically, there is contraction band necrosis (dark red/pink wavy lines extending across the myocardial fibres), loss of nuclei in the myocardial cells and the start of a homogeneous appearing cytoplasm. At 1–4 days (C) following an acute MI, the start of an extensive acute inflammatory response takes place with cell infiltration. Debris is left by the necrosis in the previous stage and the cytoplasm is homogeneous so that it is difficult to see the outlines of the myocardial fibres. Infiltration of polymorphs, and later macrophages, takes place. Removal of the debris takes place at about 5–10 days. At approximately 2 weeks (D), the area undergoes repair and a classic young scar with new capillaries is seen with early collagenization. Macrophages and myofibroblasts are present in large numbers. Some granulation tissue and collagen synthesis will continue on to the next stage. In the months (E) following an acute MI, the area starts to strengthen with the formation of a decellularizing scar. This established scar will be seen as a pale white collagenized area within the interstitium between myocardial fibres.
1276
A 41-year-old man presents with severe central chest pain which he describes as ‘tearing’ in nature and radiating to the back. He is tall, with long limbs and long thin fingers. He also has an aortic regurgitation murmur. Histologically there is cystic medial necrosis in the aortic wall. In which syndrome are these findings most likely? A Ortner’s syndrome B Ehlers–Danlos syndrome C Down syndrome D Turner syndrome E Marfan syndrome
Cystic medial necrosis is a disorder particularly affecting the aorta, causing focal degeneration of the elastic tissue and muscle fibres in the media, with accumulation of basophilic ground substance. This leads to cyst-like pools between the fibres disrupting the normally parallel arrays. Clinically, aneurysm formation becomes more likely. It is more frequent after 40 years of age and is twice as common in males. There is evidence that links cystic medial necrosis to aortic dissection in patients with a variety of syndromes, the most common of which is Marfan’s syndrome (E). These patients are characteristically tall with long limbs and long thin fingers. All other syndromes are inconsistent with this patient. The most common murmur in Ortner’s syndrome (A) is mitral stenosis, associated with an enlarged left atrium and recurrent laryngeal nerve palsy. Turner’s syndrome (D) is only present in females. Congenital heart defects include: aortic coarctation, aortic stenosis, ventricular septal defect and atrial septal defect, but aortic dissection is uncommon. In Ehlers–Danlos syndrome (B), cystic medial necrosis can cause fragile blood vessels but aortic regurgitation is not commonly present. Down syndrome (C) patients may have congenital atrioventricular septal canal defects. Other causes of cystic medial necrosis in patients without Marfan’s syndrome are advanced age and chronic hypertension.
1277
The activity of the plaques in a 25-year-old multiple sclerosis patient is described with the presence of oedema and macrophages, and some myelin breakdown. Which ICDNS (International Classification of Diseases of the Nervous System) plaque type classification best fits the description? A Acute plaque B Early chronic active plaque C Late chronic active plaque D Chronic inactive plaque E Shadow plaque
Multiple sclerosis (MS) is the leading cause of disability in Western countries among young individuals between 20 and 40 years of age. The pathological hallmark of MS is the presence of demyelinating plaques on MRI scanning of the central nerves (spinal cord and brain). Particular things to look for are myelin loss, destruction of oligodendrocytes, and reactive astrogliosis, often with relative sparing of the axon cylinder until later stages. The number of plaques, which range from a few to several hundreds, is indicative of the severity of the disease. According to the activity, the International Classification of Diseases classifies plaques as follows; Acute plaque (A): Minor changes (e.g. oedema) and often difficult to recognize Early chronic active plaque (B): Oedema and macrophages, indicative of an inflammatory disorder of the central nervous system, with some myelin breakdown. Reactive astrocytosis is present Late chronic active plaque (C): Complete loss of myelin. Some macrophages will contain myelin debris and there will be often very mild perivascular inflammation at this stage with enlarged perivascular spaces Chronic inactive plaque (D): Complete loss of myelin with the absence of macrophages Shadow plaque (E): Nearly complete remyelination as a thin myelin with some scattered macrophages and a mild microglial up-regulation.
1278
s a degenerative disease with cell death of the dopamine-producing neurons of the substantia nigra, resulting in depigmentation. Furthermore, patients with this condition are alpha-synuclein and ubiquitin positive.
Parkinson's disease
1279
characteristically shows cerebral atrophy in the caudate nucleus and putamen and several changes in neurotransmitters.
Huntingtons
1280
is associated with glial cytoplasmic inclusion bodies (i.e. Papp–Lantos bodies).
MSA
1281
is associated with pernicious anaemia in the elderly and typically affects the body of the stomach.
Autoimmune chronic gastritis
1282
the dominant feature is the epithelial change with minimal inflammation that can be idiopathic or due to reflux of bile-containing duodenal fluid or drugs
Reactive gastritis
1283
is the most common form of chronic gastritis, accounting for 90 per cent of cases, and it is known that the pyloric antrum is the most severely affected area.
H pylori associated gastritis
1284
is a rare disease characterized by gross hyperplasia of gastric pits and a marked increase in mucosal thickness and typically affects the fundus and body of the stomach.
Menetrier's disease
1285
Ninety to ninety-five per cent of patients are HLA DR3 and HLA DR4 positive.
T1DM
1286
A 39-year-old man is diagnosed with a colon cancer proximal to the splenic flexure that is poorly differentiated and highly aggressive. There are no associated adenomata. It is an autosomal dominant condition that involves gene mutations of DNA mismatch repair genes. What is the most likely diagnosis? A Familial adenomatous polyposis B Gardner’s syndrome C Colorectal carcinoma D Hereditary non-polyposis colorectal cancer E Hamartomatous polyps
18 D Hereditary non-polyposis colorectal cancer (HNPCC) (D) is an uncommon autosomal dominant disease but the cancers are poorly differentiated and highly aggressive, therefore screening for identification of carriers for surveillance is necessary. Familial adenomatous polyposis (FAP) (A) is also a rare autosomal dominant condition that is caused by a mutation in the FAP gene on chromosome 5. It is characterized by the presence of adenomata in the large bowel, which this patient did not have, yet also carries a 90 per cent risk of developing into carcinoma by the age of 45. Gardner’s syndrome (B) is similar clinically, pathologically and aetiologically to FAP and also carries a high carcinoma risk. However, there are very distinct extra-intestinal manifestations of Gardner’s syndrome, including multiple osteomas of the skull and mandible, epidermoid cysts and desmoid tumours. Colorectal carcinoma (C) of the sporadic type is inconsistent with the history and age of the patient, as it is more commonly present in patients over the age of 60. Hamartomatous polyps (E) often present in childhood or adolescence as part of Peutz–Jeghers syndrome and these carry a small risk of carcinomas. Patients also tend to have pigmented lesions around the mouth that are characteristic of this condition.
1287
A 32-year-old woman presents with generalized fatigue. Full blood count shows a reduced haemoglobin level and reduced mean corpuscular volume. A peripheral blood film has revealed iron deficiency anaemia. What features are most likely to be seen on her peripheral blood film? A Hypochromic and microcytic red blood cells with anisopoikilocytosis and acanthocytes B Hypochromic and microcytic red blood cells with hypersegmented neutrophils C Hypochromic and microcytic red blood cells with anisopoikilocytosis and no evidence of basophilic stippling D Hypochromic and microcytic red blood cells with Howell–Jolly bodies and basophilic stippling E Hypochromic and macrocytic red blood cells with target cells, acanthocytes and Howell–Jolly bodies
C Features of iron deficiency anaemia are hypochromic (pale) and microcytic (small) red blood cells. Poikilocytes are red blood cells that are abnormally shaped. When there are variations in shape and size, it is known as anisopoikilocytosis. Basophilic stippling (aggregation of ribosomal material) is absent in iron deficiency and present in b- thalassaemia trait and lead poisoning. In megaloblastic anaemia, there is impaired DNA synthesis and this can be caused by B12 deficiency, folate deficiency and drugs. Here the features are the characteristic hypersegmented neutrophils and macrocytic red blood cells. In hyposplenism, there is presence of target cells known as codocytes (red blood cells that have a high surface area:volume ratio). Acanthocytes (spiculated blood cells/spur cells) and Howell–Jolly bodies (nuclear remnants visible in red cells) are also present in the hyposplenism picture.
1288
A 27-year-old woman has developed pain in her right proximal femur. She has a history of intermittent hip pain since childhood. An X-ray has demonstrated a ‘soap bubble’ appearance indicative of osteolysis and a characteristic shepherd’s crook deformity. The biopsy would show irregular trabeculae of woven bone said to resemble Chinese letters. What is the most likely diagnosis? A Non-ossifying fibroma B Fibrous dysplasia C Giant cell reparative granuloma D Ossifying fibroma E Simple bone cyst
24 B Fibrous dysplasia (B) is a benign disorder of children and young adults, whereby lesions composed of fibrous and bony tissue develop usually in the ribs, femur, tibia or skull. It mostly presents as bone pain and weakness in female patients under 30 years of age and results from congenital dysplasia of bone, consistent with the patient’s history. There are three forms: the more common monostotic form (lesions localized to only one bone), polyostotic (multiple lesions) and McCune–Albright syndrome, which also has endocrine manifestations. Shepherd’s crook deformity refers to a varus angulation of the proximal femur commonly seen in the femoral involvement of polyostotic fibrous dysplasia. Histologically, it is characterized by loose fibrous tissue with metaplastic immature or woven bone trabeculae arranged in a ‘Chinese letters’ formation. Non-ossifying fibroma (A) is often asymptomatic and merely an incidental finding on X-ray and occurs in an even younger group of patients. Ossifying fibroma (D) is a benign, fibrous tumour with reactive bone formation that shows local aggressive behaviour. Lesions are found in the mandible for adults, and the tibia for children. Giant-cell reparative granuloma (C) is an uncommon benign reactive intraosseous lesion and can occur in the skull, jaw, hand and foot. The histology resembles giant cell tumour of bone (see below). A simple bone cyst (E) is common in the proximal metaphysis of the humerus and asymptomatic unless there has been a fracture.
1289
A 36-year-old man presents with swelling of his middle finger and subsequently a fracture. His X-ray shows cotton wool calcification and histopathology shows evidence of a tumour composed of benign hyaline cartilage. It is believed that he has only a very slight risk of malignant transformation. What is the most likely diagnosis? A Osteochondroma B Multiple myeloma C Osteoid osteoma D Giant cell tumour E Enchondroma
25 E Enchondroma (E) is a benign intramedullary cartilage tumour usually found in the central mature hyaline cartilage of the short tubular bones of the hands and feet. It may present at any age (average is 40 years) and is often asymptomatic, but some patients present with pain, fracture or swelling of the affected area. There are lytic lesions on X-rays that usually contain variably calcified chondriod matrix and histopathological findings showing bluish-grey lobules of hyaline cartilege. There may be a thin layer of lamellar bone surrounding the cartilage nodules but no permeation of pre-existing host bone which is a positive sign that the lesion is benign. Each potential enchondroma needs to be evaluated on imaging and histology to distinguish it from low-grade chondrosarcoma. Osteochondroma (A) is the most common skeletal neoplasm, often affecting boys in their teens and often found in the long bones. When the growth plates close in late adolescence, there is usually no further growth of the osteochondroma. There is a low risk of malignancy in osteochondromas associated with syndromes that involve multiple lesions such as Ollier’s disease or Maffucci’s syndrome. Osteoid osteoma (C) is a small, benign lesion surrounded by a zone of reactive bone. It mostly occurs in the long bones, with a very distinct clinical picture in that pain is dull, worse at night and relieved by aspirin. Similar radiology and histology are seen in osteoblastomatomours that are larger than 1 cm diameter and that commonly affect the vertebrae. Pathological fracture is a common presentation of giant cell tumour (D), which is more common in females, is locally aggressive and can metastasize. It does not occur in the immature skeleton. It is more common in the metaphysics of the long bones, especially the distal femur and proximal tibia. There is no calcification on X-ray and, histologically, there would be numerous multinucleated giant cells. The mono-nuclear cell population is the neoplastic cells. Multiple myeloma (B) is highly unlikely as it is a different classification of bone tumours altogether and is considered as a blood cell malignancy rather than a primary bone tumour. It is a malignant tumour of plasma cells that causes widespread osteolytic bone damage. It may be in are bone only when it is known as plasmacytoma rather than myeloma.